SlideShare a Scribd company logo
1,5 hour after start of gullet bougienage a 48 year old patient suffering from corrosive stricture felt
acute abdominal pain. Previously he had been suffering from duodenal ulcer. Examination revealed
that abdomen was very tense and painful; Ps- 110 bpm, painful sialophagia, skin pallor. What is the
most probable diagnosis?
Select one:
a. Thrombosis of mesenteric vessels
b. Perforation of duodenal ulcer
c. Strangulation of diaphragmal hernia
d. Perforation of abdominal part of esophagus
e. Acute myocardium infarction
Feedback
The correct answer is: Perforation of abdominal part of esophagus
Question 2
Not answered
Mark 0.00 out of 1.00
Flag question
Question text
15 minutes after the second vaccination with diphteria and tetanus toxoids and pertussis vaccine a 4
month old boy manifested symptoms of Quincke's edema. What medication should be applied for
emergency aid?
Select one:
a. Furosemide
b. Prednisolone
c. Seduxen
d. Heparin
e. Adrenalin
Feedback
The correct answer is: Prednisolone
Question 3
Not answered
Mark 0.00 out of 1.00
Flag question
Question text
25 children at the age of 2-3 years who don't attend any child welfare institutions should be observed
by a district pediatrician within the current year. How many initial visits of this group of children
should be planned?
Select one:
a. 50
b. 100
c. 20
d. 40
e. 200
Feedback
The correct answer is: 50
Question 4
Not answered
Mark 0.00 out of 1.00
Flag question
Question text
A 10 month old boy has been ill for 5 days after consumption of unboiled milk. Body temperature is
38-39oC, there is vomiting, liquid stool. The child is pale and inert. His tongue is covered with white
deposition. Heart sounds are muffled. Abdomen is swollen, there is borborygmus in the region of
ubbilicus, liver is enlarged by 3 cm. Stool is liquid, dark-green, with admixtures of mucus, 5 times a
day. What is the most probable diagnosis?
Select one:
a. Staphylococcal enteric infection
b. Escherichiosis
c. Salmonellosis
d. Acute shigellosis
e. Rotaviral infection
Feedback
The correct answer is: Salmonellosis
Question 5
Not answered
Mark 0.00 out of 1.00
Flag question
Question text
A 10 year old boy complains about pain in his left eye and strong photophobia after he had injured
his left eye with a pencil at school. Left eye examination: blepharospasm, ciliary and conjunctival
congestion, cornea is transparent, other parts of eyeball have no changes. Visus 0,9. Right eye is
healthy, Visus 1,0. What additional method would you choose first of all?
Select one:
a. Tonometria
b. Staining test with 1% fluorescein
c. X-ray examination of orbit
d. Cornea sensation-test
e. Gonioscopia
Feedback
The correct answer is: Staining test with 1% fluorescein
Question 6
Not answered
Mark 0.00 out of 1.00
Flag question
Question text
A 10 year old boy suffers from chronic viral hepatitis type B with maximal activity. What laboratory
test can give the most precise characteristic of cytolysis degree?
Select one:
a. Weltman's coagulation test
b. Takata-Ara test
c. Transaminase test
d. Test for whole protein
e. Prothrombin test
Feedback
The correct answer is: Transaminase test
Question 7
Not answered
Mark 0.00 out of 1.00
Flag question
Question text
A 12 year old child has the ulcer disease of stomach. What is the etiology of this disease?
Select one:
a. Helicobacter pylory
b. Salmonella
c. Lambliosis
d. Intestinal bacillus
e. Influenza
Feedback
The correct answer is: Intestinal bacillus
Question 8
Not answered
Mark 0.00 out of 1.00
Flag question
Question text
A 13 year old girl was admitted to the cardiological department because of pain in the muscles and
joints. Examination of her face revealed an edematic erythema in form of butterfly in the region of
nose bridge and cheeks. What is the most probable diagnosis?
Select one:
a. Periarteritis nodosa
b. Dermatomyositis
c. Systemic lupus erythematosus
d. Rheumatism
e. Rheumatoid arthritis
Feedback
The correct answer is: Systemic lupus erythematosus
Question 9
Not answered
Mark 0.00 out of 1.00
Flag question
Question text
A 13 year old patient is suffering from an acute disease with the following symptoms: thirst, polyuria,
weakness. Objectively: his general condition is satisfactory, there is no smell of acetone. Glucose
concentration in blood on an empty stomach is 32 micromole/l, in urine - 6%, acetone +. What
treatment should be administered?
Select one:
a. Diet
b. Biguanides
c. Long-acting insulin
d. Sulfonylurea
e. Short-acting insulin
Feedback
The correct answer is: Short-acting insulin
Question 10
Not answered
Mark 0.00 out of 1.00
Flag question
Question text
A 14 year old child suffers from vegetovascular dystonia of pubertal period. He has got
sympathoadrenal atack. What medicine should be used for attack reduction?
Select one:
a. Aminophylline
b. Amysyl
c. Corglicone
d. Obsidan
e. No-shpa
Feedback
The correct answer is: Obsidan
Question 11
Not answered
Mark 0.00 out of 1.00
Flag question
Question text
A 14 year old girl complains of profuse bloody discharges from genital tracts during 10 days after
suppresion of menses for 1,5 month. Similiar bleedings recur since 12 years on the background of
disordered menstrual cycle. On rectal examination: no pathology of the internal genitalia. In blood:
Нb - 70 g/l, RBC- 2,3*1012/l, Ht - 20. What is the most probable diagnosis?
Select one:
a. Incomplete spontaneous abortion
b. Hormonoproductive ovary tumor
c. Polycyst ovarian syndrome
d. Juvenile bleeding, posthemorrhagic anemia
e. Werlholf's disease
Feedback
The correct answer is: Juvenile bleeding, posthemorrhagic anemia
Question 12
Not answered
Mark 0.00 out of 1.00
Flag question
Question text
A 15 year old girl suddenly got arthralgia, headache, nausea, vomiting; pain and muscle tension in
the lumbar area; body temperature rose up to 38-39oC. Pasternatsky's symptom was distinctly
positive on the right. In the urine: bacteriuria, pyuria. What is the most probable diagnosis?
Select one:
a. Acute pyelonephritis
b. Acute glomerulonephritis
c. Renal colic
d. Pararenal abscess
e. Cystitis
Feedback
The correct answer is: Acute pyelonephritis
Question 13
Not answered
Mark 0.00 out of 1.00
Flag question
Question text
A 16 year old female patient underwent an operation on account of diffuse toxic goiter of the III-IV
degree 12 years ago. Now she has recurrence of thyrotoxicosis. The patient was offered operative
intervention, but it is necessary first to localize the functioning gland tissue. What method should be
applied for this purpose?
Select one:
a. Roentgenography of esophagus
b. Roentgenography of neck
c. Gland scanning
d. Puncture aspiration biopsy
e. USI
Feedback
The correct answer is: Gland scanning
Question 14
Not answered
Mark 0.00 out of 1.00
Flag question
Question text
A 19 year old boy was admitted to a hospital with closed abdominal trauma. In course of operation
multiple ruptures of spleen and small intestine were revealed. AP is falling rapidly, it is necessary to
perform hemotransfusion. Who can specify the patient's blood group and rhesus compatibility?
Select one:
a. A traumatologist
b. A surgeon
c. A laboratory physician
d. An anaesthesilogist
e. A doctor of any speciality
Feedback
The correct answer is: A doctor of any speciality
Question 15
Not answered
Mark 0.00 out of 1.00
Flag question
Question text
A 19 year old patient was admitted to a hospital with acute destructive appendicitis. He suffers from
hemophilia of B type. What antihemophilic medications should be included in pre- and post-operative
treatment plan?
Select one:
a. Fresh frozen blood
b. Cryoprecipitate
c. Fresh frozen plasma
d. Dried plasma
e. Native plasma
Feedback
The correct answer is: Fresh frozen plasma
Question 16
Not answered
Mark 0.00 out of 1.00
Flag question
Question text
A 2 month old full-term child was born with weight 3500 g and was on the mixed feeding. Current
weight is 4900 g. Evaluate the current weight of the child:
Select one:
a. Hypotrophy of the I grade
b. 150 g less than necessary
c. Paratrophy of the I grade
d. Corresponding to the age
e. Hypotrophy of the II grade
Feedback
The correct answer is: Corresponding to the age
Question 17
Not answered
Mark 0.00 out of 1.00
Flag question
Question text
A 2 year old girl has been ill for 3 days. Today she has low grade fever, severe catarrhal
presentations, slight maculopapular rash on her buttocks and enlarged occipital lymph nodes. What
is your diagnosis?
Select one:
a. Scarlet fever
b. Rubella
c. Adenoviral infection
d. Measles
e. Pseudotuberculosis
Feedback
The correct answer is: Rubella
Question 18
Not answered
Mark 0.00 out of 1.00
Flag question
Question text
A 20 year old patient was delivered to the hospital in summer from the street with haemorrage from
the brachial artery. First medical aid involved aplication of a tourniquet for provisional arrest of
bleeding. What is the maximal exposure of the tourniquet?
Select one:
a. 60 minutes
b. 15 minutes
c. 120 minutes
d. 180 minutes
e. 30 minutes
Feedback
The correct answer is: 120 minutes
Question 19
Not answered
Mark 0.00 out of 1.00
Flag question
Question text
A 22 year old female patient complains about frequent and painful urination, urge to urinate at night,
enuresis, pain in the suprapubic and lumbar area. Her urine often has beer colouring. She got
married a month ago. Objectively: general state is satisfactory. Lung examination revealed vesicular
respiration. Heart sounds are rhythmic, heart rate is 78/min, AP- 128/68 mm Hg. Abdomen is soft,
painful in the suprapubic area. Urine contains 12-18 erythrocytes and 12-15 bacteria within eyeshot.
What is the most probable diagnosis?
Select one:
a. Infection of inferior urinary tracts - cystitis
b. Infection of superior urinary tracts - pyelonephritis
c. Urolithiasis
d. Gonorrhoea
e. Primary syphilis
Feedback
The correct answer is: Infection of inferior urinary tracts - cystitis
Question 20
Not answered
Mark 0.00 out of 1.00
Flag question
Question text
A 22 year old woman complained of right subcostal aching pain, nausea, and decreased appetite.
She fell ill 2 months after appendectomy when jaundice appeared. She was treated in an infectious
hospital. 1 year later above mentioned symptoms developed. On exam: the subicteric sclerae,
enlarged firm liver. Your preliminary diagnosis:
Select one:
a. Chronic viral hepatitis
b. Acute viral hepatitis
c. Gilbert's disease
d. Chronic cholangitis
e. Calculous cholecystitis
Feedback
The correct answer is: Chronic viral hepatitis
Question 21
Not answered
Mark 0.00 out of 1.00
Flag question
Question text
A 23 year old female patient complains about periodical chill and body temperature rise up to 40oC,
sense of heat taking turns with profuse sweating. The patient has had already 3 attacks that came
once in two days and lasted 12 hours. She has lived in Africa for the last 2 months. Liver and spleen
are enlarged. In blood: erythrocytes - 2,5*1012/l. What is the most probable diagnosis?
Select one:
a. Spotted fever
b. Sepsis
c. Malaria
d. Leptospirosis
e. Haemolytic anaemia
Feedback
The correct answer is: Malaria
Question 22
Not answered
Mark 0.00 out of 1.00
Flag question
Question text
A 23 year old patient fell ill 3 weeks ago when she noticed a very painful induration in her axillary
crease. 4-5 days later it burst and discharged a lot of pus. After that some new infiltrations appeared
around the affected area. The patient has never suffered from skin diseases before. What is the
most probable diagnosis?
Select one:
a. Furuncle
b. Mycosis
c. Streptococcal impetigo
d. Herpes zoster
e. Hydradenitis
Feedback
The correct answer is: Hydradenitis
Question 23
Not answered
Mark 0.00 out of 1.00
Flag question
Question text
A 25 year old patient complains about weakness, dizziness, haemorrhagic skin rash. She has been
suffering from this for a month. Blood count: erythrocytes: 1,0*1012/l, Hb- 37 g/l, colour index - 0,9,
leukocytes - 1,2*109/l, thrombocytes - 42*109/l. What diagnostic method will be the most effective?
Select one:
a. Coagulogram
b. Abdominal ultrasound
c. Spleen biopsy
d. Liver biopsy
e. Sternal puncture
Feedback
The correct answer is: Sternal puncture
Question 24
Not answered
Mark 0.00 out of 1.00
Flag question
Question text
A 25 year old patient complains of pain in the I finger on his right hand. On examination: the finger is
homogeneously hydropic, in bent position. On attempt to unbend the finger the pain is getting worse.
Acute pain appears during the probe in ligament projection. What decease is the most likely?
Select one:
a. Thecal whitlow (ligament panaritium)
b. Subcutaneous panaritium
c. Bone panaritium
d. Articular (joint) panaritium
e. Paronychia
Feedback
The correct answer is: Thecal whitlow (ligament panaritium)
Question 25
Not answered
Mark 0.00 out of 1.00
Flag question
Question text
A 25 year old patient had pharyngitis 2 weeks ago. Now he complains about body temperature rise
up to 38oC, general weakness, dyspnea during walking, swelling and shifting pain in the
articulations. Objectively: cyanosis of lips, rhythmic pulse of poor volume - 100 bpm. Left cardiac
border deviates outwards from the mediaclavicular line by 1 cm. The first heart sound is weakened
on the apex, auscultation revealed systolic souffle. What is the most probable aetiological factor that
caused this pathological process?
Select one:
a. Virus
b. Pneumococcus
c. Staphylococcus
d. Fungi
e. Beta-haemolytic streptococcus
Feedback
The correct answer is: Beta-haemolytic streptococcus
Question 26
Not answered
Mark 0.00 out of 1.00
Flag question
Question text
A 25 year old woman applied to a maternity welfare clinic and complained about being unable to
conceive within 3 years of regular sexual life. Examination revealed weight gain, male pattern of hair
distribution on the pubis, excessive pilosis of thighs. Ovaries were dense and enlarged, basal
temperature was monophase. What is the most probable diagnosis?
Select one:
a. Adrenogenital syndrome
b. Sclerocystosis of ovaries
c. Gonadal dysgenesis
d. Tubo-ovaritis
e. Premenstrual syndrome
Feedback
The correct answer is: Sclerocystosis of ovaries
Question 27
Not answered
Mark 0.00 out of 1.00
Flag question
Question text
A 25 year old woman complained of edema on her face and legs, rise of blood pressure up to
160/100 mm Hg and weakness. She fell ill 3 weeks after recovering from angina. Urinalysis data:
protein of 0,5 g/l, erythrocytes of 17-20/field, leukocytes of 2-3/field, erythrocyte casts. What
treatment should be initiated after specifying the diagnosis?
Select one:
a. Dipyridamole
b. Penicillin OS
c. Ceftriaxone
d. Heparin
e. Ciprofloxacine
Feedback
The correct answer is: Penicillin OS
Question 28
Not answered
Mark 0.00 out of 1.00
Flag question
Question text
A 26 year old manual worker complained of 3 weeks history of fevers and fatigue, weight loss with
no other symptoms. Physical findings: Temperature 37,6oC, Ps- 88 bpm, blood pressure 115/70 mm
Hg, superficial lymph nodes (occipital, submental, cervical, axillary) are enlarged, neither tender nor
painful. Rubella-like rash on the trunk and extremities. Herpes simplex lesions on the lips.
Candidosis of oral cavity. What infectious disease would you suspect?
Select one:
a. Rubella
b. Tuberculosis
c. Influenza
d. Infectious mononucleosis
e. HIV infection
Feedback
The correct answer is: HIV infection
Question 29
Not answered
Mark 0.00 out of 1.00
Flag question
Question text
A 26 year old woman complains about edemata, swelling and painfulness of mammary glands,
headache, tearfulness, irritability. These signs turn up 5 days before menstruation and disappear
after its start. What clinical syndrome is it?
Select one:
a. Stein-Leventhal syndrome
b. Adrenogenital syndrome
c. Postcastration syndrome
d. Premenstrual syndrome
e. Climacteric syndrome
Feedback
The correct answer is: Premenstrual syndrome
Question 30
Not answered
Mark 0.00 out of 1.00
Flag question
Question text
A 26 year old woman had the second labour within the last 2 years with oxytocin application. The
child's weight is 4080 g. After the placent birth there were massive bleeding, signs of hemorrhagic
shock. Despite the injection of contractive agents, good contraction of the uterus and absence of any
cervical and vaginal disorders, the bleeding proceeds. Choose the most probable cause of bleeding:
Select one:
a. Injury of cervix of the uterus
b. Hysterorrhexis
c. Delay of the part of placenta
d. Hypotonia of the uterus
e. Atony of the uterus
Feedback
The correct answer is: Atony of the uterus
Question 31
Not answered
Mark 0.00 out of 1.00
Flag question
Question text
A 26 year old woman who delivered a child 7 months ago has been suffering from nausea, morning
vomiting, sleepiness for the last 2 weeks. She suckles the child, menstruation is absent. She hasn't
applied any contraceptives. What method should be applied in order to specify her diagnosis?
Select one:
a. Bimanual vaginal examination
b. Palpation of mammary glands and pressing-out of colostrum
c. Ultrasonic examination
d. Roentgenography of small pelvis organs
e. Speculum examination
Feedback
The correct answer is: Ultrasonic examination
Question 32
Not answered
Mark 0.00 out of 1.00
Flag question
Question text
A 27 year old patient suffers from haemophilia. He was admitted to the hospital with melena and skin
pallor. Objectively: Ps- 110 bpm, AP- 100/60 mm Hg. In blood: Hb- 80 g/l, erythrocytes - 2,8*1012/l.
What medication should be administered in the first place?
Select one:
a. Dicinone
b. Stored blood
c. Cryoprecipitate
d. Packed red blood cells
e. Epsilon-aminocapronic acid
Feedback
The correct answer is: Cryoprecipitate
Question 33
Not answered
Mark 0.00 out of 1.00
Flag question
Question text
A 28 year old parturient complains about headache, vision impairment, psychic inhibition.
Objectively: AP- 200/110 mm Hg, evident edemata of legs and anterior abdominal wall. Fetus head
is in the area of small pelvis. Fetal heartbeats is clear, rhythmic, 190/min. Internal investigation
revealed complete cervical dilatation, fetus head was in the area of small pelvis. What tactics of
labor management should be chosen?
Select one:
a. Forceps operation
b. Stimulation of labor activity
c. Conservative labor management with episiotomy
d. Embryotomy
e. Cesarean
Feedback
The correct answer is: Forceps operation
Question 34
Not answered
Mark 0.00 out of 1.00
Flag question
Question text
A 28 year old patient was admitted to the clinic with complaints of the temperature rise up to 39,0oC,
headache, weakness, constipation on the 9th day of the disease. On examination: single roseolas
on the skin of the abdomen are present. The pulse rate is 78 bpm. The liver is enlarged by 2 cm.
What is the most probable diagnosis?
Select one:
a. Brucellosis
b. Sepsis
c. Malaria
d. Typhoid fever
e. Leptospirosis
Feedback
The correct answer is: Typhoid fever
Question 35
Not answered
Mark 0.00 out of 1.00
Flag question
Question text
A 28 year old woman has bursting pain in the lower abdomen during menstruation; chocolate-like
discharges from vagina. It is known from the anamnesis that the patient suffers from chronic
adnexitis. Bimanual examination revealed a tumour-like formation of heterogenous consistency 7х7
cm large to the left from the uterus. The formation is restrictedly movable, painful when moved. What
is the most probable diagnosis?
Select one:
a. Tumour of sigmoid colon
b. Fibromatous node
c. Follicular cyst of the left ovary
d. Endometrioid cyst of the left ovary
e. Exacerbation of chronic adnexitis
Feedback
The correct answer is: Endometrioid cyst of the left ovary
Question 36
Not answered
Mark 0.00 out of 1.00
Flag question
Question text
A 29 year old female patient complains about periodical right-sided headache that is usually
provoked by strong smells or excitement. The patient's mother suffers from the same disease.
Objectively: examination of internal organs revealed no pathology. During the attack there are
general hyperesthesia and nausea, at the end of the attack polyuria is observed. Palpation of the
right temporal artery during the attack revealed tension and painfulness of the artery. Complete
blood count and urine analysis reveale dno pecularities. What is the most probable diagnosis?
Select one:
a. Neuritis of the facial nerve
b. Epilepsy
c. Migraine
d. Neuralgia of trigeminus
e. Meniere's syndrome
Feedback
The correct answer is: Migraine
Question 37
Not answered
Mark 0.00 out of 1.00
Flag question
Question text
A 3 month old infant suffering from acute segmental pneumonia has dyspnea (respiration rate - 80
per minute), paradoxical breathing, tachycardia, total cyanosis. Respiration and pulse - ratio is 1:2.
The heart dullness under normal size. Such signs characterise:
Select one:
a. Respiratory failure of I degree
b. Respiratory failure of II degree
c. Respiratory failure of III degree
d. Congenital heart malformation
e. Myocarditis
Feedback
The correct answer is: Respiratory failure of III degree
Question 38
Not answered
Mark 0.00 out of 1.00
Flag question
Question text
A 3 year old boy fell ill abruptly: fever up to 39oC, weakness, vomitng. Haemorrhagic rash of various
size appeared on his lower limbs within 5 hours. Meningococcemia with infective - toxic shock of the
1 degree was diagnosed. What medications should be administered?
Select one:
a. Chloramphenicol succinate and interferon
b. Ampicillin and immunoglobulin
c. Penicillin and prednisone
d. Penicillin and immunoglobulin
e. Chloramphenicol succinate and prednisone
Feedback
The correct answer is: Chloramphenicol succinate and prednisone
Question 39
Not answered
Mark 0.00 out of 1.00
Flag question
Question text
A 3 year old boy has petechial eruption. Examination revealed no other pathological changes.
Thrombocyte number is 20*109g/l; haemoglobin and leukocyte concentration is normal. What is the
most probable diagnosis?
Select one:
a. Schonlein-Henoch disease
b. Acute lymphoblastic leukemia
c. Immune thrombocytopenic purpura
d. Disseminated intravascular coagulopathy
e. Systemic lupus erythematosus
Feedback
The correct answer is: Immune thrombocytopenic purpura
Question 40
Not answered
Mark 0.00 out of 1.00
Flag question
Question text
A 3 year old child has been suffering from fever, cough, coryza, conjunctivitis for 4 days. He has
been taking sulfadimethoxine. Today it has fever up to 39oC and maculopapular rash on its face.
Except of rash the child's skin has no changes. What is your diagnosis?
Select one:
a. Scarlet fever
b. Allergic rash
c. Pseudotuberculosis
d. Measles
e. Rubella
Feedback
The correct answer is: Measles
Question 41
Not answered
Mark 0.00 out of 1.00
Flag question
Question text
A 3 year old child with weight deficiency suffers from permanent moist cough. In history there are
some pneumonias with obstruction. On examination: distended chest, dullness on percussion over
the lower parts of lungs. On auscultation: a great number of different rales. Level of sweat chloride is
80 millimol/l. What is the most probable diagnosis?
Select one:
a. Bronchial asthma
b. Mucoviscidosis (cystic fibrosis)
c. Bronchiectasis
d. Recurrent bronchitis
e. Pulmonary hypoplasia
Feedback
The correct answer is: Mucoviscidosis (cystic fibrosis)
Question 42
Not answered
Mark 0.00 out of 1.00
Flag question
Question text
A 30 year old man complains of intense pain, reddening of skin, edema in the ankle-joint area, fever
up to 39oC. There was an acute onset of the illness. In the past there were similar attacks lasting 5-6
days without residual changes in the joint. The skin over the joint is hyperemic and ill-defined,
without infiltrative bank on the periphery. What is the most likely diagnosis?
Select one:
a. Infectious arthritis
b. Erysipelatous inflammation
c. Rheumatoid arthritis
d. Gout
e. Osteoarthritis
Feedback
The correct answer is: Gout
Question 43
Not answered
Mark 0.00 out of 1.00
Flag question
Question text
A 30 year old patient complains about inability to become pregnant over 3 years of married life. The
patient is of supernutrition type, she has hair along the median abdominal line, on the internal thigh
surface and in the peripapillary area. Menses started at the age of 16, they are infrequent and non-
profuse. US revealed that the uterus was of normal size, ovaries were 4х5х5 cm large and had a lot
of cystic inclusions. What is the most probable diagnosis?
Select one:
a. Polycystic ovaries
b. Ovarian cystoma
c. Bilateral ovarian tumours
d. Menstrual irregularity
e. Chronic oophoritis
Feedback
The correct answer is: Polycystic ovaries
Question 44
Not answered
Mark 0.00 out of 1.00
Flag question
Question text
A 32 year old patient complains about cardiac irregularities, dizziness, dyspnea at physical stress.
He has never suffered from this before. Objectively: Ps- 74 bpm, rhythmic. AP- 130/80 mm Hg.
Auscultation revealed systolic murmur above aorta, the first heart sound was normal. ECG showed
hypertrophy of the left ventricle, signs of repolarization disturbance in the I, V5 and V6 leads.
Echocardiogram revealed that interventricular septum was 2 cm. What is the most probable
diagnosis?
Select one:
a. Hypertrophic cardiomyopathy
b. Aortic stenosis
c. Essential hypertension
d. Coarctation of aorta
e. Myocardium infarction
Feedback
The correct answer is: Hypertrophic cardiomyopathy
Question 45
Not answered
Mark 0.00 out of 1.00
Flag question
Question text
A 32 year old patient suffering from chronic viral hepatitis complains about dull pain in the right
subcostal area, nausea, dry mouth. Objectively: liver dimensions are 13-21-11 cm (according to
Kurlov), spleen is by 2 cm enlarged, aspartate aminotransferase is 3,2 micromole/l*h, alanine
aminotransferase - 4,8 millimole/l*h. Serological study revealed HBeAg, high concentration of DNA
HBV. What drug should be chosen for treatment of this patient?
Select one:
a. Acyclovir
b. Arabinoside monophosphate
c. Essentiale-forte
d. a-interferon
e. Remantadinum
Feedback
The correct answer is: a-interferon
Question 46
Not answered
Mark 0.00 out of 1.00
Flag question
Question text
A 32 year old welder complains of weakness and fever. His illness started as tonsillitis a month
before. On exam, BT of 38,9oC, RR of 24/min, HR of 100/min, BP of 100/70 mm Hg, hemorrhages
on the legs, enlargement of the lymph nodes. CBC shows Hb of 70 g/l, RBC of 2,2*1012/l, WBC of
3,0*109/l with 32% of blasts, 1% of eosinophiles, 3% of bands, 36% of segments, 20% of
lymphocytes, and 8% of monocytes, ESR of 47 mm/h. What is the cause of anemia?
Select one:
a. Aplastic anema
b. Chronic lympholeukemia
c. Vitamin B12 deficiency anemia
d. Chronic hemolytic anemia
e. Acute leukemia
Feedback
The correct answer is: Acute leukemia
Question 47
Not answered
Mark 0.00 out of 1.00
Flag question
Question text
A 33 year old patient has acute blood loss (erythrocytes - 2,2*1012/l, Hb- 55 g/l), blood group is
A(II)Rh+. Accidentally the patient got transfusion of donor packed red blood cells of AB(IV)Rh+
group. An hour later the patient became anxious, got abdominal and lumbar pain. Ps - 134 bpm, AP-
100/65 mm Hg, body temperature - 38,6oC. After catheterization of urinary bladder 12 ml/h of dark-
brown urine were obtained. What complication is it?
Select one:
a. Cardial shock
b. Acute renal insufficiency
c. Allergic reaction to the donor red blood cells
d. Toxic infectious shock
e. Citrate intoxication
Feedback
The correct answer is: Acute renal insufficiency
Question 48
Not answered
Mark 0.00 out of 1.00
Flag question
Question text
A 33 year old patient was delivered to the infectious diseases department on the 7-th day of disease.
He complained about great weakness, high temperature, pain in the lumbar area and leg muscles,
icteritiousness, dark colour of urine, headache. The acute disease started with chill, body
temperature rise up to 40oC, headache, pain in the lumbar area and sural muscles. Icterus turned
up on the 4th day, nasal and scleral haemorrhages came on the 5th day. Fever has lasted for 6
days. Diuresis - 200 ml. What is the most probable diagnosis?
Select one:
a. Typhoid fever
b. Virus A hepatitis
c. Yersiniosis
d. Leptospirosis
e. Sepsis
Feedback
The correct answer is: Leptospirosis
Question 49
Not answered
Mark 0.00 out of 1.00
Flag question
Question text
A 34 year old female patient has been suffering from anxious depression accompanied by panic
attacks for 2 years. She has been a patient of a psychotherapist. Treatment resulted in incomplete
remission. The patient had to break off psychotherapy because of moving to a new place of
residence. Soon after this her condition grew significantly worse, she was almost permanently
anxious, panic attacks turned up 5-6 times a day and were accompanied by palpitation, dyspnea,
cold sweat, thanatophobia. What drug group is the most appropriate for medicamental therapy?
Select one:
a. Cardiotonics, respiratory analeptics
b. Sedative neuroleptics
c. Lithium drugs
d. Antipsychotic neuroleptics
e. Antidepressants
Feedback
The correct answer is: Antidepressants
Question 50
Not answered
Mark 0.00 out of 1.00
Flag question
Question text
A 34 year old patient complains of profuse sweating at night, skin itching, weight loss (9 kg within the
last 3 months). Examination revealed malnutrition, skin pallor. Palpation of neck and inguinal areas
revealed dense elastic lymph nodes for about 1 cm in diameter, nonmobile, non-adhering to skin.
What is the most probable diagnosis?
Select one:
a. Lymphogranulomatosis
b. Lymphosarcoma
c. Chronic lymphadenitis
d. Burkitt's lymphoma
e. Cancer metastases
Feedback
The correct answer is: Lymphogranulomatosis
Question 51
Not answered
Mark 0.00 out of 1.00
Flag question
Question text
A 34 year old patient was delivered to the hospital because of follicular tonsillitis charactirized by
high temperature. The patient has been abusing alcohol for 12 years. In the evening on the day of
hospitalization he became anxious, couldn't stay in bed, left his ward several times and applied to
the staff on duty with different complaints. He reported about seeing alot of spiders and flies in his
ward as well as abou hearing threats from the corridor. He was exasperated by the fact that other
patients didn't hear them. He lost also spatial orientation. What psychopathological syndrome is it?
Select one:
a. Asthenic confusion
b. Delirious
c. Amentive
d. Twilight state
e. Oneiric
Feedback
The correct answer is: Delirious
Question 52
Not answered
Mark 0.00 out of 1.00
Flag question
Question text
A 35 year old female patient suffering from cholelithiasis has broken her diet, and this caused an
acute pain attack in the right subcostal are. The pain eased off on the third day, but the patient got
progressing jaundice. What non-invasive diagnostic method should be applied?
Select one:
a. Duodenal probing
b. Endoscopic retrograde cholangiopancreatography
c. Infusive cholecystocholangiography
d. Survey radiography of abdominal organs
e. Test for bilirubin
Feedback
The correct answer is: Endoscopic retrograde cholangiopancreatography
Question 53
Not answered
Mark 0.00 out of 1.00
Flag question
Question text
A 36 year old female patient complains about general weakness, edemata of her face and hands,
rapid fatigability during walking, difficult diglutition, cardiac irregularities. These symptoms turned up
11 days after holiday at the seaside. Objectively: face erythema, edema of shin muscles. Heart
sounds are muffled, AP is 100/70 mm Hg. In blood: ASAT activity is 0,95 millimole/h*l, ALAT - 1,3
micromole/h*l, aldolase - 9,2 IU/l, creatine phosphokinase - 2,5 micromole Р/g*l. What method of
study would be the most specific?
Select one:
a. Echocardiogram
b. ECG
c. Muscle biopsy
d. Determination of cortisol concentration in blood and urine
e. Electromyography
Feedback
The correct answer is: Muscle biopsy
Question 54
Not answered
Mark 0.00 out of 1.00
Flag question
Question text
A 36 year old man was delivered to the surgical department an hour after a road accident. His
condition is getting worse: respiratory insufficiency is progressing, there are cardiac abnormalities.
Clinical and roentgenological investigations revealed mediastinal displacement. What process has
caused this complication?
Select one:
a. Valvular pneumothorax
b. Closed pneumothorax
c. Subcutaneous emphysema
d. Open pneumothorax
e. Mediastinitis
Feedback
The correct answer is: Valvular pneumothorax
Question 55
Not answered
Mark 0.00 out of 1.00
Flag question
Question text
A 36 year old patient was diagnosed with right-sided pneumothorax. What method of treatment is
indicated to the patient?
Select one:
a. Symptomatic therapy
b. Pleural puncture
c. Surgical treatment: drainage of the pleural cavity
d. Thoracotomy
e. Antiinflammation therapy
Feedback
The correct answer is: Surgical treatment: drainage of the pleural cavity
Question 56
Not answered
Mark 0.00 out of 1.00
Flag question
Question text
A 37 year old male patient was admitted to the resuscitation department because of attacks of
tonoclonic spasms repeating every half an hour. Between the attacks the patient remains
unconscious. AP is 120/90 mm Hg, Ps- 100 bpm. A day before the patient was at wedding and
consumed alcohol. 5 years ago he had a closed craniocerebral trauma and brain contusion that later
caused single convulsive attacks accompanied by loss of consciousness, but the patient didn't
undergo antiepileptic treatment. What drug should be injected for emergency aid?
Select one:
a. Sodium oxybutyrate
b. Magnesium sulfate
c. Diazepam
d. Sodium thiopental
e. Aminazine
Feedback
The correct answer is: Diazepam
Question 57
Not answered
Mark 0.00 out of 1.00
Flag question
Question text
A 37 year old miner has lifted significant loads and afterwards felt pain in the lumbar area irradiating
to his left leg. He walks slowly and carefully. Lumbar lordosis is flattened. There is also left-sided
scoliosis and tension of paravertebral muscles. Neri's and Dejerine's symptoms are positive, there is
Lasegue's sign on the left from the angle of 350. What method will help to specify the diagnosis?
Select one:
a. Descending myelography
b. CT of lumbosacral part of vertebral column
c. Electromyography
d. Renal sonography
e. Lumbal puncture
Feedback
The correct answer is: CT of lumbosacral part of vertebral column
Question 58
Not answered
Mark 0.00 out of 1.00
Flag question
Question text
A 37 year old patient applied to a local therapeutist. As a result of exacerbation of chronic
obstructive bronchitis the patient had been temporarily disabled for 117 days within 1 year. What
tactics will be legally correct?
Select one:
a. The therapeutist should issue a new medical certificate
b. The patient should be referred to the medicosocial expertise
c. The patient should be referred to the medical consultation comission for extension of medical
certificate
d. The therapeutist should extend a medical certificate
e. The patient shoul be referred to the sanatorium-and-spa treatment
Feedback
The correct answer is: The patient should be referred to the medicosocial expertise
Question 59
Not answered
Mark 0.00 out of 1.00
Flag question
Question text
A 38 year old female patient complains about body stiffness in the morning, especially in the
articulations of her upper and lower limbs, that disappears 30-60 minutes later after active
movements. She has also arthritis of metacarpophalangeal and proximal phalangeal articulations,
subfebrile temperature. ESR- 45 mm/h. Roentgenography revealed osteoporosis and erosion of
articular surface of small hand and foot articulations. What is the most probable diagnosis?
Select one:
a. Reactive polyarthritis
b. Rheumatoid arthritis
c. Psoriatic arthropathy
d. Systemic lupus erythematosus
e. Osteoarthrosis deformans
Feedback
The correct answer is: Rheumatoid arthritis
Question 60
Not answered
Mark 0.00 out of 1.00
Flag question
Question text
A 38 year old man was delivered to the hospital in unconscious state. The symptoms of illness
turned up a day before: headache, nausea, vomiting, to- 38,5oC, dizziness, delusion. For the last 4
days he had been complaining of pain and hearing loss in the left ear. Objectively: sopor, rigidity of
occipital muscles, bilateral Kernig's symptom, general hyperesthesia, purulent discharges from the
left ear. What is the most probable diagnosis?
Select one:
a. Parenchymatous subarachnoidal haemorrhage
b. Tuberculous meningitis
c. Primary purulent meningitis
d. Subarachnoidal haemorrhage
e. Secondary purulent meningitis
Feedback
The correct answer is: Secondary purulent meningitis
Question 61
Not answered
Mark 0.00 out of 1.00
Flag question
Question text
A 38 year old man worked at roofing and drain pipes production for 15 years. He seeks medical help
for expiratory breathlessness on exertion, and dry cough. On exam, wheezes above both lungs,
grayish warts on fingers are seen. Factory physician has diagnosed asbestosis. What method is the
most important for this diagnosis?
Select one:
a. Bronchoscopy
b. Chest X-ray
c. Electrocardiography
d. Blood gas analysis
e. Spirography
Feedback
The correct answer is: Chest X-ray
Question 62
Not answered
Mark 0.00 out of 1.00
Flag question
Question text
A 38 year old man, previously in good health, suddenly develops severe abdominal pain radiating
from the left loin to groin and accompanied by nausea, perspiration and the need for frequent
urination. He is restless, tossing in bed but has no abnormal findings. The most likely diagnosis is:
Select one:
a. Herpes zoster
b. Leftsided renal colic
c. Sigmoid diverticulitis
d. Torsion of the left testicle
e. Retroperitoneal haemorrhage
Feedback
The correct answer is: Leftsided renal colic
Question 63
Not answered
Mark 0.00 out of 1.00
Flag question
Question text
A 38 year old patient complains about inertness, subfebrile temperature, enlargement of lymph
nodes, nasal haemorrhages, bone pain. Objectively: the patient's skin and mucous membranes are
pale, palpation revealed enlarged painless lymph nodes; sternalgia; liver was enlarged by 2 cm,
spleen - by 5 cm, painless. In blood: erythrocytes - 2,7*1012/l, Hb- 84 g/l, leukocytes – 58*109/l,
eosinophils - 1%, stab neutrophils - 2%, segmented neutrophils - 12%, lymphocytes - 83%,
lymphoblasts - 2%, smudge cells; ESR- 57 mm/h. What is the most probable diagnosis?
Select one:
a. Acute myeloleukemia
b. Lymphogranulomatosis
c. Chronic myeloleukemia
d. Acute lymphatic leukemia
e. Chronic lymphatic leukemia
Feedback
The correct answer is: Chronic lymphatic leukemia
Question 64
Not answered
Mark 0.00 out of 1.00
Flag question
Question text
A 39 year old patient complained about morning headache, appetite loss, nausea, morning vomiting,
periodic nasal haemorrhages. The patient had acute glomerulonephritis at the age of 15.
Examination revealed rise of arterial pressure up to 220/130 mm Hg, skin haemorrhages on his arms
and legs, pallor of skin and mucous membranes. What biochemical index has the greatest
diagnostic importance in this case?
Select one:
a. Blood sodium
b. Fibrinogen
c. Blood creatinine
d. Uric acid
e. Blood bilirubin
Feedback
The correct answer is: Blood creatinine
Question 65
Not answered
Mark 0.00 out of 1.00
Flag question
Question text
A 39 year old patient suffering from hypertension suddenly gotb intense headache in the region of
occiput; there appeared recurrent vomiting. These presentations has been lasting for 5 hours.
Objectively: Ps - 88 bpm, AP - 205/100 mm Hg, painfulness of occipital points, rigidity of occipital
muscles are present. Kernig's symptom is bilaterally positive. Subarachnoid haemorrhage is also
suspected. What diagnostic method will be of the greatest importance for confirmation of provisional
diagnosis?
Select one:
a. Lumbar puncture
b. Examination of eye fundus
c. Computer tomography
d. EEG
e. Ultrasonic dopplerography
Feedback
The correct answer is: Lumbar puncture
Question 66
Not answered
Mark 0.00 out of 1.00
Flag question
Question text
A 4 month old child fell seriously ill: body temperature rose up to 38,5oC, the child became inert and
had a single vomiting. 10 hours later there appeared rash over the buttocks and lower limbs in form
of petechiae, spots and papules. Some haemorrhagic elements have necrosis in the centr E. What is
the most probable disease?
Select one:
a. Meningococcemia
b. Scarlet fever
c. Haemorrhagic vasculitis
d. Influenza
e. Rubella
Feedback
The correct answer is: Meningococcemia
Question 67
Not answered
Mark 0.00 out of 1.00
Flag question
Question text
A 4 month old child was admitted to a surgical department 8 hours after the first attack of anxiety.
The attacks happen every 10 minutes and last for 2-3 minutes, there was also one-time vomiting.
Objectively: the child's condition is grave. Abdomen is soft, palpation reveals a tumour-like formation
in the right iliac area. After rectal examination the doctor's finger was stained with blood. What is the
most probable diagnosis?
Select one:
a. Ileocecal invagination
b. Pylorostenosis
c. Wilm's tumour
d. Helminthic invasion
e. Gastrointestinal haemorrhage
Feedback
The correct answer is: Ileocecal invagination
Question 68
Not answered
Mark 0.00 out of 1.00
Flag question
Question text
A 4 year old girl was playing with her toys and suddenly she got an attack of cough, dyspne A.
Objectively: respiration rate - 45/min, heart rate - 130/min. Percussion revealed dullness of percutory
sound on the right in the lower parts. Auscultation revealed diminished breath sounds with bronchial
resonance on the right. X-ray pictue showed shadowing of the lower part of lungs on the right. Blood
analysis revealed no signs of inflammation. The child was diagnosed with foreign body in the right
bronchus. What complication caused such clinical presentations?
Select one:
a. Atelectasis
b. Bronchitis
c. Pneumonia
d. Pneumothorax
e. Emphysema
Feedback
The correct answer is: Atelectasis
Question 69
Not answered
Mark 0.00 out of 1.00
Flag question
Question text
A 40 year old female patient has been observing excessive menstruation accompanied by
spasmodic pain in the lower abdomen for a year. Bimanual examination performed during
menstruation revealed a dense formation up to 5 cm in diameter in the cervical canal. Uterus is
enlarged up to 5-6 weeks of pregnancy, movable, painful, of normal consistency. Appendages are
not palpabl E. Bloody discharges are profus E. What is the most probable diagnosis?
Select one:
a. Algodismenorrhea
b. Abortion in progress
c. Nascent submucous fibromatous node
d. Cervical carcinoma
e. Cervical myoma
Feedback
The correct answer is: Nascent submucous fibromatous node
Question 70
Not answered
Mark 0.00 out of 1.00
Flag question
Question text
A 40 year old patient was bitten by a stray dog for about an hour ago. The bite can be seen on the
patient's left shin in form of a wound 4х2х0,5 cm large. What kind of aid would be recommended in
this case?
Select one:
a. Retension sutures
b. Wound lavage with soapsuds, retension sutures
c. Aseptic bandage
d. Blind suture
e. Salve bandage
Feedback
The correct answer is: Wound lavage with soapsuds, retension sutures
Question 71
Not answered
Mark 0.00 out of 1.00
Flag question
Question text
A 40 year old woman has a self-detected hard breast mass. The procedure of choice for confirming
the diagnosis is:
Select one:
a. Thermography
b. Excision biopsy
c. Mammography
d. Ultrasonography
e. Aspiration biopsy with cytology
Feedback
The correct answer is: Excision biopsy
Question 72
Not answered
Mark 0.00 out of 1.00
Flag question
Question text
A 41 year old woman has suffered from nonspecific ulcerative colitis for 5 years. On
rectoromanoscopy: evident inflammatory process of lower intestinal parts, pseudopolyposive
changes of mucous membrane. In blood: WBC- 9,8*109/l, RBC- 3,0*1012/l, ESR - 52 mm/hour.
What medication provides pathogenetic treatment of this patient?
Select one:
a. Motilium
b. Vikasolum
c. Kreon
d. Linex
e. Sulfosalasine
Feedback
The correct answer is: Sulfosalasine
Question 73
Not answered
Mark 0.00 out of 1.00
Flag question
Question text
A 42 year old man applied to a hospital 10 minutes after he got stung by a bee and complained
about face edema and difficult respiration. Objectively: Ps- 98 bpm, AP- 130/80 mm Hg. A doctor on
duty injected him 1 ml of 1% dimedrol solution intramuscularly and recommended to apply to his
local therapeutist on the next day. What tactics of treatment should be chosen for this patient?
Select one:
a. Hospitalization for observation
b. The patient needs no further medical aid
c. Intravenous introduction of calcium chloride
d. Intravenous introduction of prednisolone and hospitalization
e. Intravenous introduction of calcium chloride and hospitalization
Feedback
The correct answer is: Intravenous introduction of prednisolone and hospitalization
Question 74
Not answered
Mark 0.00 out of 1.00
Flag question
Question text
A 42 year old man works in a boiler room. He complains about girdle headache and recurring
vomiting. There was also short-term consciousness loss. Objectively: increase of tendon reflexes,
spontaneous myofibrillations. AP is 150/80 mm Hg, Ps- 104 bpm. Visible mucous membranes and
cutaneous surfaces have crimson colouring. What is the most probable diagnosis?
Select one:
a. Poisoning with carbon monooxide
b. Poisoning with hydrocyanic acid
c. Poisoning with anilin colouring agents
d. Poisoning with methane
e. Poisoning with benzine
Feedback
The correct answer is: Poisoning with carbon monooxide
Question 75
Not answered
Mark 0.00 out of 1.00
Flag question
Question text
A 42 year old woman complains of dyspnea, edema of the legs and tachycardia during minor
physical exertion. Heart borders are displaced to the left and S1 is accentuated, there is diastolic
murmur on apex. The liver is enlarged by 5 cm. What is the cause of heart failure?
Select one:
a. Tricuspid stenosis
b. Tricuspid regurgitation
c. Mitral stenosis
d. Mitral regurgitation
e. Aortic stenosis
Feedback
The correct answer is: Mitral stenosis
Question 76
Not answered
Mark 0.00 out of 1.00
Flag question
Question text
A 43 year old female patient was delivered to the hospital in grave condition. She suffers from
Addison's disease. The patient had been regularly taking prednisolone but a week before she
stopped taking this drug. Objectively: sopor, skin and visible mucous membranes are pigmented,
skin and muscle turgor is lowered. Heart sounds are muffled, rapid. AP- 60/40 mm Hg, heart rate -
96/min. In blood: Na- 120 millimole/l, K- 5,8 micromole/l. Development of this complication is
primarily caused by the deficit of the following hormone:
Select one:
a. Cortisol
b. Noradrenaline
c. Corticotropin (ACTH)
d. Adrostendion
e. Adrenaline
Feedback
The correct answer is: Cortisol
Question 77
Not answered
Mark 0.00 out of 1.00
Flag question
Question text
A 43 year old patient was admitted to the infectious diseases hospital with high body temperature
and intense headache. The iIlness has lasted for 2 days. Examination revealed a carbuncle on his
forearm. The area around it was apparently edematic and slightly painful. Regional lymphadenitis
and hepatolienal syndrome were also present. It is known from the anamnesis that the patient works
at a cattle-breeding farm. What disease should be suspected in the first place?
Select one:
a. Anthrax
b. Eczema
c. Erysipeloid
d. Skin cancer
e. Erysipelas
Feedback
The correct answer is: Anthrax
Question 78
Not answered
Mark 0.00 out of 1.00
Flag question
Question text
A 44 year old man has been working in coke industry for 16 years. Dust concentration at his
workplace is 5-10 times more than maximum permissible concentration. Roentgenography of lungs
revealed changes that are typical for pneumoconiosis. What is the most probable type of
pneumoconiosis in this case?
Select one:
a. Anthracosis
b. Silicatosis
c. Anthracosilicosis
d. Asbestosis
e. Siderosis
Feedback
The correct answer is: Anthracosis
Question 79
Not answered
Mark 0.00 out of 1.00
Flag question
Question text
A 45 year old woman complains about unbearable pain attacks in the left part of face lasting 1-2
minutes. Such attacks are provoked by mastication. These symptoms appeared 2 months ago after
exposure to cold. Objectively:the pain is localized in the ending points of trigeminus on the left. A
touch near nose wing provokes another pain attack and tonic spasm of face muscles. What is the
most probable diagnosis?
Select one:
a. Facial migraine
b. Arthritis of mandibular joint
c. Neuralgia of trigeminus
d. Neuralgia of glossopharyngeal nerve
e. Maxillary sinusitis
Feedback
The correct answer is: Neuralgia of trigeminus
Question 80
Not answered
Mark 0.00 out of 1.00
Flag question
Question text
A 46 year old patient is to be prepared to the operation on account of stomach cancer. Preoperative
preparation involves infusion therapy. It was injected up to 3 l of solutions into his right lunar vein. On
the next day he got tensive pain in the region of his right shoulder. Examination of interior brachial
surface revealed an oblong area of hyperemia, skin edema and painful cord. What complication is it?
Select one:
a. Phlegmon of paravenous cellular tissue
b. Acute lymphangitis
c. Necrosis of paravenous cellular tissue
d. Acute thrombophlebitis
e. Vein puncture and edema of paravenous cellular tissue
Feedback
The correct answer is: Acute thrombophlebitis
Question 81
Not answered
Mark 0.00 out of 1.00
Flag question
Question text
A 47 year old male patient got a flame burn of trunk and upper extremities and was delivered to the
hospital. The patient is in grave condition, confused mental state, with fever. AP- 80/50 mm Hg, Ps-
118 bpm. It was locally stated that the patient got III B degree burns with total area of 20%. What
medical actions should be taken?
Select one:
a. Necrotomy of burn surface, haemotransfusion
b. Antibacterial and detoxicating therapy
c. Administration of detoxicating blood substitutes
d. Injection of narcotic analgetics and powdered blood substitutes
e. Primary surgical pocessing
Feedback
The correct answer is: Injection of narcotic analgetics and powdered blood substitutes
Question 82
Not answered
Mark 0.00 out of 1.00
Flag question
Question text
A 48 year old female patient complains about contact haemorrhage. Speculum examination revealed
hypertrophy of uterus cervix. It resembles of cauliflower, it is dense and can be easily injured.
Bimanual examination revealed that fornices were shortened, uterine body was nonmobile. What is
the most probable diagnosis?
Select one:
a. Cervical pregnancy
b. Endometriosis
c. Metrofibroma
d. Cervical carcinoma
e. Cervical papillomatosis
Feedback
The correct answer is: Cervical carcinoma
Question 83
Not answered
Mark 0.00 out of 1.00
Flag question
Question text
A 5 month old boy was born prematurely, he didn't suffer from any disease at the infant age and later
on. Examination at an outpatient's hospital revealed paleness of skin, sleepiness. Blood count: Hb -
95 g/l, erythrocytes - 3,5*1012/l, reticulocytes - 90/00, colour index - 0,7, osmotic stability of
erythrocytes - 0,44-0,33%, serum iron - 4,9 micromole/l. What is the most probable cause of
anemia?
Select one:
a. Hemogenesis immaturity
b. Iron deficit
c. Erythrocyte hemolysis
d. B12 deficit
e. Infectious process
Feedback
The correct answer is: Iron deficit
Question 84
Not answered
Mark 0.00 out of 1.00
Flag question
Question text
A 50 year old locksmith was diagnosed with typhoid fever. The patient lives in a separate apartment
with all facilities. Apart of him there are also 2 adults in his family. What actions should be taken
about persons communicating with the patient?
Select one:
a. Antibiotic prophylaxis
b. Vaccination
c. Isolation
d. Bacteriological study
e. Dispensary observation
Feedback
The correct answer is: Bacteriological study
Question 85
Not answered
Mark 0.00 out of 1.00
Flag question
Question text
A 52 year old male patient complains about attacks of asphyxia, pain in his left side during
respiration. These manifestations turned up all of a sudden. It is known from his anamnesis that he
had been treated for thrombophlebitis of the right leg for the last month. In the admission ward the
patient suddenly lost consciousness, there was a sudden attack of asphyxia and pain in his left side.
Objectively: heart rate - 102/min, respiratory rate - 28/min, AP- 90/70 mm Hg. Auscultation revealed
diastolic shock above the pulmonary artery, gallop rhythm, small bubbling rales above the lungs
under the scapula on the right, pleural friction rub. What examination method will be the most
informative for a diagnosis?
Select one:
a. Coagulogram
b. Study of external respiration function
c. Angiography of pulmonary vessels
d. ECG
e. Echocardioscopy
Feedback
The correct answer is: Angiography of pulmonary vessels
Question 86
Not answered
Mark 0.00 out of 1.00
Flag question
Question text
A 52 year old patient complains about headache, weakness of his upper left extremity. Neurological
symptoms become more intense during physical stress of the left extremity. Pulsation on the arteries
of the left extremity is sharply dampened but it remains unchanged on the carotid arteries. What is
the most probable diagnosis?
Select one:
a. Raynaud's syndrome
b. Occlusion of brachiocephalic trunk
c. Thoracal outlet syndrome
d. Takayasu's disease
e. Occlusion of the left subclavicular artery, steal syndrome
Feedback
The correct answer is: Occlusion of the left subclavicular artery, steal syndrome
Question 87
Not answered
Mark 0.00 out of 1.00
Flag question
Question text
A 52 year old patient has hypervolaemic type of essential hypertension. Which of the following
medications is to be prescribed either as monotherapy or in complex with other antihypertensive
drugs?
Select one:
a. Hypothiazid
b. Kapoten
c. Dibazol
d. Nifedipin
e. Clonidine
Feedback
The correct answer is: Hypothiazid
Question 88
Not answered
Mark 0.00 out of 1.00
Flag question
Question text
A 52 year old patient was admitted to a hospital because of high hemorrhagic diathesis of mucous
membranes, massive skin haemorrhages in form of ecchymoses and spots, nasal and stomachal
haemorrhages. After clinical examinations her illness was diagnosed as thrombocytopenic purpura.
What is the most probable cause of this disease?
Select one:
a. Generation of antithrombocytic antibodies
b. Inherited insufficiency of plasm factors of blood coagulation
c. Disturbed hemostasis
d. Deficit of the VIII factor of blood coagulation
e. Iron deficit in blood serum, bone marrow and depot
Feedback
The correct answer is: Generation of antithrombocytic antibodies
Question 89
Not answered
Mark 0.00 out of 1.00
Flag question
Question text
A 52 year old patient with history of functional Class II angina complains of having intense and
prolonged retrosternal pains, decreased exercise tolerance for 5 days. Angina is less responsive to
nitroglycerine. What is the most probable diagnosis?
Select one:
a. IHd. Functional Class II angina
b. Myocarditis
c. IHd. Unstable angina
d. Myocardial dystrophy
e. Cardialgia due to spine problem
Feedback
The correct answer is: IHd. Unstable angina
Question 90
Not answered
Mark 0.00 out of 1.00
Flag question
Question text
A 52 year old woman complains about face distortion. It turned up 2 days ago after supercooling.
Objectively: body temperature is 38,2oC. Face asymmetry is present. Frontal folds are flattened. Left
eye is wider than right one and doesn't close. Left nasolabial fold is flattened, mouth corner is
lowered. Examination revealed no other pathology. Blood count: leukocytes - 10*109/l, ESR - 20
mm/h. What is the most probable diagnosis?
Select one:
a. Ischemic stroke
b. Facial neuritis
c. Brain tumour
d. Trigeminus neuralgia
e. Hemicrania (migraine)
Feedback
The correct answer is: Facial neuritis
Question 91
Not answered
Mark 0.00 out of 1.00
Flag question
Question text
A 54 year old female patient was admitted to the hospital with evident acrocyanosis, swollen cervical
veins, enlarged liver, ascites. Cardiac borders are dilated. Heart sounds cannot be auscultated,
apical beat is undetectable. AP is 100/50 mm Hg. X-ray picture of chest shows enlarged heart
shadow in form of a trapezium. What pathology might have caused these symptoms?
Select one:
a. Complex heart defect
b. Exudative pleuritis
c. Hiatal hernia
d. Cardiac tamponade
e. Acute cardiac insufficiency
Feedback
The correct answer is: Cardiac tamponade
Question 92
Not answered
Mark 0.00 out of 1.00
Flag question
Question text
A 54 year old male patient complains about permanent dull pain in the mesogastral region, weight
loss, dark blood admixtures in the feces, constipations. He put off 10 kg within a year. In blood:
erythrocytes: 3,5*1012/l, Hb- 87 g/l, leukocytes - 12,6*109/l, stab neutrophil shift, ESR- 43 mm/h.
What is the most probable diagnosis?
Select one:
a. Gastric ulcer
b. Chronic pancreatitis
c. Stomach cancer
d. Chronic colitis
e. Cancer of transverse colon
Feedback
The correct answer is: Cancer of transverse colon
Question 93
Not answered
Mark 0.00 out of 1.00
Flag question
Question text
A 58 year old female patient complains about periodical headache, dizziness and ear noise. She has
been suffering from diabetes mellitus for 15 years. Objectively: heart sounds are rhythmic, heart rate
is 76/min, there is diastolic shock above aorta, AP is 180/110 mm Hg. In urine: OD- 1,014. Daily loss
of protein with urine is 1,5 g. What drug should be chosen for treatment of arterial hypertension?
Select one:
a. Calcium channel antagonist
b. b-blocker
c. Ihibitor of angiotensin converting enzyme
d. a-blocker
e. Thiazide diuretic
Feedback
The correct answer is: Ihibitor of angiotensin converting enzyme
Question 94
Not answered
Mark 0.00 out of 1.00
Flag question
Question text
A 59 year old female patient applied to a maternity welfare clinic and complained about bloody
discharges from the genital tracts. Postmenopause is 12 years. Vaginal examination revealed that
external genital organs had signs of age involution, uterus cervix was not erosive, small amount of
bloody discharges came from the cervical canal. Uterus was of normal size, uterine appendages
were unpalpable. Fornices were deep and painless. What method should be applied for the
diagnosis specification?
Select one:
a. Separated diagnosic curretage
b. Laparoscopy
c. Culdoscopy
d. Extensive colposcopy
e. Puncture of abdominal cavity through posterior vaginal fornix
Feedback
The correct answer is: Separated diagnosic curretage
Question 95
Not answered
Mark 0.00 out of 1.00
Flag question
Question text
A 60 year old man with unstable angina pectoris fails to respond to heparin, nitroglycerin, beta
adrenegic blockers and calcium channel antagonist. The best management includes:
Select one:
a. Antihypertensive therapy
b. Intravenous streptokinase
c. Coronary artery bypass grafting
d. Excercise testing
e. Oral aspirin
Feedback
The correct answer is: Coronary artery bypass grafting
Question 96
Not answered
Mark 0.00 out of 1.00
Flag question
Question text
A 62 year old patient complains of rest dyspnea, heart pains. 3 years ago he had myocardial
infarction. Physical examination: orthopnea, acrocyanosis, swollen cervical veins. Ps - 92, total heart
enlargement, the liver is enlarged by 7 cm, shin edema. What is the stage of chronic heart failure
(CHF)?
Select one:
a. CHF- 1
b. CHF- 3
c. CHF- 0
d. CHF- 2 А
e. CHF- 2 B
Feedback
The correct answer is: CHF- 2 B
Question 97
Not answered
Mark 0.00 out of 1.00
Flag question
Question text
A 63 year old patient complained about pain in the lumbar area. He underwent a course of
physiological treatment on account of radiculitis but this led to no improvement of his condition. R-
graphy of spinal column and pelvic bones revealed osteoporosis and serious bone defects. Blood
analysis revealed moderate normochromic anaemia, urine analysis revealed proteinuria. Whole
blood protein made up 10,7 g/l. What disease should be suspected?
Select one:
a. Systemic osteoporosis
b. Urolithiasis
c. Metastases in bones
d. Acute radiculitis
e. Myelomatosis
Feedback
The correct answer is: Myelomatosis
Question 98
Not answered
Mark 0.00 out of 1.00
Flag question
Question text
A 63 year old patient was diagnosed with purulent mediastinitis. What of the below listed diseases
are NOT the cause of purulent mediastinitis?
Select one:
a. Deep neck phlegmon
b. Perforation of the thoracic part of the oesophagus
c. Iatrogenic injury of the trachea
d. Perforation of the cervical part of the oesophagus
e. Cervical lymphadenitis
Feedback
The correct answer is: Cervical lymphadenitis
Question 99
Not answered
Mark 0.00 out of 1.00
Flag question
Question text
A 67 year old female patient complains about edemata of face and legs, pain in the lumbar area that
is getting worse at moving; great weakness, sometimes nasal haemorrhages, rise of body
temperature up to 38,4oC. Objectively: painfulness of vertebral column and ribs on palpation.
Laboratorial study revealed daily proteinuria of 4,2 g, ESR- 52 mm/h. What changes of laboratory
indices are to be expected?
Select one:
a. Whole protein of blood serum - 101 g/l
b. Haemoglobin - 165 g/l
c. g-globulins - 14%
d. Leukocytes - 15,3 g/l
e. Albumins - 65%
Feedback
The correct answer is: Whole protein of blood serum - 101 g/l
Question 100
Not answered
Mark 0.00 out of 1.00
Flag question
Question text
A 68 year old female patient complains about temperature rise up to 38,3oC, haematuria. ESR- 55
mm/h. Antibacterial therapy turned out to be ineffective. What diagnosis might be suspected?
Select one:
a. Urolithiasis
b. Renal cancer
c. Renal amyloidosis
d. Chronic glomerulonephritis
e. Polycystic renal disease
Feedback
The correct answer is: Renal cancer
Question 101
Not answered
Mark 0.00 out of 1.00
Flag question
Question text
A 68 year old patient complains about acute pain in his right foot, toe edema and darkening of skin
of the IV toe. He has been suffering from diabetes mellitus for 15 years, doesn't receive regular
treatment. What complication of diabetes mellitus is it?
Select one:
a. Haematoma
b. Gangrene of the IV toe on the right foot
c. Panaritium
d. Erysipelas
e. Fracture of the IV toe on the right foot
Feedback
The correct answer is: Gangrene of the IV toe on the right foot
Question 102
Not answered
Mark 0.00 out of 1.00
Flag question
Question text
A 70 year old man is suffering from coronary heart disease. His mood is evidently depressed,
anxious. As a result of continious sleeplessness he has got fears, suicidal thoughts. He would sit for
a long time in the same pose, answer after a pause, in a low, monotonous voice. His face has a look
of suffering, pain, fear. What is the main psychopathologic syndrome?
Select one:
a. Phobic syndrome
b. Depressive syndrome
c. Obsessive syndrome
d. Paranoid syndrome
e. Asthenic syndrome
Feedback
The correct answer is: Depressive syndrome
Question 103
Not answered
Mark 0.00 out of 1.00
Flag question
Question text
A 72 year old female patient has been treated for urolithiasis in the urological department. After
atropine injection she got acute pain in her left eye and abrupt vision impairment. Objectively: visual
acuity of the left eye is 0,01, the eye is dense but painful on palpation, cornea is opaque, there is
cyanotic induration of eyeball vessels. What is the most probable diagnosis?
Select one:
a. Degeneration of the left eye cornea
b. Acute keratitis of the left eye
c. Acute iridocyclitis of the left eye
d. Acute attack of primary glaucoma of the left eye
e. Secondary glaucoma of the left eye
Feedback
The correct answer is: Acute attack of primary glaucoma of the left eye
Question 104
Not answered
Mark 0.00 out of 1.00
Flag question
Question text
A 72 year old male patient complains about itch in his left shin, especially around a trophic ulcer.
Skin is reddened and edematic, there are some oozing lesions, single yellowish crusts. The focus of
affection is well-defined. What is the most probable diagnosis?
Select one:
a. Seborrheic eczema
b. Streptococcal impetigo
c. Allergic dermatitis
d. Microbial eczema
e. Cutaneous tuberculosis
Feedback
The correct answer is: Microbial eczema
Question 105
Not answered
Mark 0.00 out of 1.00
Flag question
Question text
A 72 year old patient complains about pain and bleeding during defecation. Digital rectal
investigation revealed a tumour of anal canal. After verification of the diagnosis the patient was
diagnosed with squamous cell carcinoma. The secondary (metastatic) tumour will be most probably
found in:
Select one:
a. Lungs
b. Liver
c. Mediastinum
d. Brain
e. Pelvic bones
Feedback
The correct answer is: Lungs
Question 106
Not answered
Mark 0.00 out of 1.00
Flag question
Question text
A 75 year old man who has been suffering from diabetes for the last six months was found to be
jaundiced. He was asymptomatic except for weight loss at the rate of 10 pounds in 6 months.
Physical examination revealed a hard, globular, right upper quadrant mass that moves during
respiration. A CT scan shows enlargement of the head of the pancreas, with no filling defects in the
liver. The most likely diagnosis is:
Select one:
a. Carcinoma of the head of the pancreas
b. Malignant biliary stricture
c. Haemolytic jaundice
d. Metastatic disease of liver
e. Infectious hepatitis
Feedback
The correct answer is: Carcinoma of the head of the pancreas
Question 107
Not answered
Mark 0.00 out of 1.00
Flag question
Question text
A 9 year old boy had acute respiratory viral infection. After it there appeared polydipsia, polyuria,
weakness, nausea. Examination revealed the following symptoms: mental confusion, dry skin, soft
eyeballs, Kussmaul's respiration, acetone smell from the mouth, muffled heart sounds, soft and
painless abdomen. Blood sugar was 19 millimole/l. What acute condition is it?
Select one:
a. Cerebral coma
b. Ketoacidotic coma
c. Hepatic coma
d. Hyperosmolar coma
e. Acute renal insufficiency
Feedback
The correct answer is: Ketoacidotic coma
Question 108
Not answered
Mark 0.00 out of 1.00
Flag question
Question text
A boy is 1 year old. Previously he had purulent otitis. After that he started complaining about pain in
the upper third of his left thigh, body temperature rose up to 39oC. Objectively: the thigh is swollen in
its upper third, inguinal fold is smoothed. Extremity is in half-bent position. Active and passive
movements are impossible because of acute pain. What is the most probable diagnosis?
Select one:
a. Acute coxitis
b. Intermuscular phlegmon
c. Osteosarcoma
d. Acute haematogenic osteomyelitis
e. Brodie's abscess
Feedback
The correct answer is: Acute haematogenic osteomyelitis
Question 109
Not answered
Mark 0.00 out of 1.00
Flag question
Question text
A child is 1 day old. During delivery there had been problems with extraction of shoulders. Body
weight is 4300,0. Right arm hangs down along the body, hand is pronated, movement in the arm is
absent. "Scarf" symptom is positive. What is the most probable diagnosis?
Select one:
a. Hemiparesis
b. Distal right-sided obstetric paralysis
c. Tetraparesis
d. Total right-sided obstetric paralysis
e. Proximal right-sided obstetric paralysis
Feedback
The correct answer is: Total right-sided obstetric paralysis
Question 110
Not answered
Mark 0.00 out of 1.00
Flag question
Question text
A child was born with body weight 3250 g and body length 52 cm. At the age of 1,5 month the actual
weight is sufficient (4350 g), psychophysical development corresponds with the age. The child is
breast-fed, occasionally there are regurgitations. What is the cause of regurgitations?
Select one:
a. Pylorostenosis
b. Aerophagia
c. Pylorospasm
d. Acute gastroenteritis
e. Esophageal atresia
Feedback
The correct answer is: Aerophagia
Question 111
Not answered
Mark 0.00 out of 1.00
Flag question
Question text
A department chief of an in-patient hospital is going to examine resident doctors as to observation of
medical-technological standards of patient service. What documentation should be checked for this
purpose?
Select one:
a. Statistic cards of discharged patients
b. Treatment sheets
c. Health cards of in-patients
d. Registry of operative interventions
e. Annual report of a patient care institution
Feedback
The correct answer is: Health cards of in-patients
Question 112
Not answered
Mark 0.00 out of 1.00
Flag question
Question text
A female patient consulted a doctor about gain in weight, chill, edemata, dry skin, sleepiness,
problems with concentration. Objectively: the patient's height is 165 cm, weight is 90 kg, gynoid body
proportions, to- 35,8oC, ESR- 58/min, AP- 105/60 mm Hg. Heart sounds are weakened, bradycardia
is present. Other internal organs have no changes. Thyroid gland is not palpable. Mammary glands
ooze milk droplets. Hormonal study revealed rise of TSH and prolactin concentration, reduction of
T4. What factor caused obesity?
Select one:
a. Prolactinoma
b. Primary hypothyroidism
c. Hypopituitarism
d. Secondary hypothyroidism
e. Adiposogenital dystrophy
Feedback
The correct answer is: Primary hypothyroidism
Question 113
Not answered
Mark 0.00 out of 1.00
Flag question
Question text
A female patient has been suffering from pain in the right subcostal area, bitter taste in the mouth,
periodical bile vomiting for a month. The patient put off 12 kg. Body temperature in the evening is
37,6oC. Sonography revealed that bile bladder was 5,5х2,7 cm large, its wall - 0,4 cm, choledochus
- 0,8 cm in diameter. Anterior liver segment contains a roundish hypoechoic formation up to 5 cm in
diameter and another two up to 1,5 cm each, walls of these formations are up to 0,3 cm thick. What
is the most probable diagnosis?
Select one:
a. Alveolar echinococcus of liver
b. Liver abscess
c. Cystous liver cancer
d. Paravesical liver abscesses
e. Liver cancer
Feedback
The correct answer is: Alveolar echinococcus of liver
Question 114
Not answered
Mark 0.00 out of 1.00
Flag question
Question text
A fitter of a metallurgic factory with occupational exposure to high concentrations of mercury fumes
for 16 years presents instability of pulse and blood pressure, general hyperhydrosis, asymmetric
innervations of facial muscles and tongue, positive subcortical reflexes, hand tremor on physical
examination. A dentist revealed paradontosis and chronic stomatitis. What is the most probable
diagnosis?
Select one:
a. Parkinson syndrome
b. Neuroinfection
c. Acute mercury intoxication
d. Chronic mercury intoxication
e. Mercury encephalopathy
Feedback
The correct answer is: Chronic mercury intoxication
Question 115
Not answered
Mark 0.00 out of 1.00
Flag question
Question text
A healthy 75 year old woman who leads a moderately active way of life went through a preventive
examination that revealed serum concentration of common cholesterol at the rate of 5,1 millimole/l
and HDL (high-density lipoproteins) cholesterol at the rate of 70 mg/dl. ECG reveals no pathology.
What dietary recommendation is the most adequate?
Select one:
a. Decrease of saturated fats consumption
b. Any dietary changes are necessary
c. Decrease of carbohydrates consumption
d. Increase of cellulose consumption
e. Decrease of cholesterol consumption
Feedback
The correct answer is: Any dietary changes are necessary
Question 116
Not answered
Mark 0.00 out of 1.00
Flag question
Question text
A male patient, 60 years old, tobacco smoker for 30 years, alcoholic, has dysphagia and weight loss
since 4 months. Suggested diagnosis?
Select one:
a. Cancer of the esophagus
b. Esophageal achalasia
c. Hanter's disease
d. Esophagitis
e. Esophageal diverticulum
Feedback
The correct answer is: Cancer of the esophagus
Question 117
Not answered
Mark 0.00 out of 1.00
Flag question
Question text
A man, aged 37, working on the collective farm on sowing, was admitted to the infectious hospital
with the clinical symptoms: miosis, labored breathing, sweating.What kind of poisoning is it and what
is the first aid?
Select one:
a. Poisoning by lead. Treatment: tetacine
b. Poisoning by vapours of mercury. Treatment: unithiol
c. Poisoning by the methylic alcohol. Treatment: ethylic alcohol
d. Calcii Poisoning by POC. Treatment: atropine
Feedback
The correct answer is: Calcii Poisoning by POC. Treatment: atropine
Question 118
Not answered
Mark 0.00 out of 1.00
Flag question
Question text
A maternity hospital registered 616 live births, 1 stillbirth, 1 death on the 5th day of life over a 1 year
period. What index allows the most precise estimation of this situation?
Select one:
a. Neonatal mortality
b. Natural increase
c. Crude mortality rate
d. Natality
e. Perinatal mortality
Feedback
The correct answer is: Perinatal mortality
Question 119
Not answered
Mark 0.00 out of 1.00
Flag question
Question text
A nine year old child is at a hospital with acute glomerulonephritis. Clinical and laboratory
examinations show acute condition. What nutrients must not be limited during the acute period of
glomerulonephritis?
Select one:
a. Proteins
b. Carbohydrates
c. Fats
d. Salt
e. Liquid
Feedback
The correct answer is: Carbohydrates
Question 120
Not answered
Mark 0.00 out of 1.00
Flag question
Question text
A parturient complains about pain in the mammary gland. Palpation revealed a 3х4 cm large
infiltration, soft in the centre. Body temperature is 38,5oC. What is the most probable diagnosis?
Select one:
a. Retention of milk
b. Acute purulent mastitis
c. Birth trauma
d. Pneumonia
e. Pleuritis
Feedback
The correct answer is: Acute purulent mastitis
Question 121
Not answered
Mark 0.00 out of 1.00
Flag question
Question text
A parturient woman is 27 year old, it was her second labour, delivery was at term, normal course. On
the 3rd day of postpartum period body temperature is 36,8oC, Ps - 72/min, AP - 120/80 mm Hg.
Mammary glands are moderately swollen, nipples are clean. Abdomen is soft and painless. Fundus
of uterus is 3 fingers below the umbilicus. Lochia are bloody, moderate. What is the most probable
diagnosis?
Select one:
a. Physiological course of postpartum period
b. Subinvolution of uterus
c. Postpartum metroendometritis
d. Remnants of placental tissue after labour
e. Lactostasis
Feedback
The correct answer is: Physiological course of postpartum period
Question 122
Not answered
Mark 0.00 out of 1.00
Flag question
Question text
A patient applied to the traumatology cenre and complained about a trauma of the lower third of the
volar forearm surface caused by cut on a piece of galss. Objectively: flexion of the IV and V fingers
is impaired, sensitivity of the interior dorsal and palmar surface of hand as well as of the IV finger is
reduced. What nerve is damaged?
Select one:
a. Median
b. Radial
c. Ulnar
d. Musculoskeletal
e. Axillary
Feedback
The correct answer is: Ulnar
Question 123
Not answered
Mark 0.00 out of 1.00
Flag question
Question text
A patient complained about problems with pain and tactile sensitivity, pain in the nail bones at the
end of the working day. He works at a plant with mechanical devices. What pathology can be
suspected?
Select one:
a. Overwork symptoms
b. Caisson disease
c. Vibration disease
d. Hypovitaminosis of B1
e. Noise disease
Feedback
The correct answer is: Vibration disease
Question 124
Not answered
Mark 0.00 out of 1.00
Flag question
Question text
A patient complains about evaginations in the region of anus that appear during defecation and need
to be replaced. Examination with anoscope revealed 1x1 cm large evaginations of mucosa above
the pectineal line. What is the most probable diagnosis?
Select one:
a. Anal fissure
b. External hemorrhoids
c. Acute paraproctitis
d. Internal hemorrhoids
e. Don’t Guess, Think
Feedback
The correct answer is: Internal hemorrhoids
Question 125
Not answered
Mark 0.00 out of 1.00
Flag question
Question text
A patient complains about strong dyspnea that is getting worse during physical activity.
Presentations appeared suddenly 2 hours ago at work: acute chest pain on the left, cough. The pain
was abating, but dyspnea, dizziness, pallor, cold sweat and cyanosis were progressing. Vesicular
respiration is absent, X-ray picture shows a shadow on the left. What pathology might be suspected?
Select one:
a. Left-sided pneumonia
b. Pulmonary abscess
c. Pulmonary infarction
d. Pleuritis
e. Spontaneous left-sided pneumothorax
Feedback
The correct answer is: Spontaneous left-sided pneumothorax
Question 126
Not answered
Mark 0.00 out of 1.00
Flag question
Question text
A patient consulted a venereologist about painful urination, reddening of the external opening of
urethra, profuse purulent discharges from the urethra. He considers himself to be ill for 3 days. He
also associates the disease with a casual sexual contact that took place for about a week ago. If
provisional diagnosis "acute gonorrheal urethritis" will be confirmed, then bacteriological study of
urethral discharges will reveal:
Select one:
a. Spirochaete
b. Gram-negative diplococci
c. Gram-positive diplococci
d. Proteus vulgaris
e. Mycoplasma
Feedback
The correct answer is: Gram-negative diplococci
Question 127
Not answered
Mark 0.00 out of 1.00
Flag question
Question text
A patient has been in a hospital. The beginning of the disease was gradual: nausea, vomiting, dark
urine, аcholic stools, yellowness of the skin and scleras. The liver is protruded by 3 cm. Jaundice
progressed on the 14th day of the disease. The liver diminished in size. What complication of viral
hepatitis caused deterioration of the patient's condition?
Select one:
a. Relapse of viral hepatitis
b. Cholangitis
c. Infectious-toxic shock
d. Hepatic encephlopathy
e. Meningitis
Feedback
The correct answer is: Hepatic encephlopathy
Question 128
Not answered
Mark 0.00 out of 1.00
Flag question
Question text
A patient has got acute macrofocal myocardial infarction complicated by cardiogenic shock. The
latter is progresing under conditions of weak general peripheric resistance and lowered cardiac
output. What antihypotensive drug should be injected to the patient in the first place?
Select one:
a. Mesatonum
b. Noradrenaline
c. Adrenaline
d. Prednisolone
e. Dopamine
Feedback
The correct answer is: Dopamine
Question 129
Not answered
Mark 0.00 out of 1.00
Flag question
Question text
A patient has got pain in the axillary area, rise of temperature developed 10 hours ago. On
examination: shaky gait is evident, the tongue is coated with white deposit. The pulse is frequent.
The painful lymphatic nodes are revealed in the axillary area. The skin over the lymph nodes is
erythematous and glistering. What is the most probable diagnosis?
Select one:
a. Anthrax
b. Tularemia
c. Lymphogranulomatosis
d. Bubonic plague
e. Acute purulent lymphadenitis
Feedback
The correct answer is: Bubonic plague
Question 130
Not answered
Mark 0.00 out of 1.00
Flag question
Question text
A patient has restrained umbilateral hernia complicated by phlegmon hernia, it is necessary to take
following actions:
Select one:
a. Herniotomy by Mayo
b. Herniotomy by Lekser
c. Herniotomy by Sapezhko
d. Herniotomy by Grenov
e. Herniotomy by Mayo-Sapezhko
Feedback
The correct answer is: Herniotomy by Mayo-Sapezhko
Question 131
Not answered
Mark 0.00 out of 1.00
Flag question
Question text
A patient suffering from gastroesophageal reflux has taken from time to time a certain drug that
"reduces acidity" for 5 years. This drug was recommended by a pharmaceutist. The following side
effects are observed: osteoporosis, muscle weakness, indisposition. What drug has such following
effects?
Select one:
a. Gastrozepin
b. Aluminium-bearing antacid
c. Metoclopramide
d. Н2-blocker
e. Inhibitor of proton pump
Feedback
The correct answer is: Aluminium-bearing antacid
Question 132
Not answered
Mark 0.00 out of 1.00
Flag question
Question text
A patient was delivered to a surgical department after a road accident with a closed trauma of chest
and right-sided rib fracture. The patient was diagnosed with right-sided pneumothorax, it is indicated
to perform drainage of pleural cavity. Pleural puncture should be made in:
Select one:
a. In the projection of pleural sinus
b. In the 2nd intercostal space along the middle clavicular line
c. In the 7th intercostal space along the scapular line
d. In the 6th intercostal space along the posterior axillary line
e. In the point of the greatest dullness on percussion
Feedback
The correct answer is: In the 2nd intercostal space along the middle clavicular line
Question 133
Not answered
Mark 0.00 out of 1.00
Flag question
Question text
A patient who takes diuretics has developed arrhythmia as a result of cardiac glycoside overdose.
What is the treatment tactics in this case?
Select one:
a. Reduced magnesium concentration in blood
b. Increased potassium concentration in blood
c. Increased calcium concentration in blood
d. Increased sodium consentration in blood
Feedback
The correct answer is: Increased potassium concentration in blood
Question 134
Not answered
Mark 0.00 out of 1.00
Flag question
Question text
A patient with acute purulent otitis media complicated by mastoiditis was admitted to a hospital.
Roentgenogram of mastoid processes showed the shadiowing of the cellular system on the lesion,
absence of bone septa was present. What are the necessary therapeutic actions at the second
stage of mastoiditis?
Select one:
a. Mastoidotomy
b. Cateterization of the Eustachian tube
c. Paracentesis of the drum
d. Tympanoplasty
e. Radical operation on the middle ear
Feedback
The correct answer is: Mastoidotomy
Question 135
Not answered
Mark 0.00 out of 1.00
Flag question
Question text
A pediatrician talked to a mother of a 7 month old breast-fed boy and found out that the child was fed
7 times a day. How many times should the child of such age be fed?
Select one:
a. 7 times
b. 3 times
c. 6 times
d. 5 times
e. 4 times
Feedback
The correct answer is: 5 times
Question 136
Not answered
Mark 0.00 out of 1.00
Flag question
Question text
A plot of land with total area of 2,0 hectare was intended for building of a hospital. The maximal
capacity of the hospital will be:
Select one:
a. 800 beds
b. 400 beds
c. 100 beds
d. Over 1000 beds
e. 200 beds
Feedback
The correct answer is: 100 beds
Question 137
Not answered
Mark 0.00 out of 1.00
Flag question
Question text
A pregnant woman in her 40th week of pregnancy undergoes obstetric examination: the cervix of
uterus is undeveloped. The oxytocin test is negative. Examination at 32 weeks revealed: AP 140/90
mm Hg, proteinuria 1 g/l, peripheral edemata. Reflexes are normal. Choose the most correct tactics:
Select one:
a. Caesarian section immediately
b. Complex therapy of gestosis for 2 days
c. Complex therapy of gestosis for 7 days
d. Absolute bed rest for 1 month
e. Labour stimulation after preparation
Feedback
The correct answer is: Labour stimulation after preparation
Question 138
Not answered
Mark 0.00 out of 1.00
Flag question
Question text
A prematurely born girl is now 8 months old. She has dyspnea, tachycardia, hepatosplenomegaly,
physical developmental lag, limb cyanosis. There is also parasternal cardiac hump, auscultation
revealed systolodiastolic murmur in the II intercostal space on the left. AP is 90/0 mm Hg. What
disease should be suspected?
Select one:
a. Stenosis of pulmonary artery
b. Nonclosure of interventricular septum
c. Stenosis of aortal valve
d. Patent ductus arteriosus
e. Coarctation of aorta
Feedback
The correct answer is: Patent ductus arteriosus
Question 139
Not answered
Mark 0.00 out of 1.00
Flag question
Question text
A sergeant was injured by a shell splinter in the left subcostal area. He was bandaged with a first-aid
pack on a battlefield. The patient was delivered to the regiment medical aid station. He complains
about dizziness, weakness, thirst, abdominal pain. General condition is grave, the patient is pale. Ps
is 120 bpm. Abdomen is soft, painful on palpation. The bandage is well fixed but a little bit soaked
with blood. The patient should be evacuated to the medical battalion with the following transport and
in the following turn:
Select one:
a. With medical vehicle in the second turn
b. With a passing car in the first turn
c. With a passing car in the third turn
d. With a passing car in the second turn
e. With medical vehicle in the first turn
Feedback
The correct answer is: With medical vehicle in the first turn
Question 140
Not answered
Mark 0.00 out of 1.00
Flag question
Question text
A woman 26 years old has abused alcohol for 7 years. She has psychological dependence on
alcohol, but no withdrawal syndrome. Drinks almost every day approximately 50-100 g of wine. She
is in her 4-th week of pregnancy. Primary prevention of fetal alcohol syndrome requires:
Select one:
a. Medical abortion
b. of alcoholism and full abstinance from alcohol during all the period of pregnancy
c. Participation in the A-ANON group
d. Decrease of alcohol use
e. Gyneacological observation
Feedback
The correct answer is: of alcoholism and full abstinance from alcohol during all the period of
pregnancy
Question 141
Not answered
Mark 0.00 out of 1.00
Flag question
Question text
A woman complains of having slight dark bloody discharges and mild pains in the lower part of
abdomen for several days. Last menses were 7 weeks ago. The pregnancy test is positive.
Bimanual investigation: the body of the uterus indicates for about 5-6 weeks of pregnancy, it is soft,
painless. In the left appendage there is a retort-like formation, 7х5 cm large, mobile, painless. What
examination is necessary for detection of fetus localization?
Select one:
a. Hysteroscopy
b. Ultrasound
c. Cystoscopy
d. Colposcopy
e. Hromohydrotubation
Feedback
The correct answer is: Ultrasound
Question 142
Not answered
Mark 0.00 out of 1.00
Flag question
Question text
A woman consulted a doctor on the 14th day after labour about sudden pain, hyperemy and
induration of the left mammary gland, body temperature rise up to 39oC, headache, indisposition.
Objectively: fissure of nipple, enlargement of the left mammary gland, pain on palpation. What
pathology would you think about in this case?
Select one:
a. Lactational mastitis
b. Breast cancer
c. Lacteal cyst with suppuration
d. Phlegmon of mammary gland
e. Fibrous adenoma of the left mammary gland
Feedback
The correct answer is: Lactational mastitis
Question 143
Not answered
Mark 0.00 out of 1.00
Flag question
Question text
A woman consulted a therapeutist about fatigability, significant weight loss, weakness, loss of
appetite. She has had amenorrhea for 8 months. A year ago she born a full-term child. Haemorrhage
during labour made up 2 l. She got blood and blood substitute transfusions. What is the most
probable diagnosis?
Select one:
a. Stein-Leventhal syndrome
b. Shereshevsky-Turner's syndrome
c. Homological blood syndrome
d. Sheehan's syndrome
e. Vegetovascular dystonia
Feedback
The correct answer is: Sheehan's syndrome
Question 144
Not answered
Mark 0.00 out of 1.00
Flag question
Question text
A woman delivered a child. It was her fifth pregnancy but the first delivery. Mother's blood group is
A(II)Rh-, newborn's - A(II)Rh+. The level of indirect bilirubin in umbilical blood was 58 micromole/l,
haemoglobin - 140 g/l, RBC- 3,8*1012/l. In 2 hours the level of indirect bilirubin turned 82
micromole/l. The hemolytic disease of newborn (icteric-anemic type, Rh-incompatibility) was
diagnosed. Choose the therapeutic tactics:
Select one:
a. Replacement blood transfusion (conservative therapy)
b. Antibiotics
c. Blood transfusion (conservative therapy)
d. Conservative therapy
e. Symptomatic therapy
Feedback
The correct answer is: Replacement blood transfusion (conservative therapy)
Question 145
Not answered
Mark 0.00 out of 1.00
Flag question
Question text
A woman is 34 years old, it is her tenth labor at full term. It is known from the anamnesis that the
labor started 11 hours ago, labor was active, painful contractions started after discharge of waters
and became continuous. Suddenly the parturient got knife-like pain in the lower abdomen and labor
activity stopped. Examination revealed positive symptoms of peritoneum irritation, ill-defined uterus
outlines. Fetus was easily palpable, movable. Fetal heartbeats wasn't auscultable. What is the most
probable diagnosis?
Select one:
a. Rupture of uterus
b. Risk of uterus rupture
c. Discoordinated labor activity
d. II labor period
e. Uterine inertia
Feedback
The correct answer is: Rupture of uterus
Question 146
Not answered
Mark 0.00 out of 1.00
Flag question
Question text
A woman of a high-risk group (chronic pyelonephritis in anamnesis) had vaginal delivery. The day
after labour she complained of fever and loin pains, frequent urodynia. Specify the most probable
complication:
Select one:
a. Apostasis of sutures after episiotomy
b. Endometritis
c. Infectious hematoma
d. Thrombophlebitis of veins of the pelvis
e. Infectious contamination of the urinary system
Feedback
The correct answer is: Infectious contamination of the urinary system
Question 147
Not answered
Mark 0.00 out of 1.00
Flag question
Question text
Administration of a plant producing red lead paint intends to form a group of medical specialists for
periodical medical examinations. What specialist must be obligatory included into this group?
Select one:
a. Neuropathologist
b. Gynaecologist
c. Otolaryngologyst
d. Psychiatrist
e. Dermatologist
Feedback
The correct answer is: Neuropathologist
Question 148
Not answered
Mark 0.00 out of 1.00
Flag question
Question text
After objective clinical examination a 12 year old child was diagnosed with mitral valve prolapse.
What complementary instrumental method of examination should be applied for the diagnosis
confirmation?
Select one:
a. Phonocardiography
b. ECG
c. Veloergometry
d. Roentgenography of chest
e. Echocardiography
Feedback
The correct answer is: Echocardiography
Question 149
Not answered
Mark 0.00 out of 1.00
Flag question
Question text
An 18 year old patient was admitted to a hospital with complaints of headache, weakness, high
temperature, sore throat. Objectively: enlargement of all groups of lymph nodes was revealed. The
liver is enlarged by 3 cm, spleen - by 1 cm. In blood: leukocytosis, atypical lymphocytes - 15%. What
is the most probable diagnosis?
Select one:
a. Adenoviral infection
b. Diphtheria
c. Acute lymphoid leukosis
d. Angina
e. Infectious mononucleosis
Feedback
The correct answer is: Infectious mononucleosis
Question 150
Not answered
Mark 0.00 out of 1.00
Flag question
Question text
An 18 year old primigravida in her 27-28 week of pregnancy underwent an operation on account of
acute phlegmonous appendicitis. In the postoperative period it is necessary to take measures for
prevention of the following pegnancy complication:
Select one:
a. Noncarrying of pregnancy
b. Fetus hypotrophy
c. Late gestosis
d. Premature placenta detachment
e. Intestinal obstruction
Feedback
The correct answer is: Noncarrying of pregnancy
Question 151
Not answered
Mark 0.00 out of 1.00
Flag question
Question text
An 8 year old boy suffering from haemophilia was undergoing transfusion of packed red cells.
Suddenly he got pain behind the breastbone and in the lumbar area, dyspnea, cold sweat.
Objectively: pale skin, heart rate - 100/min, AP- 60/40 Hg; oliguria, brown urine. For treatment of this
complication the following drug should be administered:
Select one:
a. Prednisolone
b. Analgine
c. Lasix
d. Adrenaline
e. Aminophylline
Feedback
The correct answer is: Prednisolone
Question 152
Not answered
Mark 0.00 out of 1.00
Flag question
Question text
An infant is full-term. Delivery was pathological, with breech presentation. Examination of the infant
revealed limited abduction of the right leg to 50o, positive "clicking" symptom on the right,
asymmetric inguinal folds. What is the most probable diagnosis?
Select one:
a. Inborn dislocation of both hips
b. Varus deformity of both femoral necks
c. Inborn dislocation of the right hip
d. Right hip dysplasia
e. Fracture of both femoral necks
Feedback
The correct answer is: Inborn dislocation of the right hip
Question 153
Not answered
Mark 0.00 out of 1.00
Flag question
Question text
An outbreak of food poisoning was recorded in an urban settlement. The illness was diagnosed as
botulism on the grounds of clinical presentations. What foodstuffs should be chosen for analysis in
the first place in order to confirm the diagnosis?
Select one:
a. Potatoes
b. Cabbage
c. Pasteurized milk
d. Tinned food
e. Boiled meat
Feedback
The correct answer is: Tinned food
Question 154
Not answered
Mark 0.00 out of 1.00
Flag question
Question text
As a result of prophylactic medical examination a 35 year old woman was diagnosed with alimentary
and constitutive obesity of the III degree. It is known from her anamnesis that the patient doesn't
observe rules of rational nutrition: she often overeats, the last food intake is usually 10-15 minutes
before going to bed, prefers fattening and rich in carbohydrates food. What is the main alimentary
risk factor of obesity development?
Select one:
a. Excess of fats
b. Excess of carbohydrates
c. Energetic unprofitableness of nutrition
d. Violation of dietary pattern
e. Lack of cellulose
Feedback
The correct answer is: Energetic unprofitableness of nutrition
Question 155
Not answered
Mark 0.00 out of 1.00
Flag question
Question text
At year-end hospital administration has obtained the following data: annual number of treated
patients and average annual number of beds used for patients' treatment. What index of hospital
work can be calculated on the base of this data?
Select one:
a. Average duration of patients' presence in the hospital
b. Bed resources of the hospital
c. Average annual bed occupancy
d. Average bed idle time
e. Bed turnover
Feedback
The correct answer is: Bed turnover
Question 156
Not answered
Mark 0.00 out of 1.00
Flag question
Question text
During examination a patient is unconscious, his skin is dry and hot, face hyperemia is present. The
patient has Kussmaul's respiration, there is also smell of acetone in the air. Symptoms of peritoneum
irritation are positive. Blood sugar is at the rate of 33 millimole/l. What emergency actions should be
taken?
Select one:
a. Intravenous infusion of short-acting insulin
b. Intravenous infusion of sodium chloride saline
c. Intravenous infusion of neohaemodesum along with glutamic acid
d. Introduction of long-acting insulin
e. Intravenous infusion of glucose along with insulin
Feedback
The correct answer is: Intravenous infusion of short-acting insulin
Question 157
Not answered
Mark 0.00 out of 1.00
Flag question
Question text
During preventive examination a 16 year old patient presented no problems. Objectively: the patient
has signs of malnutrition, he is asthenic, AP is 110/70 mm Hg, Ps is 80 bpm, cardiac border is
normal, auscultation above the cardiac apex reveals three sounds, cardiac murmur is absent. ECG
shows no pathological changes, phonocardiogram shows that the third sound comes 0,15 s after the
second one above the apex. How are these changes called?
Select one:
a. Atrial gallop rhythm
b. III physiological sound
c. Protodiastolic gallop rhythm
d. Fout-ta-ta-rou (reduplication of the 2nd sound)
e. IV physiological sound
Feedback
The correct answer is: III physiological sound
Question 158
Not answered
Mark 0.00 out of 1.00
Flag question
Question text
Estimation of community health level involved analysis of a report on diseases registered among the
population of district under charge (reporting form 12). What index is calculated on the grounds of
this report?
Select one:
a. Index of hospitalized morbidity
b. Index of basic non-epidemic morbidity
c. Common sickness rate
d. Index of pathological affection
e. Index of morbidity with temporary disability
Feedback
The correct answer is: Common sickness rate
Question 159
Not answered
Mark 0.00 out of 1.00
Flag question
Question text
Estimation of physical development of a child involved dynamometry and estimation of body weight
and length, annual gain in body length, chest circumference, number of permanent teeth, secondary
sexual characters, lung vital capacity. Which of the mentioned indices relates to the physiometric
ones?
Select one:
a. Secondary sexual characters
b. Body length and weight, chest circumference
c. Annual gain in body length
d. Lung vital capacity, dynamometry
e. Number of permanent teeth
Feedback
The correct answer is: Lung vital capacity, dynamometry
Question 160
Not answered
Mark 0.00 out of 1.00
Flag question
Question text
Examination of a 12 year old child revealed diffuse thyroid enlargement of the II degree. Heart
auscultation revealed dullness of heart sounds, heart rate was 64/min. The child has frequent
constipations, anemia. Concentration of thyreoglobulin antibodies is increased. What disease might
have caused such symptoms?
Select one:
a. Endemic goiter
b. Diffuse toxic goiter
c. Thyroid hyperplasia
d. Thyroid carcinoma
e. Autoimmune thyroiditis
Feedback
The correct answer is: Autoimmune thyroiditis
Question 161
Not answered
Mark 0.00 out of 1.00
Flag question
Question text
Examination of a 22 year old man suffering from polyarthralgia and high fever revealed right-sided
exudative pleuritis. X-ray picture showed a homogenous shadow below the IV rib on the right. In the
II segment there were single dense focal shadows. Mantoux test with 2 TU resulted in formation of a
papula 16 mm largeE. Pleural liquid has increased protein concentration, Rivalta's reaction is
positive, there was also increased number of leukocytes with prevailing lymphocytes. What is the
most probable etiology of pleuritis?
Select one:
a. Viral
b. Autoimmune
c. Tuberculous
d. Cancerous
e. Staphylococcal
Feedback
The correct answer is: Tuberculous
Question 162
Not answered
Mark 0.00 out of 1.00
Flag question
Question text
Examination of a 26 year old female patient revealed a node in the right lobe of thyroid gland. The
node appeared no earlier than 3 months ago. The patient associates this node with stress. She
doesn't complain either about pain or enlargement of the node. Ultrasonic scanning revealed a 2x2,5
cm large node in the inferior part of the right lobe of thyroid gland. What treatment should be
administered?
Select one:
a. Told you to stop guessing
b. Surgical intervention
c. Conservative therapy
d. Dynamic observation
e. No need for treatment
Feedback
The correct answer is: Surgical intervention
Question 163
Not answered
Mark 0.00 out of 1.00
Flag question
Question text
Examination of a 3-month-old child revealed scrotum growth on the right. This formation has elastic
consistency, its size decreases during sleep and increases when the child is crying. What
examination will be helpful for making a correct diagnosis?
Select one:
a. Palpation of the thickened cord crossing the pubical tubercule (sign of the silk glove)
b. Examination of the formation in Trendelenburg's position
c. Palpation of the external inguinal ring
d. Diaphanoscоpy
e. Punction of the scrotum
Feedback
The correct answer is: Palpation of the thickened cord crossing the pubical tubercule (sign of the silk
glove)
Book 2009 krok 2
Book 2009 krok 2
Book 2009 krok 2
Book 2009 krok 2
Book 2009 krok 2
Book 2009 krok 2
Book 2009 krok 2
Book 2009 krok 2
Book 2009 krok 2
Book 2009 krok 2
Book 2009 krok 2
Book 2009 krok 2
Book 2009 krok 2
Book 2009 krok 2
Book 2009 krok 2
Book 2009 krok 2
Book 2009 krok 2
Book 2009 krok 2
Book 2009 krok 2
Book 2009 krok 2
Book 2009 krok 2
Book 2009 krok 2

More Related Content

What's hot

Krok 2 - 2009 Question Paper (General Medicine)
Krok 2 - 2009 Question Paper (General Medicine)Krok 2 - 2009 Question Paper (General Medicine)
Krok 2 - 2009 Question Paper (General Medicine)
Eneutron
 
Booklet 2015 krok 2
Booklet 2015 krok 2Booklet 2015 krok 2
Booklet 2015 krok 2
Raj Twix
 
Krok 2 - 2006 Question Paper (General Medicine)
Krok 2 - 2006 Question Paper (General Medicine)Krok 2 - 2006 Question Paper (General Medicine)
Krok 2 - 2006 Question Paper (General Medicine)
Eneutron
 
Krok 2 - 2010 Question Paper (General Medicine)
Krok 2 - 2010 Question Paper (General Medicine)Krok 2 - 2010 Question Paper (General Medicine)
Krok 2 - 2010 Question Paper (General Medicine)
Eneutron
 
Krok 2 - 2014 (Surgery)
Krok 2 - 2014 (Surgery)Krok 2 - 2014 (Surgery)
Krok 2 - 2014 (Surgery)
Eneutron
 
Krok 2 - 2013 Question Paper (General Medicine)
Krok 2 - 2013 Question Paper (General Medicine)Krok 2 - 2013 Question Paper (General Medicine)
Krok 2 - 2013 Question Paper (General Medicine)
Eneutron
 
Krok 2 - 2012 Question Paper (General Medicine)
Krok 2 - 2012 Question Paper (General Medicine)Krok 2 - 2012 Question Paper (General Medicine)
Krok 2 - 2012 Question Paper (General Medicine)
Eneutron
 
Krok 2 - 2013 (Hygiene)
Krok 2 - 2013 (Hygiene)Krok 2 - 2013 (Hygiene)
Krok 2 - 2013 (Hygiene)
Eneutron
 
Krok 1 - 2015 (Path-Anatomy)
Krok 1 - 2015 (Path-Anatomy)Krok 1 - 2015 (Path-Anatomy)
Krok 1 - 2015 (Path-Anatomy)
Eneutron
 
Krok 1 - 2014 (Path-Anatomy)
Krok 1 - 2014 (Path-Anatomy)Krok 1 - 2014 (Path-Anatomy)
Krok 1 - 2014 (Path-Anatomy)
Eneutron
 
Krok 2 - 2008 Question Paper (General Medicine)
Krok 2 - 2008 Question Paper (General Medicine)Krok 2 - 2008 Question Paper (General Medicine)
Krok 2 - 2008 Question Paper (General Medicine)
Eneutron
 
Krok 2 - 2007 Question Paper (General Medicine)
Krok 2 - 2007 Question Paper (General Medicine)Krok 2 - 2007 Question Paper (General Medicine)
Krok 2 - 2007 Question Paper (General Medicine)
Eneutron
 
Pediatric 2013 krok 2
Pediatric 2013 krok 2Pediatric 2013 krok 2
Pediatric 2013 krok 2
Raj Twix
 
Pulmonology mcqs -dr.ahmed_mowafy
Pulmonology mcqs -dr.ahmed_mowafyPulmonology mcqs -dr.ahmed_mowafy
Pulmonology mcqs -dr.ahmed_mowafy
czer Shmary
 
Krok 2 - 2014 Question Paper (General Medicine)
Krok 2 - 2014 Question Paper (General Medicine)Krok 2 - 2014 Question Paper (General Medicine)
Krok 2 - 2014 Question Paper (General Medicine)
Eneutron
 
Krok 2 Medicine - 2017 Question Paper
Krok 2 Medicine - 2017 Question PaperKrok 2 Medicine - 2017 Question Paper
Krok 2 Medicine - 2017 Question Paper
Eneutron
 
Nepal nursing council licencing exam mcqs sample
Nepal nursing council licencing exam mcqs sampleNepal nursing council licencing exam mcqs sample
Nepal nursing council licencing exam mcqs sample
BP KOIRALA INSTITUTE OF HELATH SCIENCS,, NEPAL
 
Krok 2 - 2014 (Gynecology)
Krok 2 - 2014 (Gynecology)Krok 2 - 2014 (Gynecology)
Krok 2 - 2014 (Gynecology)
Eneutron
 
Krok 2 - 2014 (Hygiene)
Krok 2 - 2014 (Hygiene)Krok 2 - 2014 (Hygiene)
Krok 2 - 2014 (Hygiene)
Eneutron
 
Rheumatology MCQs Practice questions with explanation
Rheumatology MCQs Practice questions with explanationRheumatology MCQs Practice questions with explanation
Rheumatology MCQs Practice questions with explanation
Dr. Almas A
 

What's hot (20)

Krok 2 - 2009 Question Paper (General Medicine)
Krok 2 - 2009 Question Paper (General Medicine)Krok 2 - 2009 Question Paper (General Medicine)
Krok 2 - 2009 Question Paper (General Medicine)
 
Booklet 2015 krok 2
Booklet 2015 krok 2Booklet 2015 krok 2
Booklet 2015 krok 2
 
Krok 2 - 2006 Question Paper (General Medicine)
Krok 2 - 2006 Question Paper (General Medicine)Krok 2 - 2006 Question Paper (General Medicine)
Krok 2 - 2006 Question Paper (General Medicine)
 
Krok 2 - 2010 Question Paper (General Medicine)
Krok 2 - 2010 Question Paper (General Medicine)Krok 2 - 2010 Question Paper (General Medicine)
Krok 2 - 2010 Question Paper (General Medicine)
 
Krok 2 - 2014 (Surgery)
Krok 2 - 2014 (Surgery)Krok 2 - 2014 (Surgery)
Krok 2 - 2014 (Surgery)
 
Krok 2 - 2013 Question Paper (General Medicine)
Krok 2 - 2013 Question Paper (General Medicine)Krok 2 - 2013 Question Paper (General Medicine)
Krok 2 - 2013 Question Paper (General Medicine)
 
Krok 2 - 2012 Question Paper (General Medicine)
Krok 2 - 2012 Question Paper (General Medicine)Krok 2 - 2012 Question Paper (General Medicine)
Krok 2 - 2012 Question Paper (General Medicine)
 
Krok 2 - 2013 (Hygiene)
Krok 2 - 2013 (Hygiene)Krok 2 - 2013 (Hygiene)
Krok 2 - 2013 (Hygiene)
 
Krok 1 - 2015 (Path-Anatomy)
Krok 1 - 2015 (Path-Anatomy)Krok 1 - 2015 (Path-Anatomy)
Krok 1 - 2015 (Path-Anatomy)
 
Krok 1 - 2014 (Path-Anatomy)
Krok 1 - 2014 (Path-Anatomy)Krok 1 - 2014 (Path-Anatomy)
Krok 1 - 2014 (Path-Anatomy)
 
Krok 2 - 2008 Question Paper (General Medicine)
Krok 2 - 2008 Question Paper (General Medicine)Krok 2 - 2008 Question Paper (General Medicine)
Krok 2 - 2008 Question Paper (General Medicine)
 
Krok 2 - 2007 Question Paper (General Medicine)
Krok 2 - 2007 Question Paper (General Medicine)Krok 2 - 2007 Question Paper (General Medicine)
Krok 2 - 2007 Question Paper (General Medicine)
 
Pediatric 2013 krok 2
Pediatric 2013 krok 2Pediatric 2013 krok 2
Pediatric 2013 krok 2
 
Pulmonology mcqs -dr.ahmed_mowafy
Pulmonology mcqs -dr.ahmed_mowafyPulmonology mcqs -dr.ahmed_mowafy
Pulmonology mcqs -dr.ahmed_mowafy
 
Krok 2 - 2014 Question Paper (General Medicine)
Krok 2 - 2014 Question Paper (General Medicine)Krok 2 - 2014 Question Paper (General Medicine)
Krok 2 - 2014 Question Paper (General Medicine)
 
Krok 2 Medicine - 2017 Question Paper
Krok 2 Medicine - 2017 Question PaperKrok 2 Medicine - 2017 Question Paper
Krok 2 Medicine - 2017 Question Paper
 
Nepal nursing council licencing exam mcqs sample
Nepal nursing council licencing exam mcqs sampleNepal nursing council licencing exam mcqs sample
Nepal nursing council licencing exam mcqs sample
 
Krok 2 - 2014 (Gynecology)
Krok 2 - 2014 (Gynecology)Krok 2 - 2014 (Gynecology)
Krok 2 - 2014 (Gynecology)
 
Krok 2 - 2014 (Hygiene)
Krok 2 - 2014 (Hygiene)Krok 2 - 2014 (Hygiene)
Krok 2 - 2014 (Hygiene)
 
Rheumatology MCQs Practice questions with explanation
Rheumatology MCQs Practice questions with explanationRheumatology MCQs Practice questions with explanation
Rheumatology MCQs Practice questions with explanation
 

Viewers also liked

Gynac 170mcq krok 2
Gynac 170mcq krok 2Gynac 170mcq krok 2
Gynac 170mcq krok 2
Raj Twix
 
Therapeutic 176 additional krok 2
Therapeutic 176 additional krok 2Therapeutic 176 additional krok 2
Therapeutic 176 additional krok 2
Raj Twix
 
Booklet 2013 krok 2
Booklet 2013 krok 2Booklet 2013 krok 2
Booklet 2013 krok 2
Raj Twix
 
Book 2007 krok 2
Book 2007 krok 2Book 2007 krok 2
Book 2007 krok 2
Raj Twix
 
Hygienic addi 134 krok 2
Hygienic addi 134 krok 2Hygienic addi 134 krok 2
Hygienic addi 134 krok 2
Raj Twix
 
Hygien 2013 krok 2
Hygien 2013 krok 2Hygien 2013 krok 2
Hygien 2013 krok 2
Raj Twix
 
Project Monitoring and Controlling Processes
Project Monitoring and Controlling ProcessesProject Monitoring and Controlling Processes
Project Monitoring and Controlling Processes
Nishant Munjal
 
Research ethics
Research ethicsResearch ethics
Research ethics
Suleyman Ally
 
Microbiology Virology book MBBS
Microbiology Virology book MBBSMicrobiology Virology book MBBS
Microbiology Virology book MBBS
Raj Twix
 
Occupational book internal medicine 5th year BSMU
Occupational book internal medicine 5th year BSMUOccupational book internal medicine 5th year BSMU
Occupational book internal medicine 5th year BSMU
Raj Twix
 
ETS Services Outline
ETS Services OutlineETS Services Outline
ETS Services Outline
Tony DeGonia (LION)
 
Book 2011 krok 2
Book 2011 krok 2Book 2011 krok 2
Book 2011 krok 2
Raj Twix
 
Lenovo Presentation for Sys Logic Lunch and Learn
Lenovo Presentation for Sys Logic Lunch and LearnLenovo Presentation for Sys Logic Lunch and Learn
Lenovo Presentation for Sys Logic Lunch and Learn
Tony DeGonia (LION)
 
Why Social Media?
Why Social Media?Why Social Media?
Why Social Media?
Gautam Shravan
 
Wales
WalesWales
Wales
mlex3000
 
Hematology-urgent condition book internal medicine 5th year BSMU
Hematology-urgent condition book internal medicine 5th year BSMUHematology-urgent condition book internal medicine 5th year BSMU
Hematology-urgent condition book internal medicine 5th year BSMU
Raj Twix
 
Gync additional krok 2
Gync additional krok 2Gync additional krok 2
Gync additional krok 2
Raj Twix
 
Book 2008 krok 2
Book 2008 krok 2Book 2008 krok 2
Book 2008 krok 2
Raj Twix
 
Pediatric additional krok 2
Pediatric additional krok 2Pediatric additional krok 2
Pediatric additional krok 2
Raj Twix
 
Flip board in education by Shaikhah Almutairi
Flip board in education by Shaikhah AlmutairiFlip board in education by Shaikhah Almutairi
Flip board in education by Shaikhah Almutairi
shaikhah1404
 

Viewers also liked (20)

Gynac 170mcq krok 2
Gynac 170mcq krok 2Gynac 170mcq krok 2
Gynac 170mcq krok 2
 
Therapeutic 176 additional krok 2
Therapeutic 176 additional krok 2Therapeutic 176 additional krok 2
Therapeutic 176 additional krok 2
 
Booklet 2013 krok 2
Booklet 2013 krok 2Booklet 2013 krok 2
Booklet 2013 krok 2
 
Book 2007 krok 2
Book 2007 krok 2Book 2007 krok 2
Book 2007 krok 2
 
Hygienic addi 134 krok 2
Hygienic addi 134 krok 2Hygienic addi 134 krok 2
Hygienic addi 134 krok 2
 
Hygien 2013 krok 2
Hygien 2013 krok 2Hygien 2013 krok 2
Hygien 2013 krok 2
 
Project Monitoring and Controlling Processes
Project Monitoring and Controlling ProcessesProject Monitoring and Controlling Processes
Project Monitoring and Controlling Processes
 
Research ethics
Research ethicsResearch ethics
Research ethics
 
Microbiology Virology book MBBS
Microbiology Virology book MBBSMicrobiology Virology book MBBS
Microbiology Virology book MBBS
 
Occupational book internal medicine 5th year BSMU
Occupational book internal medicine 5th year BSMUOccupational book internal medicine 5th year BSMU
Occupational book internal medicine 5th year BSMU
 
ETS Services Outline
ETS Services OutlineETS Services Outline
ETS Services Outline
 
Book 2011 krok 2
Book 2011 krok 2Book 2011 krok 2
Book 2011 krok 2
 
Lenovo Presentation for Sys Logic Lunch and Learn
Lenovo Presentation for Sys Logic Lunch and LearnLenovo Presentation for Sys Logic Lunch and Learn
Lenovo Presentation for Sys Logic Lunch and Learn
 
Why Social Media?
Why Social Media?Why Social Media?
Why Social Media?
 
Wales
WalesWales
Wales
 
Hematology-urgent condition book internal medicine 5th year BSMU
Hematology-urgent condition book internal medicine 5th year BSMUHematology-urgent condition book internal medicine 5th year BSMU
Hematology-urgent condition book internal medicine 5th year BSMU
 
Gync additional krok 2
Gync additional krok 2Gync additional krok 2
Gync additional krok 2
 
Book 2008 krok 2
Book 2008 krok 2Book 2008 krok 2
Book 2008 krok 2
 
Pediatric additional krok 2
Pediatric additional krok 2Pediatric additional krok 2
Pediatric additional krok 2
 
Flip board in education by Shaikhah Almutairi
Flip board in education by Shaikhah AlmutairiFlip board in education by Shaikhah Almutairi
Flip board in education by Shaikhah Almutairi
 

Similar to Book 2009 krok 2

Book 2006 krok-2
Book 2006 krok-2Book 2006 krok-2
Book 2006 krok-2
Raj Twix
 
Therapy 2013 krok 2
Therapy 2013 krok 2Therapy 2013 krok 2
Therapy 2013 krok 2
Raj Twix
 
Therapeutic 271 krok 2
Therapeutic 271 krok 2Therapeutic 271 krok 2
Therapeutic 271 krok 2
Raj Twix
 
Surgey addi krok 2
Surgey addi krok 2Surgey addi krok 2
Surgey addi krok 2
Raj Twix
 
Surgery 209 krok 2
Surgery 209 krok 2Surgery 209 krok 2
Surgery 209 krok 2
Raj Twix
 
History taking a case based discussion
History taking a case based discussionHistory taking a case based discussion
History taking a case based discussion
Pritom Das
 
Document
DocumentDocument
Segundo simulador primera parte
Segundo simulador primera parteSegundo simulador primera parte
Segundo simulador primera parte
Pharmed Solutions Institute
 
Krok2 - Medicine - 2012.pdfKrok2 - Medicine - 2012.pdf
Krok2 - Medicine - 2012.pdfKrok2 - Medicine - 2012.pdfKrok2 - Medicine - 2012.pdfKrok2 - Medicine - 2012.pdf
Krok2 - Medicine - 2012.pdfKrok2 - Medicine - 2012.pdf
trinhtruong44
 
Internal Medicine Board Review
Internal Medicine  Board ReviewInternal Medicine  Board Review
Internal Medicine Board Review
jcm MD
 
Gd oar 3.12
Gd oar 3.12Gd oar 3.12
Gd oar 3.12
sean_kono
 
Questions 1700 - Medical MCQ without Answer
Questions 1700 - Medical MCQ without AnswerQuestions 1700 - Medical MCQ without Answer
Questions 1700 - Medical MCQ without Answer
MBBS Help
 
Krok 2 - 2013 (Pediatrics)
Krok 2 - 2013 (Pediatrics)Krok 2 - 2013 (Pediatrics)
Krok 2 - 2013 (Pediatrics)
Eneutron
 
Nclex test review
Nclex test reviewNclex test review
Nclex test review
merris35
 
2017留学生试卷-2021-2022-2线上试题-A卷.pdf
2017留学生试卷-2021-2022-2线上试题-A卷.pdf2017留学生试卷-2021-2022-2线上试题-A卷.pdf
2017留学生试卷-2021-2022-2线上试题-A卷.pdf
SamikshyaKharel
 
Mcq. medical surgical nursing
Mcq. medical surgical nursingMcq. medical surgical nursing
Mcq. medical surgical nursing
Nursing Path
 

Similar to Book 2009 krok 2 (16)

Book 2006 krok-2
Book 2006 krok-2Book 2006 krok-2
Book 2006 krok-2
 
Therapy 2013 krok 2
Therapy 2013 krok 2Therapy 2013 krok 2
Therapy 2013 krok 2
 
Therapeutic 271 krok 2
Therapeutic 271 krok 2Therapeutic 271 krok 2
Therapeutic 271 krok 2
 
Surgey addi krok 2
Surgey addi krok 2Surgey addi krok 2
Surgey addi krok 2
 
Surgery 209 krok 2
Surgery 209 krok 2Surgery 209 krok 2
Surgery 209 krok 2
 
History taking a case based discussion
History taking a case based discussionHistory taking a case based discussion
History taking a case based discussion
 
Document
DocumentDocument
Document
 
Segundo simulador primera parte
Segundo simulador primera parteSegundo simulador primera parte
Segundo simulador primera parte
 
Krok2 - Medicine - 2012.pdfKrok2 - Medicine - 2012.pdf
Krok2 - Medicine - 2012.pdfKrok2 - Medicine - 2012.pdfKrok2 - Medicine - 2012.pdfKrok2 - Medicine - 2012.pdf
Krok2 - Medicine - 2012.pdfKrok2 - Medicine - 2012.pdf
 
Internal Medicine Board Review
Internal Medicine  Board ReviewInternal Medicine  Board Review
Internal Medicine Board Review
 
Gd oar 3.12
Gd oar 3.12Gd oar 3.12
Gd oar 3.12
 
Questions 1700 - Medical MCQ without Answer
Questions 1700 - Medical MCQ without AnswerQuestions 1700 - Medical MCQ without Answer
Questions 1700 - Medical MCQ without Answer
 
Krok 2 - 2013 (Pediatrics)
Krok 2 - 2013 (Pediatrics)Krok 2 - 2013 (Pediatrics)
Krok 2 - 2013 (Pediatrics)
 
Nclex test review
Nclex test reviewNclex test review
Nclex test review
 
2017留学生试卷-2021-2022-2线上试题-A卷.pdf
2017留学生试卷-2021-2022-2线上试题-A卷.pdf2017留学生试卷-2021-2022-2线上试题-A卷.pdf
2017留学生试卷-2021-2022-2线上试题-A卷.pdf
 
Mcq. medical surgical nursing
Mcq. medical surgical nursingMcq. medical surgical nursing
Mcq. medical surgical nursing
 

Recently uploaded

How to Add Chatter in the odoo 17 ERP Module
How to Add Chatter in the odoo 17 ERP ModuleHow to Add Chatter in the odoo 17 ERP Module
How to Add Chatter in the odoo 17 ERP Module
Celine George
 
LAND USE LAND COVER AND NDVI OF MIRZAPUR DISTRICT, UP
LAND USE LAND COVER AND NDVI OF MIRZAPUR DISTRICT, UPLAND USE LAND COVER AND NDVI OF MIRZAPUR DISTRICT, UP
LAND USE LAND COVER AND NDVI OF MIRZAPUR DISTRICT, UP
RAHUL
 
South African Journal of Science: Writing with integrity workshop (2024)
South African Journal of Science: Writing with integrity workshop (2024)South African Journal of Science: Writing with integrity workshop (2024)
South African Journal of Science: Writing with integrity workshop (2024)
Academy of Science of South Africa
 
World environment day ppt For 5 June 2024
World environment day ppt For 5 June 2024World environment day ppt For 5 June 2024
World environment day ppt For 5 June 2024
ak6969907
 
S1-Introduction-Biopesticides in ICM.pptx
S1-Introduction-Biopesticides in ICM.pptxS1-Introduction-Biopesticides in ICM.pptx
S1-Introduction-Biopesticides in ICM.pptx
tarandeep35
 
Azure Interview Questions and Answers PDF By ScholarHat
Azure Interview Questions and Answers PDF By ScholarHatAzure Interview Questions and Answers PDF By ScholarHat
Azure Interview Questions and Answers PDF By ScholarHat
Scholarhat
 
Natural birth techniques - Mrs.Akanksha Trivedi Rama University
Natural birth techniques - Mrs.Akanksha Trivedi Rama UniversityNatural birth techniques - Mrs.Akanksha Trivedi Rama University
Natural birth techniques - Mrs.Akanksha Trivedi Rama University
Akanksha trivedi rama nursing college kanpur.
 
Community pharmacy- Social and preventive pharmacy UNIT 5
Community pharmacy- Social and preventive pharmacy UNIT 5Community pharmacy- Social and preventive pharmacy UNIT 5
Community pharmacy- Social and preventive pharmacy UNIT 5
sayalidalavi006
 
A Independência da América Espanhola LAPBOOK.pdf
A Independência da América Espanhola LAPBOOK.pdfA Independência da América Espanhola LAPBOOK.pdf
A Independência da América Espanhola LAPBOOK.pdf
Jean Carlos Nunes Paixão
 
The Diamonds of 2023-2024 in the IGRA collection
The Diamonds of 2023-2024 in the IGRA collectionThe Diamonds of 2023-2024 in the IGRA collection
The Diamonds of 2023-2024 in the IGRA collection
Israel Genealogy Research Association
 
PIMS Job Advertisement 2024.pdf Islamabad
PIMS Job Advertisement 2024.pdf IslamabadPIMS Job Advertisement 2024.pdf Islamabad
PIMS Job Advertisement 2024.pdf Islamabad
AyyanKhan40
 
Advanced Java[Extra Concepts, Not Difficult].docx
Advanced Java[Extra Concepts, Not Difficult].docxAdvanced Java[Extra Concepts, Not Difficult].docx
Advanced Java[Extra Concepts, Not Difficult].docx
adhitya5119
 
RPMS TEMPLATE FOR SCHOOL YEAR 2023-2024 FOR TEACHER 1 TO TEACHER 3
RPMS TEMPLATE FOR SCHOOL YEAR 2023-2024 FOR TEACHER 1 TO TEACHER 3RPMS TEMPLATE FOR SCHOOL YEAR 2023-2024 FOR TEACHER 1 TO TEACHER 3
RPMS TEMPLATE FOR SCHOOL YEAR 2023-2024 FOR TEACHER 1 TO TEACHER 3
IreneSebastianRueco1
 
Pride Month Slides 2024 David Douglas School District
Pride Month Slides 2024 David Douglas School DistrictPride Month Slides 2024 David Douglas School District
Pride Month Slides 2024 David Douglas School District
David Douglas School District
 
ISO/IEC 27001, ISO/IEC 42001, and GDPR: Best Practices for Implementation and...
ISO/IEC 27001, ISO/IEC 42001, and GDPR: Best Practices for Implementation and...ISO/IEC 27001, ISO/IEC 42001, and GDPR: Best Practices for Implementation and...
ISO/IEC 27001, ISO/IEC 42001, and GDPR: Best Practices for Implementation and...
PECB
 
MARY JANE WILSON, A “BOA MÃE” .
MARY JANE WILSON, A “BOA MÃE”           .MARY JANE WILSON, A “BOA MÃE”           .
MARY JANE WILSON, A “BOA MÃE” .
Colégio Santa Teresinha
 
Exploiting Artificial Intelligence for Empowering Researchers and Faculty, In...
Exploiting Artificial Intelligence for Empowering Researchers and Faculty, In...Exploiting Artificial Intelligence for Empowering Researchers and Faculty, In...
Exploiting Artificial Intelligence for Empowering Researchers and Faculty, In...
Dr. Vinod Kumar Kanvaria
 
How to Setup Warehouse & Location in Odoo 17 Inventory
How to Setup Warehouse & Location in Odoo 17 InventoryHow to Setup Warehouse & Location in Odoo 17 Inventory
How to Setup Warehouse & Location in Odoo 17 Inventory
Celine George
 
How to Make a Field Mandatory in Odoo 17
How to Make a Field Mandatory in Odoo 17How to Make a Field Mandatory in Odoo 17
How to Make a Field Mandatory in Odoo 17
Celine George
 
How to Build a Module in Odoo 17 Using the Scaffold Method
How to Build a Module in Odoo 17 Using the Scaffold MethodHow to Build a Module in Odoo 17 Using the Scaffold Method
How to Build a Module in Odoo 17 Using the Scaffold Method
Celine George
 

Recently uploaded (20)

How to Add Chatter in the odoo 17 ERP Module
How to Add Chatter in the odoo 17 ERP ModuleHow to Add Chatter in the odoo 17 ERP Module
How to Add Chatter in the odoo 17 ERP Module
 
LAND USE LAND COVER AND NDVI OF MIRZAPUR DISTRICT, UP
LAND USE LAND COVER AND NDVI OF MIRZAPUR DISTRICT, UPLAND USE LAND COVER AND NDVI OF MIRZAPUR DISTRICT, UP
LAND USE LAND COVER AND NDVI OF MIRZAPUR DISTRICT, UP
 
South African Journal of Science: Writing with integrity workshop (2024)
South African Journal of Science: Writing with integrity workshop (2024)South African Journal of Science: Writing with integrity workshop (2024)
South African Journal of Science: Writing with integrity workshop (2024)
 
World environment day ppt For 5 June 2024
World environment day ppt For 5 June 2024World environment day ppt For 5 June 2024
World environment day ppt For 5 June 2024
 
S1-Introduction-Biopesticides in ICM.pptx
S1-Introduction-Biopesticides in ICM.pptxS1-Introduction-Biopesticides in ICM.pptx
S1-Introduction-Biopesticides in ICM.pptx
 
Azure Interview Questions and Answers PDF By ScholarHat
Azure Interview Questions and Answers PDF By ScholarHatAzure Interview Questions and Answers PDF By ScholarHat
Azure Interview Questions and Answers PDF By ScholarHat
 
Natural birth techniques - Mrs.Akanksha Trivedi Rama University
Natural birth techniques - Mrs.Akanksha Trivedi Rama UniversityNatural birth techniques - Mrs.Akanksha Trivedi Rama University
Natural birth techniques - Mrs.Akanksha Trivedi Rama University
 
Community pharmacy- Social and preventive pharmacy UNIT 5
Community pharmacy- Social and preventive pharmacy UNIT 5Community pharmacy- Social and preventive pharmacy UNIT 5
Community pharmacy- Social and preventive pharmacy UNIT 5
 
A Independência da América Espanhola LAPBOOK.pdf
A Independência da América Espanhola LAPBOOK.pdfA Independência da América Espanhola LAPBOOK.pdf
A Independência da América Espanhola LAPBOOK.pdf
 
The Diamonds of 2023-2024 in the IGRA collection
The Diamonds of 2023-2024 in the IGRA collectionThe Diamonds of 2023-2024 in the IGRA collection
The Diamonds of 2023-2024 in the IGRA collection
 
PIMS Job Advertisement 2024.pdf Islamabad
PIMS Job Advertisement 2024.pdf IslamabadPIMS Job Advertisement 2024.pdf Islamabad
PIMS Job Advertisement 2024.pdf Islamabad
 
Advanced Java[Extra Concepts, Not Difficult].docx
Advanced Java[Extra Concepts, Not Difficult].docxAdvanced Java[Extra Concepts, Not Difficult].docx
Advanced Java[Extra Concepts, Not Difficult].docx
 
RPMS TEMPLATE FOR SCHOOL YEAR 2023-2024 FOR TEACHER 1 TO TEACHER 3
RPMS TEMPLATE FOR SCHOOL YEAR 2023-2024 FOR TEACHER 1 TO TEACHER 3RPMS TEMPLATE FOR SCHOOL YEAR 2023-2024 FOR TEACHER 1 TO TEACHER 3
RPMS TEMPLATE FOR SCHOOL YEAR 2023-2024 FOR TEACHER 1 TO TEACHER 3
 
Pride Month Slides 2024 David Douglas School District
Pride Month Slides 2024 David Douglas School DistrictPride Month Slides 2024 David Douglas School District
Pride Month Slides 2024 David Douglas School District
 
ISO/IEC 27001, ISO/IEC 42001, and GDPR: Best Practices for Implementation and...
ISO/IEC 27001, ISO/IEC 42001, and GDPR: Best Practices for Implementation and...ISO/IEC 27001, ISO/IEC 42001, and GDPR: Best Practices for Implementation and...
ISO/IEC 27001, ISO/IEC 42001, and GDPR: Best Practices for Implementation and...
 
MARY JANE WILSON, A “BOA MÃE” .
MARY JANE WILSON, A “BOA MÃE”           .MARY JANE WILSON, A “BOA MÃE”           .
MARY JANE WILSON, A “BOA MÃE” .
 
Exploiting Artificial Intelligence for Empowering Researchers and Faculty, In...
Exploiting Artificial Intelligence for Empowering Researchers and Faculty, In...Exploiting Artificial Intelligence for Empowering Researchers and Faculty, In...
Exploiting Artificial Intelligence for Empowering Researchers and Faculty, In...
 
How to Setup Warehouse & Location in Odoo 17 Inventory
How to Setup Warehouse & Location in Odoo 17 InventoryHow to Setup Warehouse & Location in Odoo 17 Inventory
How to Setup Warehouse & Location in Odoo 17 Inventory
 
How to Make a Field Mandatory in Odoo 17
How to Make a Field Mandatory in Odoo 17How to Make a Field Mandatory in Odoo 17
How to Make a Field Mandatory in Odoo 17
 
How to Build a Module in Odoo 17 Using the Scaffold Method
How to Build a Module in Odoo 17 Using the Scaffold MethodHow to Build a Module in Odoo 17 Using the Scaffold Method
How to Build a Module in Odoo 17 Using the Scaffold Method
 

Book 2009 krok 2

  • 1. 1,5 hour after start of gullet bougienage a 48 year old patient suffering from corrosive stricture felt acute abdominal pain. Previously he had been suffering from duodenal ulcer. Examination revealed that abdomen was very tense and painful; Ps- 110 bpm, painful sialophagia, skin pallor. What is the most probable diagnosis? Select one: a. Thrombosis of mesenteric vessels b. Perforation of duodenal ulcer c. Strangulation of diaphragmal hernia d. Perforation of abdominal part of esophagus e. Acute myocardium infarction Feedback The correct answer is: Perforation of abdominal part of esophagus Question 2 Not answered Mark 0.00 out of 1.00 Flag question Question text 15 minutes after the second vaccination with diphteria and tetanus toxoids and pertussis vaccine a 4 month old boy manifested symptoms of Quincke's edema. What medication should be applied for emergency aid? Select one: a. Furosemide b. Prednisolone c. Seduxen d. Heparin e. Adrenalin Feedback The correct answer is: Prednisolone Question 3 Not answered Mark 0.00 out of 1.00
  • 2. Flag question Question text 25 children at the age of 2-3 years who don't attend any child welfare institutions should be observed by a district pediatrician within the current year. How many initial visits of this group of children should be planned? Select one: a. 50 b. 100 c. 20 d. 40 e. 200 Feedback The correct answer is: 50 Question 4 Not answered Mark 0.00 out of 1.00 Flag question Question text A 10 month old boy has been ill for 5 days after consumption of unboiled milk. Body temperature is 38-39oC, there is vomiting, liquid stool. The child is pale and inert. His tongue is covered with white deposition. Heart sounds are muffled. Abdomen is swollen, there is borborygmus in the region of ubbilicus, liver is enlarged by 3 cm. Stool is liquid, dark-green, with admixtures of mucus, 5 times a day. What is the most probable diagnosis? Select one: a. Staphylococcal enteric infection b. Escherichiosis c. Salmonellosis d. Acute shigellosis
  • 3. e. Rotaviral infection Feedback The correct answer is: Salmonellosis Question 5 Not answered Mark 0.00 out of 1.00 Flag question Question text A 10 year old boy complains about pain in his left eye and strong photophobia after he had injured his left eye with a pencil at school. Left eye examination: blepharospasm, ciliary and conjunctival congestion, cornea is transparent, other parts of eyeball have no changes. Visus 0,9. Right eye is healthy, Visus 1,0. What additional method would you choose first of all? Select one: a. Tonometria b. Staining test with 1% fluorescein c. X-ray examination of orbit d. Cornea sensation-test e. Gonioscopia Feedback The correct answer is: Staining test with 1% fluorescein Question 6 Not answered Mark 0.00 out of 1.00 Flag question Question text A 10 year old boy suffers from chronic viral hepatitis type B with maximal activity. What laboratory test can give the most precise characteristic of cytolysis degree?
  • 4. Select one: a. Weltman's coagulation test b. Takata-Ara test c. Transaminase test d. Test for whole protein e. Prothrombin test Feedback The correct answer is: Transaminase test Question 7 Not answered Mark 0.00 out of 1.00 Flag question Question text A 12 year old child has the ulcer disease of stomach. What is the etiology of this disease? Select one: a. Helicobacter pylory b. Salmonella c. Lambliosis d. Intestinal bacillus e. Influenza Feedback The correct answer is: Intestinal bacillus Question 8 Not answered Mark 0.00 out of 1.00 Flag question
  • 5. Question text A 13 year old girl was admitted to the cardiological department because of pain in the muscles and joints. Examination of her face revealed an edematic erythema in form of butterfly in the region of nose bridge and cheeks. What is the most probable diagnosis? Select one: a. Periarteritis nodosa b. Dermatomyositis c. Systemic lupus erythematosus d. Rheumatism e. Rheumatoid arthritis Feedback The correct answer is: Systemic lupus erythematosus Question 9 Not answered Mark 0.00 out of 1.00 Flag question Question text A 13 year old patient is suffering from an acute disease with the following symptoms: thirst, polyuria, weakness. Objectively: his general condition is satisfactory, there is no smell of acetone. Glucose concentration in blood on an empty stomach is 32 micromole/l, in urine - 6%, acetone +. What treatment should be administered? Select one: a. Diet b. Biguanides c. Long-acting insulin d. Sulfonylurea e. Short-acting insulin Feedback The correct answer is: Short-acting insulin
  • 6. Question 10 Not answered Mark 0.00 out of 1.00 Flag question Question text A 14 year old child suffers from vegetovascular dystonia of pubertal period. He has got sympathoadrenal atack. What medicine should be used for attack reduction? Select one: a. Aminophylline b. Amysyl c. Corglicone d. Obsidan e. No-shpa Feedback The correct answer is: Obsidan Question 11 Not answered Mark 0.00 out of 1.00 Flag question Question text A 14 year old girl complains of profuse bloody discharges from genital tracts during 10 days after suppresion of menses for 1,5 month. Similiar bleedings recur since 12 years on the background of disordered menstrual cycle. On rectal examination: no pathology of the internal genitalia. In blood: Нb - 70 g/l, RBC- 2,3*1012/l, Ht - 20. What is the most probable diagnosis? Select one: a. Incomplete spontaneous abortion b. Hormonoproductive ovary tumor c. Polycyst ovarian syndrome d. Juvenile bleeding, posthemorrhagic anemia
  • 7. e. Werlholf's disease Feedback The correct answer is: Juvenile bleeding, posthemorrhagic anemia Question 12 Not answered Mark 0.00 out of 1.00 Flag question Question text A 15 year old girl suddenly got arthralgia, headache, nausea, vomiting; pain and muscle tension in the lumbar area; body temperature rose up to 38-39oC. Pasternatsky's symptom was distinctly positive on the right. In the urine: bacteriuria, pyuria. What is the most probable diagnosis? Select one: a. Acute pyelonephritis b. Acute glomerulonephritis c. Renal colic d. Pararenal abscess e. Cystitis Feedback The correct answer is: Acute pyelonephritis Question 13 Not answered Mark 0.00 out of 1.00 Flag question Question text A 16 year old female patient underwent an operation on account of diffuse toxic goiter of the III-IV degree 12 years ago. Now she has recurrence of thyrotoxicosis. The patient was offered operative
  • 8. intervention, but it is necessary first to localize the functioning gland tissue. What method should be applied for this purpose? Select one: a. Roentgenography of esophagus b. Roentgenography of neck c. Gland scanning d. Puncture aspiration biopsy e. USI Feedback The correct answer is: Gland scanning Question 14 Not answered Mark 0.00 out of 1.00 Flag question Question text A 19 year old boy was admitted to a hospital with closed abdominal trauma. In course of operation multiple ruptures of spleen and small intestine were revealed. AP is falling rapidly, it is necessary to perform hemotransfusion. Who can specify the patient's blood group and rhesus compatibility? Select one: a. A traumatologist b. A surgeon c. A laboratory physician d. An anaesthesilogist e. A doctor of any speciality Feedback The correct answer is: A doctor of any speciality Question 15 Not answered Mark 0.00 out of 1.00
  • 9. Flag question Question text A 19 year old patient was admitted to a hospital with acute destructive appendicitis. He suffers from hemophilia of B type. What antihemophilic medications should be included in pre- and post-operative treatment plan? Select one: a. Fresh frozen blood b. Cryoprecipitate c. Fresh frozen plasma d. Dried plasma e. Native plasma Feedback The correct answer is: Fresh frozen plasma Question 16 Not answered Mark 0.00 out of 1.00 Flag question Question text A 2 month old full-term child was born with weight 3500 g and was on the mixed feeding. Current weight is 4900 g. Evaluate the current weight of the child: Select one: a. Hypotrophy of the I grade b. 150 g less than necessary c. Paratrophy of the I grade d. Corresponding to the age e. Hypotrophy of the II grade
  • 10. Feedback The correct answer is: Corresponding to the age Question 17 Not answered Mark 0.00 out of 1.00 Flag question Question text A 2 year old girl has been ill for 3 days. Today she has low grade fever, severe catarrhal presentations, slight maculopapular rash on her buttocks and enlarged occipital lymph nodes. What is your diagnosis? Select one: a. Scarlet fever b. Rubella c. Adenoviral infection d. Measles e. Pseudotuberculosis Feedback The correct answer is: Rubella Question 18 Not answered Mark 0.00 out of 1.00 Flag question Question text A 20 year old patient was delivered to the hospital in summer from the street with haemorrage from the brachial artery. First medical aid involved aplication of a tourniquet for provisional arrest of bleeding. What is the maximal exposure of the tourniquet? Select one: a. 60 minutes b. 15 minutes
  • 11. c. 120 minutes d. 180 minutes e. 30 minutes Feedback The correct answer is: 120 minutes Question 19 Not answered Mark 0.00 out of 1.00 Flag question Question text A 22 year old female patient complains about frequent and painful urination, urge to urinate at night, enuresis, pain in the suprapubic and lumbar area. Her urine often has beer colouring. She got married a month ago. Objectively: general state is satisfactory. Lung examination revealed vesicular respiration. Heart sounds are rhythmic, heart rate is 78/min, AP- 128/68 mm Hg. Abdomen is soft, painful in the suprapubic area. Urine contains 12-18 erythrocytes and 12-15 bacteria within eyeshot. What is the most probable diagnosis? Select one: a. Infection of inferior urinary tracts - cystitis b. Infection of superior urinary tracts - pyelonephritis c. Urolithiasis d. Gonorrhoea e. Primary syphilis Feedback The correct answer is: Infection of inferior urinary tracts - cystitis Question 20 Not answered Mark 0.00 out of 1.00 Flag question
  • 12. Question text A 22 year old woman complained of right subcostal aching pain, nausea, and decreased appetite. She fell ill 2 months after appendectomy when jaundice appeared. She was treated in an infectious hospital. 1 year later above mentioned symptoms developed. On exam: the subicteric sclerae, enlarged firm liver. Your preliminary diagnosis: Select one: a. Chronic viral hepatitis b. Acute viral hepatitis c. Gilbert's disease d. Chronic cholangitis e. Calculous cholecystitis Feedback The correct answer is: Chronic viral hepatitis Question 21 Not answered Mark 0.00 out of 1.00 Flag question Question text A 23 year old female patient complains about periodical chill and body temperature rise up to 40oC, sense of heat taking turns with profuse sweating. The patient has had already 3 attacks that came once in two days and lasted 12 hours. She has lived in Africa for the last 2 months. Liver and spleen are enlarged. In blood: erythrocytes - 2,5*1012/l. What is the most probable diagnosis? Select one: a. Spotted fever b. Sepsis c. Malaria d. Leptospirosis e. Haemolytic anaemia Feedback The correct answer is: Malaria
  • 13. Question 22 Not answered Mark 0.00 out of 1.00 Flag question Question text A 23 year old patient fell ill 3 weeks ago when she noticed a very painful induration in her axillary crease. 4-5 days later it burst and discharged a lot of pus. After that some new infiltrations appeared around the affected area. The patient has never suffered from skin diseases before. What is the most probable diagnosis? Select one: a. Furuncle b. Mycosis c. Streptococcal impetigo d. Herpes zoster e. Hydradenitis Feedback The correct answer is: Hydradenitis Question 23 Not answered Mark 0.00 out of 1.00 Flag question Question text A 25 year old patient complains about weakness, dizziness, haemorrhagic skin rash. She has been suffering from this for a month. Blood count: erythrocytes: 1,0*1012/l, Hb- 37 g/l, colour index - 0,9, leukocytes - 1,2*109/l, thrombocytes - 42*109/l. What diagnostic method will be the most effective? Select one: a. Coagulogram b. Abdominal ultrasound c. Spleen biopsy
  • 14. d. Liver biopsy e. Sternal puncture Feedback The correct answer is: Sternal puncture Question 24 Not answered Mark 0.00 out of 1.00 Flag question Question text A 25 year old patient complains of pain in the I finger on his right hand. On examination: the finger is homogeneously hydropic, in bent position. On attempt to unbend the finger the pain is getting worse. Acute pain appears during the probe in ligament projection. What decease is the most likely? Select one: a. Thecal whitlow (ligament panaritium) b. Subcutaneous panaritium c. Bone panaritium d. Articular (joint) panaritium e. Paronychia Feedback The correct answer is: Thecal whitlow (ligament panaritium) Question 25 Not answered Mark 0.00 out of 1.00 Flag question Question text A 25 year old patient had pharyngitis 2 weeks ago. Now he complains about body temperature rise up to 38oC, general weakness, dyspnea during walking, swelling and shifting pain in the
  • 15. articulations. Objectively: cyanosis of lips, rhythmic pulse of poor volume - 100 bpm. Left cardiac border deviates outwards from the mediaclavicular line by 1 cm. The first heart sound is weakened on the apex, auscultation revealed systolic souffle. What is the most probable aetiological factor that caused this pathological process? Select one: a. Virus b. Pneumococcus c. Staphylococcus d. Fungi e. Beta-haemolytic streptococcus Feedback The correct answer is: Beta-haemolytic streptococcus Question 26 Not answered Mark 0.00 out of 1.00 Flag question Question text A 25 year old woman applied to a maternity welfare clinic and complained about being unable to conceive within 3 years of regular sexual life. Examination revealed weight gain, male pattern of hair distribution on the pubis, excessive pilosis of thighs. Ovaries were dense and enlarged, basal temperature was monophase. What is the most probable diagnosis? Select one: a. Adrenogenital syndrome b. Sclerocystosis of ovaries c. Gonadal dysgenesis d. Tubo-ovaritis e. Premenstrual syndrome Feedback The correct answer is: Sclerocystosis of ovaries Question 27 Not answered
  • 16. Mark 0.00 out of 1.00 Flag question Question text A 25 year old woman complained of edema on her face and legs, rise of blood pressure up to 160/100 mm Hg and weakness. She fell ill 3 weeks after recovering from angina. Urinalysis data: protein of 0,5 g/l, erythrocytes of 17-20/field, leukocytes of 2-3/field, erythrocyte casts. What treatment should be initiated after specifying the diagnosis? Select one: a. Dipyridamole b. Penicillin OS c. Ceftriaxone d. Heparin e. Ciprofloxacine Feedback The correct answer is: Penicillin OS Question 28 Not answered Mark 0.00 out of 1.00 Flag question Question text A 26 year old manual worker complained of 3 weeks history of fevers and fatigue, weight loss with no other symptoms. Physical findings: Temperature 37,6oC, Ps- 88 bpm, blood pressure 115/70 mm Hg, superficial lymph nodes (occipital, submental, cervical, axillary) are enlarged, neither tender nor painful. Rubella-like rash on the trunk and extremities. Herpes simplex lesions on the lips. Candidosis of oral cavity. What infectious disease would you suspect? Select one: a. Rubella b. Tuberculosis c. Influenza
  • 17. d. Infectious mononucleosis e. HIV infection Feedback The correct answer is: HIV infection Question 29 Not answered Mark 0.00 out of 1.00 Flag question Question text A 26 year old woman complains about edemata, swelling and painfulness of mammary glands, headache, tearfulness, irritability. These signs turn up 5 days before menstruation and disappear after its start. What clinical syndrome is it? Select one: a. Stein-Leventhal syndrome b. Adrenogenital syndrome c. Postcastration syndrome d. Premenstrual syndrome e. Climacteric syndrome Feedback The correct answer is: Premenstrual syndrome Question 30 Not answered Mark 0.00 out of 1.00 Flag question Question text A 26 year old woman had the second labour within the last 2 years with oxytocin application. The child's weight is 4080 g. After the placent birth there were massive bleeding, signs of hemorrhagic
  • 18. shock. Despite the injection of contractive agents, good contraction of the uterus and absence of any cervical and vaginal disorders, the bleeding proceeds. Choose the most probable cause of bleeding: Select one: a. Injury of cervix of the uterus b. Hysterorrhexis c. Delay of the part of placenta d. Hypotonia of the uterus e. Atony of the uterus Feedback The correct answer is: Atony of the uterus Question 31 Not answered Mark 0.00 out of 1.00 Flag question Question text A 26 year old woman who delivered a child 7 months ago has been suffering from nausea, morning vomiting, sleepiness for the last 2 weeks. She suckles the child, menstruation is absent. She hasn't applied any contraceptives. What method should be applied in order to specify her diagnosis? Select one: a. Bimanual vaginal examination b. Palpation of mammary glands and pressing-out of colostrum c. Ultrasonic examination d. Roentgenography of small pelvis organs e. Speculum examination Feedback The correct answer is: Ultrasonic examination Question 32 Not answered Mark 0.00 out of 1.00
  • 19. Flag question Question text A 27 year old patient suffers from haemophilia. He was admitted to the hospital with melena and skin pallor. Objectively: Ps- 110 bpm, AP- 100/60 mm Hg. In blood: Hb- 80 g/l, erythrocytes - 2,8*1012/l. What medication should be administered in the first place? Select one: a. Dicinone b. Stored blood c. Cryoprecipitate d. Packed red blood cells e. Epsilon-aminocapronic acid Feedback The correct answer is: Cryoprecipitate Question 33 Not answered Mark 0.00 out of 1.00 Flag question Question text A 28 year old parturient complains about headache, vision impairment, psychic inhibition. Objectively: AP- 200/110 mm Hg, evident edemata of legs and anterior abdominal wall. Fetus head is in the area of small pelvis. Fetal heartbeats is clear, rhythmic, 190/min. Internal investigation revealed complete cervical dilatation, fetus head was in the area of small pelvis. What tactics of labor management should be chosen? Select one: a. Forceps operation b. Stimulation of labor activity c. Conservative labor management with episiotomy d. Embryotomy
  • 20. e. Cesarean Feedback The correct answer is: Forceps operation Question 34 Not answered Mark 0.00 out of 1.00 Flag question Question text A 28 year old patient was admitted to the clinic with complaints of the temperature rise up to 39,0oC, headache, weakness, constipation on the 9th day of the disease. On examination: single roseolas on the skin of the abdomen are present. The pulse rate is 78 bpm. The liver is enlarged by 2 cm. What is the most probable diagnosis? Select one: a. Brucellosis b. Sepsis c. Malaria d. Typhoid fever e. Leptospirosis Feedback The correct answer is: Typhoid fever Question 35 Not answered Mark 0.00 out of 1.00 Flag question Question text A 28 year old woman has bursting pain in the lower abdomen during menstruation; chocolate-like discharges from vagina. It is known from the anamnesis that the patient suffers from chronic
  • 21. adnexitis. Bimanual examination revealed a tumour-like formation of heterogenous consistency 7х7 cm large to the left from the uterus. The formation is restrictedly movable, painful when moved. What is the most probable diagnosis? Select one: a. Tumour of sigmoid colon b. Fibromatous node c. Follicular cyst of the left ovary d. Endometrioid cyst of the left ovary e. Exacerbation of chronic adnexitis Feedback The correct answer is: Endometrioid cyst of the left ovary Question 36 Not answered Mark 0.00 out of 1.00 Flag question Question text A 29 year old female patient complains about periodical right-sided headache that is usually provoked by strong smells or excitement. The patient's mother suffers from the same disease. Objectively: examination of internal organs revealed no pathology. During the attack there are general hyperesthesia and nausea, at the end of the attack polyuria is observed. Palpation of the right temporal artery during the attack revealed tension and painfulness of the artery. Complete blood count and urine analysis reveale dno pecularities. What is the most probable diagnosis? Select one: a. Neuritis of the facial nerve b. Epilepsy c. Migraine d. Neuralgia of trigeminus e. Meniere's syndrome Feedback The correct answer is: Migraine
  • 22. Question 37 Not answered Mark 0.00 out of 1.00 Flag question Question text A 3 month old infant suffering from acute segmental pneumonia has dyspnea (respiration rate - 80 per minute), paradoxical breathing, tachycardia, total cyanosis. Respiration and pulse - ratio is 1:2. The heart dullness under normal size. Such signs characterise: Select one: a. Respiratory failure of I degree b. Respiratory failure of II degree c. Respiratory failure of III degree d. Congenital heart malformation e. Myocarditis Feedback The correct answer is: Respiratory failure of III degree Question 38 Not answered Mark 0.00 out of 1.00 Flag question Question text A 3 year old boy fell ill abruptly: fever up to 39oC, weakness, vomitng. Haemorrhagic rash of various size appeared on his lower limbs within 5 hours. Meningococcemia with infective - toxic shock of the 1 degree was diagnosed. What medications should be administered? Select one: a. Chloramphenicol succinate and interferon b. Ampicillin and immunoglobulin c. Penicillin and prednisone d. Penicillin and immunoglobulin
  • 23. e. Chloramphenicol succinate and prednisone Feedback The correct answer is: Chloramphenicol succinate and prednisone Question 39 Not answered Mark 0.00 out of 1.00 Flag question Question text A 3 year old boy has petechial eruption. Examination revealed no other pathological changes. Thrombocyte number is 20*109g/l; haemoglobin and leukocyte concentration is normal. What is the most probable diagnosis? Select one: a. Schonlein-Henoch disease b. Acute lymphoblastic leukemia c. Immune thrombocytopenic purpura d. Disseminated intravascular coagulopathy e. Systemic lupus erythematosus Feedback The correct answer is: Immune thrombocytopenic purpura Question 40 Not answered Mark 0.00 out of 1.00 Flag question Question text A 3 year old child has been suffering from fever, cough, coryza, conjunctivitis for 4 days. He has been taking sulfadimethoxine. Today it has fever up to 39oC and maculopapular rash on its face. Except of rash the child's skin has no changes. What is your diagnosis?
  • 24. Select one: a. Scarlet fever b. Allergic rash c. Pseudotuberculosis d. Measles e. Rubella Feedback The correct answer is: Measles Question 41 Not answered Mark 0.00 out of 1.00 Flag question Question text A 3 year old child with weight deficiency suffers from permanent moist cough. In history there are some pneumonias with obstruction. On examination: distended chest, dullness on percussion over the lower parts of lungs. On auscultation: a great number of different rales. Level of sweat chloride is 80 millimol/l. What is the most probable diagnosis? Select one: a. Bronchial asthma b. Mucoviscidosis (cystic fibrosis) c. Bronchiectasis d. Recurrent bronchitis e. Pulmonary hypoplasia Feedback The correct answer is: Mucoviscidosis (cystic fibrosis) Question 42 Not answered Mark 0.00 out of 1.00
  • 25. Flag question Question text A 30 year old man complains of intense pain, reddening of skin, edema in the ankle-joint area, fever up to 39oC. There was an acute onset of the illness. In the past there were similar attacks lasting 5-6 days without residual changes in the joint. The skin over the joint is hyperemic and ill-defined, without infiltrative bank on the periphery. What is the most likely diagnosis? Select one: a. Infectious arthritis b. Erysipelatous inflammation c. Rheumatoid arthritis d. Gout e. Osteoarthritis Feedback The correct answer is: Gout Question 43 Not answered Mark 0.00 out of 1.00 Flag question Question text A 30 year old patient complains about inability to become pregnant over 3 years of married life. The patient is of supernutrition type, she has hair along the median abdominal line, on the internal thigh surface and in the peripapillary area. Menses started at the age of 16, they are infrequent and non- profuse. US revealed that the uterus was of normal size, ovaries were 4х5х5 cm large and had a lot of cystic inclusions. What is the most probable diagnosis? Select one: a. Polycystic ovaries b. Ovarian cystoma c. Bilateral ovarian tumours d. Menstrual irregularity
  • 26. e. Chronic oophoritis Feedback The correct answer is: Polycystic ovaries Question 44 Not answered Mark 0.00 out of 1.00 Flag question Question text A 32 year old patient complains about cardiac irregularities, dizziness, dyspnea at physical stress. He has never suffered from this before. Objectively: Ps- 74 bpm, rhythmic. AP- 130/80 mm Hg. Auscultation revealed systolic murmur above aorta, the first heart sound was normal. ECG showed hypertrophy of the left ventricle, signs of repolarization disturbance in the I, V5 and V6 leads. Echocardiogram revealed that interventricular septum was 2 cm. What is the most probable diagnosis? Select one: a. Hypertrophic cardiomyopathy b. Aortic stenosis c. Essential hypertension d. Coarctation of aorta e. Myocardium infarction Feedback The correct answer is: Hypertrophic cardiomyopathy Question 45 Not answered Mark 0.00 out of 1.00 Flag question Question text
  • 27. A 32 year old patient suffering from chronic viral hepatitis complains about dull pain in the right subcostal area, nausea, dry mouth. Objectively: liver dimensions are 13-21-11 cm (according to Kurlov), spleen is by 2 cm enlarged, aspartate aminotransferase is 3,2 micromole/l*h, alanine aminotransferase - 4,8 millimole/l*h. Serological study revealed HBeAg, high concentration of DNA HBV. What drug should be chosen for treatment of this patient? Select one: a. Acyclovir b. Arabinoside monophosphate c. Essentiale-forte d. a-interferon e. Remantadinum Feedback The correct answer is: a-interferon Question 46 Not answered Mark 0.00 out of 1.00 Flag question Question text A 32 year old welder complains of weakness and fever. His illness started as tonsillitis a month before. On exam, BT of 38,9oC, RR of 24/min, HR of 100/min, BP of 100/70 mm Hg, hemorrhages on the legs, enlargement of the lymph nodes. CBC shows Hb of 70 g/l, RBC of 2,2*1012/l, WBC of 3,0*109/l with 32% of blasts, 1% of eosinophiles, 3% of bands, 36% of segments, 20% of lymphocytes, and 8% of monocytes, ESR of 47 mm/h. What is the cause of anemia? Select one: a. Aplastic anema b. Chronic lympholeukemia c. Vitamin B12 deficiency anemia d. Chronic hemolytic anemia e. Acute leukemia Feedback The correct answer is: Acute leukemia
  • 28. Question 47 Not answered Mark 0.00 out of 1.00 Flag question Question text A 33 year old patient has acute blood loss (erythrocytes - 2,2*1012/l, Hb- 55 g/l), blood group is A(II)Rh+. Accidentally the patient got transfusion of donor packed red blood cells of AB(IV)Rh+ group. An hour later the patient became anxious, got abdominal and lumbar pain. Ps - 134 bpm, AP- 100/65 mm Hg, body temperature - 38,6oC. After catheterization of urinary bladder 12 ml/h of dark- brown urine were obtained. What complication is it? Select one: a. Cardial shock b. Acute renal insufficiency c. Allergic reaction to the donor red blood cells d. Toxic infectious shock e. Citrate intoxication Feedback The correct answer is: Acute renal insufficiency Question 48 Not answered Mark 0.00 out of 1.00 Flag question Question text A 33 year old patient was delivered to the infectious diseases department on the 7-th day of disease. He complained about great weakness, high temperature, pain in the lumbar area and leg muscles, icteritiousness, dark colour of urine, headache. The acute disease started with chill, body temperature rise up to 40oC, headache, pain in the lumbar area and sural muscles. Icterus turned up on the 4th day, nasal and scleral haemorrhages came on the 5th day. Fever has lasted for 6 days. Diuresis - 200 ml. What is the most probable diagnosis? Select one: a. Typhoid fever
  • 29. b. Virus A hepatitis c. Yersiniosis d. Leptospirosis e. Sepsis Feedback The correct answer is: Leptospirosis Question 49 Not answered Mark 0.00 out of 1.00 Flag question Question text A 34 year old female patient has been suffering from anxious depression accompanied by panic attacks for 2 years. She has been a patient of a psychotherapist. Treatment resulted in incomplete remission. The patient had to break off psychotherapy because of moving to a new place of residence. Soon after this her condition grew significantly worse, she was almost permanently anxious, panic attacks turned up 5-6 times a day and were accompanied by palpitation, dyspnea, cold sweat, thanatophobia. What drug group is the most appropriate for medicamental therapy? Select one: a. Cardiotonics, respiratory analeptics b. Sedative neuroleptics c. Lithium drugs d. Antipsychotic neuroleptics e. Antidepressants Feedback The correct answer is: Antidepressants Question 50 Not answered Mark 0.00 out of 1.00
  • 30. Flag question Question text A 34 year old patient complains of profuse sweating at night, skin itching, weight loss (9 kg within the last 3 months). Examination revealed malnutrition, skin pallor. Palpation of neck and inguinal areas revealed dense elastic lymph nodes for about 1 cm in diameter, nonmobile, non-adhering to skin. What is the most probable diagnosis? Select one: a. Lymphogranulomatosis b. Lymphosarcoma c. Chronic lymphadenitis d. Burkitt's lymphoma e. Cancer metastases Feedback The correct answer is: Lymphogranulomatosis Question 51 Not answered Mark 0.00 out of 1.00 Flag question Question text A 34 year old patient was delivered to the hospital because of follicular tonsillitis charactirized by high temperature. The patient has been abusing alcohol for 12 years. In the evening on the day of hospitalization he became anxious, couldn't stay in bed, left his ward several times and applied to the staff on duty with different complaints. He reported about seeing alot of spiders and flies in his ward as well as abou hearing threats from the corridor. He was exasperated by the fact that other patients didn't hear them. He lost also spatial orientation. What psychopathological syndrome is it? Select one: a. Asthenic confusion b. Delirious c. Amentive
  • 31. d. Twilight state e. Oneiric Feedback The correct answer is: Delirious Question 52 Not answered Mark 0.00 out of 1.00 Flag question Question text A 35 year old female patient suffering from cholelithiasis has broken her diet, and this caused an acute pain attack in the right subcostal are. The pain eased off on the third day, but the patient got progressing jaundice. What non-invasive diagnostic method should be applied? Select one: a. Duodenal probing b. Endoscopic retrograde cholangiopancreatography c. Infusive cholecystocholangiography d. Survey radiography of abdominal organs e. Test for bilirubin Feedback The correct answer is: Endoscopic retrograde cholangiopancreatography Question 53 Not answered Mark 0.00 out of 1.00 Flag question Question text A 36 year old female patient complains about general weakness, edemata of her face and hands, rapid fatigability during walking, difficult diglutition, cardiac irregularities. These symptoms turned up
  • 32. 11 days after holiday at the seaside. Objectively: face erythema, edema of shin muscles. Heart sounds are muffled, AP is 100/70 mm Hg. In blood: ASAT activity is 0,95 millimole/h*l, ALAT - 1,3 micromole/h*l, aldolase - 9,2 IU/l, creatine phosphokinase - 2,5 micromole Р/g*l. What method of study would be the most specific? Select one: a. Echocardiogram b. ECG c. Muscle biopsy d. Determination of cortisol concentration in blood and urine e. Electromyography Feedback The correct answer is: Muscle biopsy Question 54 Not answered Mark 0.00 out of 1.00 Flag question Question text A 36 year old man was delivered to the surgical department an hour after a road accident. His condition is getting worse: respiratory insufficiency is progressing, there are cardiac abnormalities. Clinical and roentgenological investigations revealed mediastinal displacement. What process has caused this complication? Select one: a. Valvular pneumothorax b. Closed pneumothorax c. Subcutaneous emphysema d. Open pneumothorax e. Mediastinitis Feedback The correct answer is: Valvular pneumothorax Question 55 Not answered
  • 33. Mark 0.00 out of 1.00 Flag question Question text A 36 year old patient was diagnosed with right-sided pneumothorax. What method of treatment is indicated to the patient? Select one: a. Symptomatic therapy b. Pleural puncture c. Surgical treatment: drainage of the pleural cavity d. Thoracotomy e. Antiinflammation therapy Feedback The correct answer is: Surgical treatment: drainage of the pleural cavity Question 56 Not answered Mark 0.00 out of 1.00 Flag question Question text A 37 year old male patient was admitted to the resuscitation department because of attacks of tonoclonic spasms repeating every half an hour. Between the attacks the patient remains unconscious. AP is 120/90 mm Hg, Ps- 100 bpm. A day before the patient was at wedding and consumed alcohol. 5 years ago he had a closed craniocerebral trauma and brain contusion that later caused single convulsive attacks accompanied by loss of consciousness, but the patient didn't undergo antiepileptic treatment. What drug should be injected for emergency aid? Select one: a. Sodium oxybutyrate b. Magnesium sulfate c. Diazepam d. Sodium thiopental
  • 34. e. Aminazine Feedback The correct answer is: Diazepam Question 57 Not answered Mark 0.00 out of 1.00 Flag question Question text A 37 year old miner has lifted significant loads and afterwards felt pain in the lumbar area irradiating to his left leg. He walks slowly and carefully. Lumbar lordosis is flattened. There is also left-sided scoliosis and tension of paravertebral muscles. Neri's and Dejerine's symptoms are positive, there is Lasegue's sign on the left from the angle of 350. What method will help to specify the diagnosis? Select one: a. Descending myelography b. CT of lumbosacral part of vertebral column c. Electromyography d. Renal sonography e. Lumbal puncture Feedback The correct answer is: CT of lumbosacral part of vertebral column Question 58 Not answered Mark 0.00 out of 1.00 Flag question Question text
  • 35. A 37 year old patient applied to a local therapeutist. As a result of exacerbation of chronic obstructive bronchitis the patient had been temporarily disabled for 117 days within 1 year. What tactics will be legally correct? Select one: a. The therapeutist should issue a new medical certificate b. The patient should be referred to the medicosocial expertise c. The patient should be referred to the medical consultation comission for extension of medical certificate d. The therapeutist should extend a medical certificate e. The patient shoul be referred to the sanatorium-and-spa treatment Feedback The correct answer is: The patient should be referred to the medicosocial expertise Question 59 Not answered Mark 0.00 out of 1.00 Flag question Question text A 38 year old female patient complains about body stiffness in the morning, especially in the articulations of her upper and lower limbs, that disappears 30-60 minutes later after active movements. She has also arthritis of metacarpophalangeal and proximal phalangeal articulations, subfebrile temperature. ESR- 45 mm/h. Roentgenography revealed osteoporosis and erosion of articular surface of small hand and foot articulations. What is the most probable diagnosis? Select one: a. Reactive polyarthritis b. Rheumatoid arthritis c. Psoriatic arthropathy d. Systemic lupus erythematosus e. Osteoarthrosis deformans Feedback The correct answer is: Rheumatoid arthritis
  • 36. Question 60 Not answered Mark 0.00 out of 1.00 Flag question Question text A 38 year old man was delivered to the hospital in unconscious state. The symptoms of illness turned up a day before: headache, nausea, vomiting, to- 38,5oC, dizziness, delusion. For the last 4 days he had been complaining of pain and hearing loss in the left ear. Objectively: sopor, rigidity of occipital muscles, bilateral Kernig's symptom, general hyperesthesia, purulent discharges from the left ear. What is the most probable diagnosis? Select one: a. Parenchymatous subarachnoidal haemorrhage b. Tuberculous meningitis c. Primary purulent meningitis d. Subarachnoidal haemorrhage e. Secondary purulent meningitis Feedback The correct answer is: Secondary purulent meningitis Question 61 Not answered Mark 0.00 out of 1.00 Flag question Question text A 38 year old man worked at roofing and drain pipes production for 15 years. He seeks medical help for expiratory breathlessness on exertion, and dry cough. On exam, wheezes above both lungs, grayish warts on fingers are seen. Factory physician has diagnosed asbestosis. What method is the most important for this diagnosis? Select one: a. Bronchoscopy b. Chest X-ray
  • 37. c. Electrocardiography d. Blood gas analysis e. Spirography Feedback The correct answer is: Chest X-ray Question 62 Not answered Mark 0.00 out of 1.00 Flag question Question text A 38 year old man, previously in good health, suddenly develops severe abdominal pain radiating from the left loin to groin and accompanied by nausea, perspiration and the need for frequent urination. He is restless, tossing in bed but has no abnormal findings. The most likely diagnosis is: Select one: a. Herpes zoster b. Leftsided renal colic c. Sigmoid diverticulitis d. Torsion of the left testicle e. Retroperitoneal haemorrhage Feedback The correct answer is: Leftsided renal colic Question 63 Not answered Mark 0.00 out of 1.00 Flag question Question text
  • 38. A 38 year old patient complains about inertness, subfebrile temperature, enlargement of lymph nodes, nasal haemorrhages, bone pain. Objectively: the patient's skin and mucous membranes are pale, palpation revealed enlarged painless lymph nodes; sternalgia; liver was enlarged by 2 cm, spleen - by 5 cm, painless. In blood: erythrocytes - 2,7*1012/l, Hb- 84 g/l, leukocytes – 58*109/l, eosinophils - 1%, stab neutrophils - 2%, segmented neutrophils - 12%, lymphocytes - 83%, lymphoblasts - 2%, smudge cells; ESR- 57 mm/h. What is the most probable diagnosis? Select one: a. Acute myeloleukemia b. Lymphogranulomatosis c. Chronic myeloleukemia d. Acute lymphatic leukemia e. Chronic lymphatic leukemia Feedback The correct answer is: Chronic lymphatic leukemia Question 64 Not answered Mark 0.00 out of 1.00 Flag question Question text A 39 year old patient complained about morning headache, appetite loss, nausea, morning vomiting, periodic nasal haemorrhages. The patient had acute glomerulonephritis at the age of 15. Examination revealed rise of arterial pressure up to 220/130 mm Hg, skin haemorrhages on his arms and legs, pallor of skin and mucous membranes. What biochemical index has the greatest diagnostic importance in this case? Select one: a. Blood sodium b. Fibrinogen c. Blood creatinine d. Uric acid e. Blood bilirubin Feedback
  • 39. The correct answer is: Blood creatinine Question 65 Not answered Mark 0.00 out of 1.00 Flag question Question text A 39 year old patient suffering from hypertension suddenly gotb intense headache in the region of occiput; there appeared recurrent vomiting. These presentations has been lasting for 5 hours. Objectively: Ps - 88 bpm, AP - 205/100 mm Hg, painfulness of occipital points, rigidity of occipital muscles are present. Kernig's symptom is bilaterally positive. Subarachnoid haemorrhage is also suspected. What diagnostic method will be of the greatest importance for confirmation of provisional diagnosis? Select one: a. Lumbar puncture b. Examination of eye fundus c. Computer tomography d. EEG e. Ultrasonic dopplerography Feedback The correct answer is: Lumbar puncture Question 66 Not answered Mark 0.00 out of 1.00 Flag question Question text A 4 month old child fell seriously ill: body temperature rose up to 38,5oC, the child became inert and had a single vomiting. 10 hours later there appeared rash over the buttocks and lower limbs in form of petechiae, spots and papules. Some haemorrhagic elements have necrosis in the centr E. What is the most probable disease? Select one:
  • 40. a. Meningococcemia b. Scarlet fever c. Haemorrhagic vasculitis d. Influenza e. Rubella Feedback The correct answer is: Meningococcemia Question 67 Not answered Mark 0.00 out of 1.00 Flag question Question text A 4 month old child was admitted to a surgical department 8 hours after the first attack of anxiety. The attacks happen every 10 minutes and last for 2-3 minutes, there was also one-time vomiting. Objectively: the child's condition is grave. Abdomen is soft, palpation reveals a tumour-like formation in the right iliac area. After rectal examination the doctor's finger was stained with blood. What is the most probable diagnosis? Select one: a. Ileocecal invagination b. Pylorostenosis c. Wilm's tumour d. Helminthic invasion e. Gastrointestinal haemorrhage Feedback The correct answer is: Ileocecal invagination Question 68 Not answered Mark 0.00 out of 1.00
  • 41. Flag question Question text A 4 year old girl was playing with her toys and suddenly she got an attack of cough, dyspne A. Objectively: respiration rate - 45/min, heart rate - 130/min. Percussion revealed dullness of percutory sound on the right in the lower parts. Auscultation revealed diminished breath sounds with bronchial resonance on the right. X-ray pictue showed shadowing of the lower part of lungs on the right. Blood analysis revealed no signs of inflammation. The child was diagnosed with foreign body in the right bronchus. What complication caused such clinical presentations? Select one: a. Atelectasis b. Bronchitis c. Pneumonia d. Pneumothorax e. Emphysema Feedback The correct answer is: Atelectasis Question 69 Not answered Mark 0.00 out of 1.00 Flag question Question text A 40 year old female patient has been observing excessive menstruation accompanied by spasmodic pain in the lower abdomen for a year. Bimanual examination performed during menstruation revealed a dense formation up to 5 cm in diameter in the cervical canal. Uterus is enlarged up to 5-6 weeks of pregnancy, movable, painful, of normal consistency. Appendages are not palpabl E. Bloody discharges are profus E. What is the most probable diagnosis? Select one: a. Algodismenorrhea b. Abortion in progress
  • 42. c. Nascent submucous fibromatous node d. Cervical carcinoma e. Cervical myoma Feedback The correct answer is: Nascent submucous fibromatous node Question 70 Not answered Mark 0.00 out of 1.00 Flag question Question text A 40 year old patient was bitten by a stray dog for about an hour ago. The bite can be seen on the patient's left shin in form of a wound 4х2х0,5 cm large. What kind of aid would be recommended in this case? Select one: a. Retension sutures b. Wound lavage with soapsuds, retension sutures c. Aseptic bandage d. Blind suture e. Salve bandage Feedback The correct answer is: Wound lavage with soapsuds, retension sutures Question 71 Not answered Mark 0.00 out of 1.00 Flag question Question text
  • 43. A 40 year old woman has a self-detected hard breast mass. The procedure of choice for confirming the diagnosis is: Select one: a. Thermography b. Excision biopsy c. Mammography d. Ultrasonography e. Aspiration biopsy with cytology Feedback The correct answer is: Excision biopsy Question 72 Not answered Mark 0.00 out of 1.00 Flag question Question text A 41 year old woman has suffered from nonspecific ulcerative colitis for 5 years. On rectoromanoscopy: evident inflammatory process of lower intestinal parts, pseudopolyposive changes of mucous membrane. In blood: WBC- 9,8*109/l, RBC- 3,0*1012/l, ESR - 52 mm/hour. What medication provides pathogenetic treatment of this patient? Select one: a. Motilium b. Vikasolum c. Kreon d. Linex e. Sulfosalasine Feedback The correct answer is: Sulfosalasine Question 73 Not answered Mark 0.00 out of 1.00
  • 44. Flag question Question text A 42 year old man applied to a hospital 10 minutes after he got stung by a bee and complained about face edema and difficult respiration. Objectively: Ps- 98 bpm, AP- 130/80 mm Hg. A doctor on duty injected him 1 ml of 1% dimedrol solution intramuscularly and recommended to apply to his local therapeutist on the next day. What tactics of treatment should be chosen for this patient? Select one: a. Hospitalization for observation b. The patient needs no further medical aid c. Intravenous introduction of calcium chloride d. Intravenous introduction of prednisolone and hospitalization e. Intravenous introduction of calcium chloride and hospitalization Feedback The correct answer is: Intravenous introduction of prednisolone and hospitalization Question 74 Not answered Mark 0.00 out of 1.00 Flag question Question text A 42 year old man works in a boiler room. He complains about girdle headache and recurring vomiting. There was also short-term consciousness loss. Objectively: increase of tendon reflexes, spontaneous myofibrillations. AP is 150/80 mm Hg, Ps- 104 bpm. Visible mucous membranes and cutaneous surfaces have crimson colouring. What is the most probable diagnosis? Select one: a. Poisoning with carbon monooxide b. Poisoning with hydrocyanic acid c. Poisoning with anilin colouring agents d. Poisoning with methane
  • 45. e. Poisoning with benzine Feedback The correct answer is: Poisoning with carbon monooxide Question 75 Not answered Mark 0.00 out of 1.00 Flag question Question text A 42 year old woman complains of dyspnea, edema of the legs and tachycardia during minor physical exertion. Heart borders are displaced to the left and S1 is accentuated, there is diastolic murmur on apex. The liver is enlarged by 5 cm. What is the cause of heart failure? Select one: a. Tricuspid stenosis b. Tricuspid regurgitation c. Mitral stenosis d. Mitral regurgitation e. Aortic stenosis Feedback The correct answer is: Mitral stenosis Question 76 Not answered Mark 0.00 out of 1.00 Flag question Question text A 43 year old female patient was delivered to the hospital in grave condition. She suffers from Addison's disease. The patient had been regularly taking prednisolone but a week before she stopped taking this drug. Objectively: sopor, skin and visible mucous membranes are pigmented,
  • 46. skin and muscle turgor is lowered. Heart sounds are muffled, rapid. AP- 60/40 mm Hg, heart rate - 96/min. In blood: Na- 120 millimole/l, K- 5,8 micromole/l. Development of this complication is primarily caused by the deficit of the following hormone: Select one: a. Cortisol b. Noradrenaline c. Corticotropin (ACTH) d. Adrostendion e. Adrenaline Feedback The correct answer is: Cortisol Question 77 Not answered Mark 0.00 out of 1.00 Flag question Question text A 43 year old patient was admitted to the infectious diseases hospital with high body temperature and intense headache. The iIlness has lasted for 2 days. Examination revealed a carbuncle on his forearm. The area around it was apparently edematic and slightly painful. Regional lymphadenitis and hepatolienal syndrome were also present. It is known from the anamnesis that the patient works at a cattle-breeding farm. What disease should be suspected in the first place? Select one: a. Anthrax b. Eczema c. Erysipeloid d. Skin cancer e. Erysipelas Feedback The correct answer is: Anthrax Question 78 Not answered
  • 47. Mark 0.00 out of 1.00 Flag question Question text A 44 year old man has been working in coke industry for 16 years. Dust concentration at his workplace is 5-10 times more than maximum permissible concentration. Roentgenography of lungs revealed changes that are typical for pneumoconiosis. What is the most probable type of pneumoconiosis in this case? Select one: a. Anthracosis b. Silicatosis c. Anthracosilicosis d. Asbestosis e. Siderosis Feedback The correct answer is: Anthracosis Question 79 Not answered Mark 0.00 out of 1.00 Flag question Question text A 45 year old woman complains about unbearable pain attacks in the left part of face lasting 1-2 minutes. Such attacks are provoked by mastication. These symptoms appeared 2 months ago after exposure to cold. Objectively:the pain is localized in the ending points of trigeminus on the left. A touch near nose wing provokes another pain attack and tonic spasm of face muscles. What is the most probable diagnosis? Select one: a. Facial migraine b. Arthritis of mandibular joint c. Neuralgia of trigeminus
  • 48. d. Neuralgia of glossopharyngeal nerve e. Maxillary sinusitis Feedback The correct answer is: Neuralgia of trigeminus Question 80 Not answered Mark 0.00 out of 1.00 Flag question Question text A 46 year old patient is to be prepared to the operation on account of stomach cancer. Preoperative preparation involves infusion therapy. It was injected up to 3 l of solutions into his right lunar vein. On the next day he got tensive pain in the region of his right shoulder. Examination of interior brachial surface revealed an oblong area of hyperemia, skin edema and painful cord. What complication is it? Select one: a. Phlegmon of paravenous cellular tissue b. Acute lymphangitis c. Necrosis of paravenous cellular tissue d. Acute thrombophlebitis e. Vein puncture and edema of paravenous cellular tissue Feedback The correct answer is: Acute thrombophlebitis Question 81 Not answered Mark 0.00 out of 1.00 Flag question Question text
  • 49. A 47 year old male patient got a flame burn of trunk and upper extremities and was delivered to the hospital. The patient is in grave condition, confused mental state, with fever. AP- 80/50 mm Hg, Ps- 118 bpm. It was locally stated that the patient got III B degree burns with total area of 20%. What medical actions should be taken? Select one: a. Necrotomy of burn surface, haemotransfusion b. Antibacterial and detoxicating therapy c. Administration of detoxicating blood substitutes d. Injection of narcotic analgetics and powdered blood substitutes e. Primary surgical pocessing Feedback The correct answer is: Injection of narcotic analgetics and powdered blood substitutes Question 82 Not answered Mark 0.00 out of 1.00 Flag question Question text A 48 year old female patient complains about contact haemorrhage. Speculum examination revealed hypertrophy of uterus cervix. It resembles of cauliflower, it is dense and can be easily injured. Bimanual examination revealed that fornices were shortened, uterine body was nonmobile. What is the most probable diagnosis? Select one: a. Cervical pregnancy b. Endometriosis c. Metrofibroma d. Cervical carcinoma e. Cervical papillomatosis Feedback The correct answer is: Cervical carcinoma Question 83 Not answered
  • 50. Mark 0.00 out of 1.00 Flag question Question text A 5 month old boy was born prematurely, he didn't suffer from any disease at the infant age and later on. Examination at an outpatient's hospital revealed paleness of skin, sleepiness. Blood count: Hb - 95 g/l, erythrocytes - 3,5*1012/l, reticulocytes - 90/00, colour index - 0,7, osmotic stability of erythrocytes - 0,44-0,33%, serum iron - 4,9 micromole/l. What is the most probable cause of anemia? Select one: a. Hemogenesis immaturity b. Iron deficit c. Erythrocyte hemolysis d. B12 deficit e. Infectious process Feedback The correct answer is: Iron deficit Question 84 Not answered Mark 0.00 out of 1.00 Flag question Question text A 50 year old locksmith was diagnosed with typhoid fever. The patient lives in a separate apartment with all facilities. Apart of him there are also 2 adults in his family. What actions should be taken about persons communicating with the patient? Select one: a. Antibiotic prophylaxis b. Vaccination c. Isolation d. Bacteriological study
  • 51. e. Dispensary observation Feedback The correct answer is: Bacteriological study Question 85 Not answered Mark 0.00 out of 1.00 Flag question Question text A 52 year old male patient complains about attacks of asphyxia, pain in his left side during respiration. These manifestations turned up all of a sudden. It is known from his anamnesis that he had been treated for thrombophlebitis of the right leg for the last month. In the admission ward the patient suddenly lost consciousness, there was a sudden attack of asphyxia and pain in his left side. Objectively: heart rate - 102/min, respiratory rate - 28/min, AP- 90/70 mm Hg. Auscultation revealed diastolic shock above the pulmonary artery, gallop rhythm, small bubbling rales above the lungs under the scapula on the right, pleural friction rub. What examination method will be the most informative for a diagnosis? Select one: a. Coagulogram b. Study of external respiration function c. Angiography of pulmonary vessels d. ECG e. Echocardioscopy Feedback The correct answer is: Angiography of pulmonary vessels Question 86 Not answered Mark 0.00 out of 1.00 Flag question
  • 52. Question text A 52 year old patient complains about headache, weakness of his upper left extremity. Neurological symptoms become more intense during physical stress of the left extremity. Pulsation on the arteries of the left extremity is sharply dampened but it remains unchanged on the carotid arteries. What is the most probable diagnosis? Select one: a. Raynaud's syndrome b. Occlusion of brachiocephalic trunk c. Thoracal outlet syndrome d. Takayasu's disease e. Occlusion of the left subclavicular artery, steal syndrome Feedback The correct answer is: Occlusion of the left subclavicular artery, steal syndrome Question 87 Not answered Mark 0.00 out of 1.00 Flag question Question text A 52 year old patient has hypervolaemic type of essential hypertension. Which of the following medications is to be prescribed either as monotherapy or in complex with other antihypertensive drugs? Select one: a. Hypothiazid b. Kapoten c. Dibazol d. Nifedipin e. Clonidine Feedback The correct answer is: Hypothiazid
  • 53. Question 88 Not answered Mark 0.00 out of 1.00 Flag question Question text A 52 year old patient was admitted to a hospital because of high hemorrhagic diathesis of mucous membranes, massive skin haemorrhages in form of ecchymoses and spots, nasal and stomachal haemorrhages. After clinical examinations her illness was diagnosed as thrombocytopenic purpura. What is the most probable cause of this disease? Select one: a. Generation of antithrombocytic antibodies b. Inherited insufficiency of plasm factors of blood coagulation c. Disturbed hemostasis d. Deficit of the VIII factor of blood coagulation e. Iron deficit in blood serum, bone marrow and depot Feedback The correct answer is: Generation of antithrombocytic antibodies Question 89 Not answered Mark 0.00 out of 1.00 Flag question Question text A 52 year old patient with history of functional Class II angina complains of having intense and prolonged retrosternal pains, decreased exercise tolerance for 5 days. Angina is less responsive to nitroglycerine. What is the most probable diagnosis? Select one: a. IHd. Functional Class II angina b. Myocarditis c. IHd. Unstable angina
  • 54. d. Myocardial dystrophy e. Cardialgia due to spine problem Feedback The correct answer is: IHd. Unstable angina Question 90 Not answered Mark 0.00 out of 1.00 Flag question Question text A 52 year old woman complains about face distortion. It turned up 2 days ago after supercooling. Objectively: body temperature is 38,2oC. Face asymmetry is present. Frontal folds are flattened. Left eye is wider than right one and doesn't close. Left nasolabial fold is flattened, mouth corner is lowered. Examination revealed no other pathology. Blood count: leukocytes - 10*109/l, ESR - 20 mm/h. What is the most probable diagnosis? Select one: a. Ischemic stroke b. Facial neuritis c. Brain tumour d. Trigeminus neuralgia e. Hemicrania (migraine) Feedback The correct answer is: Facial neuritis Question 91 Not answered Mark 0.00 out of 1.00 Flag question Question text
  • 55. A 54 year old female patient was admitted to the hospital with evident acrocyanosis, swollen cervical veins, enlarged liver, ascites. Cardiac borders are dilated. Heart sounds cannot be auscultated, apical beat is undetectable. AP is 100/50 mm Hg. X-ray picture of chest shows enlarged heart shadow in form of a trapezium. What pathology might have caused these symptoms? Select one: a. Complex heart defect b. Exudative pleuritis c. Hiatal hernia d. Cardiac tamponade e. Acute cardiac insufficiency Feedback The correct answer is: Cardiac tamponade Question 92 Not answered Mark 0.00 out of 1.00 Flag question Question text A 54 year old male patient complains about permanent dull pain in the mesogastral region, weight loss, dark blood admixtures in the feces, constipations. He put off 10 kg within a year. In blood: erythrocytes: 3,5*1012/l, Hb- 87 g/l, leukocytes - 12,6*109/l, stab neutrophil shift, ESR- 43 mm/h. What is the most probable diagnosis? Select one: a. Gastric ulcer b. Chronic pancreatitis c. Stomach cancer d. Chronic colitis e. Cancer of transverse colon Feedback The correct answer is: Cancer of transverse colon Question 93 Not answered
  • 56. Mark 0.00 out of 1.00 Flag question Question text A 58 year old female patient complains about periodical headache, dizziness and ear noise. She has been suffering from diabetes mellitus for 15 years. Objectively: heart sounds are rhythmic, heart rate is 76/min, there is diastolic shock above aorta, AP is 180/110 mm Hg. In urine: OD- 1,014. Daily loss of protein with urine is 1,5 g. What drug should be chosen for treatment of arterial hypertension? Select one: a. Calcium channel antagonist b. b-blocker c. Ihibitor of angiotensin converting enzyme d. a-blocker e. Thiazide diuretic Feedback The correct answer is: Ihibitor of angiotensin converting enzyme Question 94 Not answered Mark 0.00 out of 1.00 Flag question Question text A 59 year old female patient applied to a maternity welfare clinic and complained about bloody discharges from the genital tracts. Postmenopause is 12 years. Vaginal examination revealed that external genital organs had signs of age involution, uterus cervix was not erosive, small amount of bloody discharges came from the cervical canal. Uterus was of normal size, uterine appendages were unpalpable. Fornices were deep and painless. What method should be applied for the diagnosis specification? Select one: a. Separated diagnosic curretage b. Laparoscopy c. Culdoscopy
  • 57. d. Extensive colposcopy e. Puncture of abdominal cavity through posterior vaginal fornix Feedback The correct answer is: Separated diagnosic curretage Question 95 Not answered Mark 0.00 out of 1.00 Flag question Question text A 60 year old man with unstable angina pectoris fails to respond to heparin, nitroglycerin, beta adrenegic blockers and calcium channel antagonist. The best management includes: Select one: a. Antihypertensive therapy b. Intravenous streptokinase c. Coronary artery bypass grafting d. Excercise testing e. Oral aspirin Feedback The correct answer is: Coronary artery bypass grafting Question 96 Not answered Mark 0.00 out of 1.00 Flag question Question text A 62 year old patient complains of rest dyspnea, heart pains. 3 years ago he had myocardial infarction. Physical examination: orthopnea, acrocyanosis, swollen cervical veins. Ps - 92, total heart
  • 58. enlargement, the liver is enlarged by 7 cm, shin edema. What is the stage of chronic heart failure (CHF)? Select one: a. CHF- 1 b. CHF- 3 c. CHF- 0 d. CHF- 2 А e. CHF- 2 B Feedback The correct answer is: CHF- 2 B Question 97 Not answered Mark 0.00 out of 1.00 Flag question Question text A 63 year old patient complained about pain in the lumbar area. He underwent a course of physiological treatment on account of radiculitis but this led to no improvement of his condition. R- graphy of spinal column and pelvic bones revealed osteoporosis and serious bone defects. Blood analysis revealed moderate normochromic anaemia, urine analysis revealed proteinuria. Whole blood protein made up 10,7 g/l. What disease should be suspected? Select one: a. Systemic osteoporosis b. Urolithiasis c. Metastases in bones d. Acute radiculitis e. Myelomatosis Feedback The correct answer is: Myelomatosis Question 98 Not answered Mark 0.00 out of 1.00
  • 59. Flag question Question text A 63 year old patient was diagnosed with purulent mediastinitis. What of the below listed diseases are NOT the cause of purulent mediastinitis? Select one: a. Deep neck phlegmon b. Perforation of the thoracic part of the oesophagus c. Iatrogenic injury of the trachea d. Perforation of the cervical part of the oesophagus e. Cervical lymphadenitis Feedback The correct answer is: Cervical lymphadenitis Question 99 Not answered Mark 0.00 out of 1.00 Flag question Question text A 67 year old female patient complains about edemata of face and legs, pain in the lumbar area that is getting worse at moving; great weakness, sometimes nasal haemorrhages, rise of body temperature up to 38,4oC. Objectively: painfulness of vertebral column and ribs on palpation. Laboratorial study revealed daily proteinuria of 4,2 g, ESR- 52 mm/h. What changes of laboratory indices are to be expected? Select one: a. Whole protein of blood serum - 101 g/l b. Haemoglobin - 165 g/l c. g-globulins - 14% d. Leukocytes - 15,3 g/l e. Albumins - 65%
  • 60. Feedback The correct answer is: Whole protein of blood serum - 101 g/l Question 100 Not answered Mark 0.00 out of 1.00 Flag question Question text A 68 year old female patient complains about temperature rise up to 38,3oC, haematuria. ESR- 55 mm/h. Antibacterial therapy turned out to be ineffective. What diagnosis might be suspected? Select one: a. Urolithiasis b. Renal cancer c. Renal amyloidosis d. Chronic glomerulonephritis e. Polycystic renal disease Feedback The correct answer is: Renal cancer Question 101 Not answered Mark 0.00 out of 1.00 Flag question Question text A 68 year old patient complains about acute pain in his right foot, toe edema and darkening of skin of the IV toe. He has been suffering from diabetes mellitus for 15 years, doesn't receive regular treatment. What complication of diabetes mellitus is it? Select one: a. Haematoma
  • 61. b. Gangrene of the IV toe on the right foot c. Panaritium d. Erysipelas e. Fracture of the IV toe on the right foot Feedback The correct answer is: Gangrene of the IV toe on the right foot Question 102 Not answered Mark 0.00 out of 1.00 Flag question Question text A 70 year old man is suffering from coronary heart disease. His mood is evidently depressed, anxious. As a result of continious sleeplessness he has got fears, suicidal thoughts. He would sit for a long time in the same pose, answer after a pause, in a low, monotonous voice. His face has a look of suffering, pain, fear. What is the main psychopathologic syndrome? Select one: a. Phobic syndrome b. Depressive syndrome c. Obsessive syndrome d. Paranoid syndrome e. Asthenic syndrome Feedback The correct answer is: Depressive syndrome Question 103 Not answered Mark 0.00 out of 1.00 Flag question
  • 62. Question text A 72 year old female patient has been treated for urolithiasis in the urological department. After atropine injection she got acute pain in her left eye and abrupt vision impairment. Objectively: visual acuity of the left eye is 0,01, the eye is dense but painful on palpation, cornea is opaque, there is cyanotic induration of eyeball vessels. What is the most probable diagnosis? Select one: a. Degeneration of the left eye cornea b. Acute keratitis of the left eye c. Acute iridocyclitis of the left eye d. Acute attack of primary glaucoma of the left eye e. Secondary glaucoma of the left eye Feedback The correct answer is: Acute attack of primary glaucoma of the left eye Question 104 Not answered Mark 0.00 out of 1.00 Flag question Question text A 72 year old male patient complains about itch in his left shin, especially around a trophic ulcer. Skin is reddened and edematic, there are some oozing lesions, single yellowish crusts. The focus of affection is well-defined. What is the most probable diagnosis? Select one: a. Seborrheic eczema b. Streptococcal impetigo c. Allergic dermatitis d. Microbial eczema e. Cutaneous tuberculosis Feedback The correct answer is: Microbial eczema
  • 63. Question 105 Not answered Mark 0.00 out of 1.00 Flag question Question text A 72 year old patient complains about pain and bleeding during defecation. Digital rectal investigation revealed a tumour of anal canal. After verification of the diagnosis the patient was diagnosed with squamous cell carcinoma. The secondary (metastatic) tumour will be most probably found in: Select one: a. Lungs b. Liver c. Mediastinum d. Brain e. Pelvic bones Feedback The correct answer is: Lungs Question 106 Not answered Mark 0.00 out of 1.00 Flag question Question text A 75 year old man who has been suffering from diabetes for the last six months was found to be jaundiced. He was asymptomatic except for weight loss at the rate of 10 pounds in 6 months. Physical examination revealed a hard, globular, right upper quadrant mass that moves during respiration. A CT scan shows enlargement of the head of the pancreas, with no filling defects in the liver. The most likely diagnosis is: Select one: a. Carcinoma of the head of the pancreas b. Malignant biliary stricture
  • 64. c. Haemolytic jaundice d. Metastatic disease of liver e. Infectious hepatitis Feedback The correct answer is: Carcinoma of the head of the pancreas Question 107 Not answered Mark 0.00 out of 1.00 Flag question Question text A 9 year old boy had acute respiratory viral infection. After it there appeared polydipsia, polyuria, weakness, nausea. Examination revealed the following symptoms: mental confusion, dry skin, soft eyeballs, Kussmaul's respiration, acetone smell from the mouth, muffled heart sounds, soft and painless abdomen. Blood sugar was 19 millimole/l. What acute condition is it? Select one: a. Cerebral coma b. Ketoacidotic coma c. Hepatic coma d. Hyperosmolar coma e. Acute renal insufficiency Feedback The correct answer is: Ketoacidotic coma Question 108 Not answered Mark 0.00 out of 1.00 Flag question Question text
  • 65. A boy is 1 year old. Previously he had purulent otitis. After that he started complaining about pain in the upper third of his left thigh, body temperature rose up to 39oC. Objectively: the thigh is swollen in its upper third, inguinal fold is smoothed. Extremity is in half-bent position. Active and passive movements are impossible because of acute pain. What is the most probable diagnosis? Select one: a. Acute coxitis b. Intermuscular phlegmon c. Osteosarcoma d. Acute haematogenic osteomyelitis e. Brodie's abscess Feedback The correct answer is: Acute haematogenic osteomyelitis Question 109 Not answered Mark 0.00 out of 1.00 Flag question Question text A child is 1 day old. During delivery there had been problems with extraction of shoulders. Body weight is 4300,0. Right arm hangs down along the body, hand is pronated, movement in the arm is absent. "Scarf" symptom is positive. What is the most probable diagnosis? Select one: a. Hemiparesis b. Distal right-sided obstetric paralysis c. Tetraparesis d. Total right-sided obstetric paralysis e. Proximal right-sided obstetric paralysis Feedback The correct answer is: Total right-sided obstetric paralysis Question 110 Not answered Mark 0.00 out of 1.00
  • 66. Flag question Question text A child was born with body weight 3250 g and body length 52 cm. At the age of 1,5 month the actual weight is sufficient (4350 g), psychophysical development corresponds with the age. The child is breast-fed, occasionally there are regurgitations. What is the cause of regurgitations? Select one: a. Pylorostenosis b. Aerophagia c. Pylorospasm d. Acute gastroenteritis e. Esophageal atresia Feedback The correct answer is: Aerophagia Question 111 Not answered Mark 0.00 out of 1.00 Flag question Question text A department chief of an in-patient hospital is going to examine resident doctors as to observation of medical-technological standards of patient service. What documentation should be checked for this purpose? Select one: a. Statistic cards of discharged patients b. Treatment sheets c. Health cards of in-patients d. Registry of operative interventions e. Annual report of a patient care institution
  • 67. Feedback The correct answer is: Health cards of in-patients Question 112 Not answered Mark 0.00 out of 1.00 Flag question Question text A female patient consulted a doctor about gain in weight, chill, edemata, dry skin, sleepiness, problems with concentration. Objectively: the patient's height is 165 cm, weight is 90 kg, gynoid body proportions, to- 35,8oC, ESR- 58/min, AP- 105/60 mm Hg. Heart sounds are weakened, bradycardia is present. Other internal organs have no changes. Thyroid gland is not palpable. Mammary glands ooze milk droplets. Hormonal study revealed rise of TSH and prolactin concentration, reduction of T4. What factor caused obesity? Select one: a. Prolactinoma b. Primary hypothyroidism c. Hypopituitarism d. Secondary hypothyroidism e. Adiposogenital dystrophy Feedback The correct answer is: Primary hypothyroidism Question 113 Not answered Mark 0.00 out of 1.00 Flag question Question text A female patient has been suffering from pain in the right subcostal area, bitter taste in the mouth, periodical bile vomiting for a month. The patient put off 12 kg. Body temperature in the evening is
  • 68. 37,6oC. Sonography revealed that bile bladder was 5,5х2,7 cm large, its wall - 0,4 cm, choledochus - 0,8 cm in diameter. Anterior liver segment contains a roundish hypoechoic formation up to 5 cm in diameter and another two up to 1,5 cm each, walls of these formations are up to 0,3 cm thick. What is the most probable diagnosis? Select one: a. Alveolar echinococcus of liver b. Liver abscess c. Cystous liver cancer d. Paravesical liver abscesses e. Liver cancer Feedback The correct answer is: Alveolar echinococcus of liver Question 114 Not answered Mark 0.00 out of 1.00 Flag question Question text A fitter of a metallurgic factory with occupational exposure to high concentrations of mercury fumes for 16 years presents instability of pulse and blood pressure, general hyperhydrosis, asymmetric innervations of facial muscles and tongue, positive subcortical reflexes, hand tremor on physical examination. A dentist revealed paradontosis and chronic stomatitis. What is the most probable diagnosis? Select one: a. Parkinson syndrome b. Neuroinfection c. Acute mercury intoxication d. Chronic mercury intoxication e. Mercury encephalopathy Feedback The correct answer is: Chronic mercury intoxication
  • 69. Question 115 Not answered Mark 0.00 out of 1.00 Flag question Question text A healthy 75 year old woman who leads a moderately active way of life went through a preventive examination that revealed serum concentration of common cholesterol at the rate of 5,1 millimole/l and HDL (high-density lipoproteins) cholesterol at the rate of 70 mg/dl. ECG reveals no pathology. What dietary recommendation is the most adequate? Select one: a. Decrease of saturated fats consumption b. Any dietary changes are necessary c. Decrease of carbohydrates consumption d. Increase of cellulose consumption e. Decrease of cholesterol consumption Feedback The correct answer is: Any dietary changes are necessary Question 116 Not answered Mark 0.00 out of 1.00 Flag question Question text A male patient, 60 years old, tobacco smoker for 30 years, alcoholic, has dysphagia and weight loss since 4 months. Suggested diagnosis? Select one: a. Cancer of the esophagus b. Esophageal achalasia c. Hanter's disease d. Esophagitis
  • 70. e. Esophageal diverticulum Feedback The correct answer is: Cancer of the esophagus Question 117 Not answered Mark 0.00 out of 1.00 Flag question Question text A man, aged 37, working on the collective farm on sowing, was admitted to the infectious hospital with the clinical symptoms: miosis, labored breathing, sweating.What kind of poisoning is it and what is the first aid? Select one: a. Poisoning by lead. Treatment: tetacine b. Poisoning by vapours of mercury. Treatment: unithiol c. Poisoning by the methylic alcohol. Treatment: ethylic alcohol d. Calcii Poisoning by POC. Treatment: atropine Feedback The correct answer is: Calcii Poisoning by POC. Treatment: atropine Question 118 Not answered Mark 0.00 out of 1.00 Flag question Question text A maternity hospital registered 616 live births, 1 stillbirth, 1 death on the 5th day of life over a 1 year period. What index allows the most precise estimation of this situation? Select one: a. Neonatal mortality
  • 71. b. Natural increase c. Crude mortality rate d. Natality e. Perinatal mortality Feedback The correct answer is: Perinatal mortality Question 119 Not answered Mark 0.00 out of 1.00 Flag question Question text A nine year old child is at a hospital with acute glomerulonephritis. Clinical and laboratory examinations show acute condition. What nutrients must not be limited during the acute period of glomerulonephritis? Select one: a. Proteins b. Carbohydrates c. Fats d. Salt e. Liquid Feedback The correct answer is: Carbohydrates Question 120 Not answered Mark 0.00 out of 1.00 Flag question
  • 72. Question text A parturient complains about pain in the mammary gland. Palpation revealed a 3х4 cm large infiltration, soft in the centre. Body temperature is 38,5oC. What is the most probable diagnosis? Select one: a. Retention of milk b. Acute purulent mastitis c. Birth trauma d. Pneumonia e. Pleuritis Feedback The correct answer is: Acute purulent mastitis Question 121 Not answered Mark 0.00 out of 1.00 Flag question Question text A parturient woman is 27 year old, it was her second labour, delivery was at term, normal course. On the 3rd day of postpartum period body temperature is 36,8oC, Ps - 72/min, AP - 120/80 mm Hg. Mammary glands are moderately swollen, nipples are clean. Abdomen is soft and painless. Fundus of uterus is 3 fingers below the umbilicus. Lochia are bloody, moderate. What is the most probable diagnosis? Select one: a. Physiological course of postpartum period b. Subinvolution of uterus c. Postpartum metroendometritis d. Remnants of placental tissue after labour e. Lactostasis Feedback The correct answer is: Physiological course of postpartum period
  • 73. Question 122 Not answered Mark 0.00 out of 1.00 Flag question Question text A patient applied to the traumatology cenre and complained about a trauma of the lower third of the volar forearm surface caused by cut on a piece of galss. Objectively: flexion of the IV and V fingers is impaired, sensitivity of the interior dorsal and palmar surface of hand as well as of the IV finger is reduced. What nerve is damaged? Select one: a. Median b. Radial c. Ulnar d. Musculoskeletal e. Axillary Feedback The correct answer is: Ulnar Question 123 Not answered Mark 0.00 out of 1.00 Flag question Question text A patient complained about problems with pain and tactile sensitivity, pain in the nail bones at the end of the working day. He works at a plant with mechanical devices. What pathology can be suspected? Select one: a. Overwork symptoms b. Caisson disease c. Vibration disease
  • 74. d. Hypovitaminosis of B1 e. Noise disease Feedback The correct answer is: Vibration disease Question 124 Not answered Mark 0.00 out of 1.00 Flag question Question text A patient complains about evaginations in the region of anus that appear during defecation and need to be replaced. Examination with anoscope revealed 1x1 cm large evaginations of mucosa above the pectineal line. What is the most probable diagnosis? Select one: a. Anal fissure b. External hemorrhoids c. Acute paraproctitis d. Internal hemorrhoids e. Don’t Guess, Think Feedback The correct answer is: Internal hemorrhoids Question 125 Not answered Mark 0.00 out of 1.00 Flag question Question text A patient complains about strong dyspnea that is getting worse during physical activity. Presentations appeared suddenly 2 hours ago at work: acute chest pain on the left, cough. The pain
  • 75. was abating, but dyspnea, dizziness, pallor, cold sweat and cyanosis were progressing. Vesicular respiration is absent, X-ray picture shows a shadow on the left. What pathology might be suspected? Select one: a. Left-sided pneumonia b. Pulmonary abscess c. Pulmonary infarction d. Pleuritis e. Spontaneous left-sided pneumothorax Feedback The correct answer is: Spontaneous left-sided pneumothorax Question 126 Not answered Mark 0.00 out of 1.00 Flag question Question text A patient consulted a venereologist about painful urination, reddening of the external opening of urethra, profuse purulent discharges from the urethra. He considers himself to be ill for 3 days. He also associates the disease with a casual sexual contact that took place for about a week ago. If provisional diagnosis "acute gonorrheal urethritis" will be confirmed, then bacteriological study of urethral discharges will reveal: Select one: a. Spirochaete b. Gram-negative diplococci c. Gram-positive diplococci d. Proteus vulgaris e. Mycoplasma Feedback The correct answer is: Gram-negative diplococci Question 127 Not answered Mark 0.00 out of 1.00
  • 76. Flag question Question text A patient has been in a hospital. The beginning of the disease was gradual: nausea, vomiting, dark urine, аcholic stools, yellowness of the skin and scleras. The liver is protruded by 3 cm. Jaundice progressed on the 14th day of the disease. The liver diminished in size. What complication of viral hepatitis caused deterioration of the patient's condition? Select one: a. Relapse of viral hepatitis b. Cholangitis c. Infectious-toxic shock d. Hepatic encephlopathy e. Meningitis Feedback The correct answer is: Hepatic encephlopathy Question 128 Not answered Mark 0.00 out of 1.00 Flag question Question text A patient has got acute macrofocal myocardial infarction complicated by cardiogenic shock. The latter is progresing under conditions of weak general peripheric resistance and lowered cardiac output. What antihypotensive drug should be injected to the patient in the first place? Select one: a. Mesatonum b. Noradrenaline c. Adrenaline d. Prednisolone e. Dopamine
  • 77. Feedback The correct answer is: Dopamine Question 129 Not answered Mark 0.00 out of 1.00 Flag question Question text A patient has got pain in the axillary area, rise of temperature developed 10 hours ago. On examination: shaky gait is evident, the tongue is coated with white deposit. The pulse is frequent. The painful lymphatic nodes are revealed in the axillary area. The skin over the lymph nodes is erythematous and glistering. What is the most probable diagnosis? Select one: a. Anthrax b. Tularemia c. Lymphogranulomatosis d. Bubonic plague e. Acute purulent lymphadenitis Feedback The correct answer is: Bubonic plague Question 130 Not answered Mark 0.00 out of 1.00 Flag question Question text A patient has restrained umbilateral hernia complicated by phlegmon hernia, it is necessary to take following actions: Select one:
  • 78. a. Herniotomy by Mayo b. Herniotomy by Lekser c. Herniotomy by Sapezhko d. Herniotomy by Grenov e. Herniotomy by Mayo-Sapezhko Feedback The correct answer is: Herniotomy by Mayo-Sapezhko Question 131 Not answered Mark 0.00 out of 1.00 Flag question Question text A patient suffering from gastroesophageal reflux has taken from time to time a certain drug that "reduces acidity" for 5 years. This drug was recommended by a pharmaceutist. The following side effects are observed: osteoporosis, muscle weakness, indisposition. What drug has such following effects? Select one: a. Gastrozepin b. Aluminium-bearing antacid c. Metoclopramide d. Н2-blocker e. Inhibitor of proton pump Feedback The correct answer is: Aluminium-bearing antacid Question 132 Not answered Mark 0.00 out of 1.00
  • 79. Flag question Question text A patient was delivered to a surgical department after a road accident with a closed trauma of chest and right-sided rib fracture. The patient was diagnosed with right-sided pneumothorax, it is indicated to perform drainage of pleural cavity. Pleural puncture should be made in: Select one: a. In the projection of pleural sinus b. In the 2nd intercostal space along the middle clavicular line c. In the 7th intercostal space along the scapular line d. In the 6th intercostal space along the posterior axillary line e. In the point of the greatest dullness on percussion Feedback The correct answer is: In the 2nd intercostal space along the middle clavicular line Question 133 Not answered Mark 0.00 out of 1.00 Flag question Question text A patient who takes diuretics has developed arrhythmia as a result of cardiac glycoside overdose. What is the treatment tactics in this case? Select one: a. Reduced magnesium concentration in blood b. Increased potassium concentration in blood c. Increased calcium concentration in blood d. Increased sodium consentration in blood Feedback
  • 80. The correct answer is: Increased potassium concentration in blood Question 134 Not answered Mark 0.00 out of 1.00 Flag question Question text A patient with acute purulent otitis media complicated by mastoiditis was admitted to a hospital. Roentgenogram of mastoid processes showed the shadiowing of the cellular system on the lesion, absence of bone septa was present. What are the necessary therapeutic actions at the second stage of mastoiditis? Select one: a. Mastoidotomy b. Cateterization of the Eustachian tube c. Paracentesis of the drum d. Tympanoplasty e. Radical operation on the middle ear Feedback The correct answer is: Mastoidotomy Question 135 Not answered Mark 0.00 out of 1.00 Flag question Question text A pediatrician talked to a mother of a 7 month old breast-fed boy and found out that the child was fed 7 times a day. How many times should the child of such age be fed? Select one: a. 7 times b. 3 times c. 6 times
  • 81. d. 5 times e. 4 times Feedback The correct answer is: 5 times Question 136 Not answered Mark 0.00 out of 1.00 Flag question Question text A plot of land with total area of 2,0 hectare was intended for building of a hospital. The maximal capacity of the hospital will be: Select one: a. 800 beds b. 400 beds c. 100 beds d. Over 1000 beds e. 200 beds Feedback The correct answer is: 100 beds Question 137 Not answered Mark 0.00 out of 1.00 Flag question Question text A pregnant woman in her 40th week of pregnancy undergoes obstetric examination: the cervix of uterus is undeveloped. The oxytocin test is negative. Examination at 32 weeks revealed: AP 140/90 mm Hg, proteinuria 1 g/l, peripheral edemata. Reflexes are normal. Choose the most correct tactics:
  • 82. Select one: a. Caesarian section immediately b. Complex therapy of gestosis for 2 days c. Complex therapy of gestosis for 7 days d. Absolute bed rest for 1 month e. Labour stimulation after preparation Feedback The correct answer is: Labour stimulation after preparation Question 138 Not answered Mark 0.00 out of 1.00 Flag question Question text A prematurely born girl is now 8 months old. She has dyspnea, tachycardia, hepatosplenomegaly, physical developmental lag, limb cyanosis. There is also parasternal cardiac hump, auscultation revealed systolodiastolic murmur in the II intercostal space on the left. AP is 90/0 mm Hg. What disease should be suspected? Select one: a. Stenosis of pulmonary artery b. Nonclosure of interventricular septum c. Stenosis of aortal valve d. Patent ductus arteriosus e. Coarctation of aorta Feedback The correct answer is: Patent ductus arteriosus Question 139 Not answered Mark 0.00 out of 1.00
  • 83. Flag question Question text A sergeant was injured by a shell splinter in the left subcostal area. He was bandaged with a first-aid pack on a battlefield. The patient was delivered to the regiment medical aid station. He complains about dizziness, weakness, thirst, abdominal pain. General condition is grave, the patient is pale. Ps is 120 bpm. Abdomen is soft, painful on palpation. The bandage is well fixed but a little bit soaked with blood. The patient should be evacuated to the medical battalion with the following transport and in the following turn: Select one: a. With medical vehicle in the second turn b. With a passing car in the first turn c. With a passing car in the third turn d. With a passing car in the second turn e. With medical vehicle in the first turn Feedback The correct answer is: With medical vehicle in the first turn Question 140 Not answered Mark 0.00 out of 1.00 Flag question Question text A woman 26 years old has abused alcohol for 7 years. She has psychological dependence on alcohol, but no withdrawal syndrome. Drinks almost every day approximately 50-100 g of wine. She is in her 4-th week of pregnancy. Primary prevention of fetal alcohol syndrome requires: Select one: a. Medical abortion b. of alcoholism and full abstinance from alcohol during all the period of pregnancy c. Participation in the A-ANON group d. Decrease of alcohol use
  • 84. e. Gyneacological observation Feedback The correct answer is: of alcoholism and full abstinance from alcohol during all the period of pregnancy Question 141 Not answered Mark 0.00 out of 1.00 Flag question Question text A woman complains of having slight dark bloody discharges and mild pains in the lower part of abdomen for several days. Last menses were 7 weeks ago. The pregnancy test is positive. Bimanual investigation: the body of the uterus indicates for about 5-6 weeks of pregnancy, it is soft, painless. In the left appendage there is a retort-like formation, 7х5 cm large, mobile, painless. What examination is necessary for detection of fetus localization? Select one: a. Hysteroscopy b. Ultrasound c. Cystoscopy d. Colposcopy e. Hromohydrotubation Feedback The correct answer is: Ultrasound Question 142 Not answered Mark 0.00 out of 1.00 Flag question Question text
  • 85. A woman consulted a doctor on the 14th day after labour about sudden pain, hyperemy and induration of the left mammary gland, body temperature rise up to 39oC, headache, indisposition. Objectively: fissure of nipple, enlargement of the left mammary gland, pain on palpation. What pathology would you think about in this case? Select one: a. Lactational mastitis b. Breast cancer c. Lacteal cyst with suppuration d. Phlegmon of mammary gland e. Fibrous adenoma of the left mammary gland Feedback The correct answer is: Lactational mastitis Question 143 Not answered Mark 0.00 out of 1.00 Flag question Question text A woman consulted a therapeutist about fatigability, significant weight loss, weakness, loss of appetite. She has had amenorrhea for 8 months. A year ago she born a full-term child. Haemorrhage during labour made up 2 l. She got blood and blood substitute transfusions. What is the most probable diagnosis? Select one: a. Stein-Leventhal syndrome b. Shereshevsky-Turner's syndrome c. Homological blood syndrome d. Sheehan's syndrome e. Vegetovascular dystonia Feedback The correct answer is: Sheehan's syndrome Question 144 Not answered
  • 86. Mark 0.00 out of 1.00 Flag question Question text A woman delivered a child. It was her fifth pregnancy but the first delivery. Mother's blood group is A(II)Rh-, newborn's - A(II)Rh+. The level of indirect bilirubin in umbilical blood was 58 micromole/l, haemoglobin - 140 g/l, RBC- 3,8*1012/l. In 2 hours the level of indirect bilirubin turned 82 micromole/l. The hemolytic disease of newborn (icteric-anemic type, Rh-incompatibility) was diagnosed. Choose the therapeutic tactics: Select one: a. Replacement blood transfusion (conservative therapy) b. Antibiotics c. Blood transfusion (conservative therapy) d. Conservative therapy e. Symptomatic therapy Feedback The correct answer is: Replacement blood transfusion (conservative therapy) Question 145 Not answered Mark 0.00 out of 1.00 Flag question Question text A woman is 34 years old, it is her tenth labor at full term. It is known from the anamnesis that the labor started 11 hours ago, labor was active, painful contractions started after discharge of waters and became continuous. Suddenly the parturient got knife-like pain in the lower abdomen and labor activity stopped. Examination revealed positive symptoms of peritoneum irritation, ill-defined uterus outlines. Fetus was easily palpable, movable. Fetal heartbeats wasn't auscultable. What is the most probable diagnosis? Select one: a. Rupture of uterus b. Risk of uterus rupture
  • 87. c. Discoordinated labor activity d. II labor period e. Uterine inertia Feedback The correct answer is: Rupture of uterus Question 146 Not answered Mark 0.00 out of 1.00 Flag question Question text A woman of a high-risk group (chronic pyelonephritis in anamnesis) had vaginal delivery. The day after labour she complained of fever and loin pains, frequent urodynia. Specify the most probable complication: Select one: a. Apostasis of sutures after episiotomy b. Endometritis c. Infectious hematoma d. Thrombophlebitis of veins of the pelvis e. Infectious contamination of the urinary system Feedback The correct answer is: Infectious contamination of the urinary system Question 147 Not answered Mark 0.00 out of 1.00 Flag question Question text
  • 88. Administration of a plant producing red lead paint intends to form a group of medical specialists for periodical medical examinations. What specialist must be obligatory included into this group? Select one: a. Neuropathologist b. Gynaecologist c. Otolaryngologyst d. Psychiatrist e. Dermatologist Feedback The correct answer is: Neuropathologist Question 148 Not answered Mark 0.00 out of 1.00 Flag question Question text After objective clinical examination a 12 year old child was diagnosed with mitral valve prolapse. What complementary instrumental method of examination should be applied for the diagnosis confirmation? Select one: a. Phonocardiography b. ECG c. Veloergometry d. Roentgenography of chest e. Echocardiography Feedback The correct answer is: Echocardiography Question 149 Not answered Mark 0.00 out of 1.00
  • 89. Flag question Question text An 18 year old patient was admitted to a hospital with complaints of headache, weakness, high temperature, sore throat. Objectively: enlargement of all groups of lymph nodes was revealed. The liver is enlarged by 3 cm, spleen - by 1 cm. In blood: leukocytosis, atypical lymphocytes - 15%. What is the most probable diagnosis? Select one: a. Adenoviral infection b. Diphtheria c. Acute lymphoid leukosis d. Angina e. Infectious mononucleosis Feedback The correct answer is: Infectious mononucleosis Question 150 Not answered Mark 0.00 out of 1.00 Flag question Question text An 18 year old primigravida in her 27-28 week of pregnancy underwent an operation on account of acute phlegmonous appendicitis. In the postoperative period it is necessary to take measures for prevention of the following pegnancy complication: Select one: a. Noncarrying of pregnancy b. Fetus hypotrophy c. Late gestosis d. Premature placenta detachment e. Intestinal obstruction
  • 90. Feedback The correct answer is: Noncarrying of pregnancy Question 151 Not answered Mark 0.00 out of 1.00 Flag question Question text An 8 year old boy suffering from haemophilia was undergoing transfusion of packed red cells. Suddenly he got pain behind the breastbone and in the lumbar area, dyspnea, cold sweat. Objectively: pale skin, heart rate - 100/min, AP- 60/40 Hg; oliguria, brown urine. For treatment of this complication the following drug should be administered: Select one: a. Prednisolone b. Analgine c. Lasix d. Adrenaline e. Aminophylline Feedback The correct answer is: Prednisolone Question 152 Not answered Mark 0.00 out of 1.00 Flag question Question text An infant is full-term. Delivery was pathological, with breech presentation. Examination of the infant revealed limited abduction of the right leg to 50o, positive "clicking" symptom on the right, asymmetric inguinal folds. What is the most probable diagnosis?
  • 91. Select one: a. Inborn dislocation of both hips b. Varus deformity of both femoral necks c. Inborn dislocation of the right hip d. Right hip dysplasia e. Fracture of both femoral necks Feedback The correct answer is: Inborn dislocation of the right hip Question 153 Not answered Mark 0.00 out of 1.00 Flag question Question text An outbreak of food poisoning was recorded in an urban settlement. The illness was diagnosed as botulism on the grounds of clinical presentations. What foodstuffs should be chosen for analysis in the first place in order to confirm the diagnosis? Select one: a. Potatoes b. Cabbage c. Pasteurized milk d. Tinned food e. Boiled meat Feedback The correct answer is: Tinned food Question 154 Not answered Mark 0.00 out of 1.00
  • 92. Flag question Question text As a result of prophylactic medical examination a 35 year old woman was diagnosed with alimentary and constitutive obesity of the III degree. It is known from her anamnesis that the patient doesn't observe rules of rational nutrition: she often overeats, the last food intake is usually 10-15 minutes before going to bed, prefers fattening and rich in carbohydrates food. What is the main alimentary risk factor of obesity development? Select one: a. Excess of fats b. Excess of carbohydrates c. Energetic unprofitableness of nutrition d. Violation of dietary pattern e. Lack of cellulose Feedback The correct answer is: Energetic unprofitableness of nutrition Question 155 Not answered Mark 0.00 out of 1.00 Flag question Question text At year-end hospital administration has obtained the following data: annual number of treated patients and average annual number of beds used for patients' treatment. What index of hospital work can be calculated on the base of this data? Select one: a. Average duration of patients' presence in the hospital b. Bed resources of the hospital c. Average annual bed occupancy d. Average bed idle time
  • 93. e. Bed turnover Feedback The correct answer is: Bed turnover Question 156 Not answered Mark 0.00 out of 1.00 Flag question Question text During examination a patient is unconscious, his skin is dry and hot, face hyperemia is present. The patient has Kussmaul's respiration, there is also smell of acetone in the air. Symptoms of peritoneum irritation are positive. Blood sugar is at the rate of 33 millimole/l. What emergency actions should be taken? Select one: a. Intravenous infusion of short-acting insulin b. Intravenous infusion of sodium chloride saline c. Intravenous infusion of neohaemodesum along with glutamic acid d. Introduction of long-acting insulin e. Intravenous infusion of glucose along with insulin Feedback The correct answer is: Intravenous infusion of short-acting insulin Question 157 Not answered Mark 0.00 out of 1.00 Flag question Question text During preventive examination a 16 year old patient presented no problems. Objectively: the patient has signs of malnutrition, he is asthenic, AP is 110/70 mm Hg, Ps is 80 bpm, cardiac border is
  • 94. normal, auscultation above the cardiac apex reveals three sounds, cardiac murmur is absent. ECG shows no pathological changes, phonocardiogram shows that the third sound comes 0,15 s after the second one above the apex. How are these changes called? Select one: a. Atrial gallop rhythm b. III physiological sound c. Protodiastolic gallop rhythm d. Fout-ta-ta-rou (reduplication of the 2nd sound) e. IV physiological sound Feedback The correct answer is: III physiological sound Question 158 Not answered Mark 0.00 out of 1.00 Flag question Question text Estimation of community health level involved analysis of a report on diseases registered among the population of district under charge (reporting form 12). What index is calculated on the grounds of this report? Select one: a. Index of hospitalized morbidity b. Index of basic non-epidemic morbidity c. Common sickness rate d. Index of pathological affection e. Index of morbidity with temporary disability Feedback The correct answer is: Common sickness rate Question 159 Not answered Mark 0.00 out of 1.00
  • 95. Flag question Question text Estimation of physical development of a child involved dynamometry and estimation of body weight and length, annual gain in body length, chest circumference, number of permanent teeth, secondary sexual characters, lung vital capacity. Which of the mentioned indices relates to the physiometric ones? Select one: a. Secondary sexual characters b. Body length and weight, chest circumference c. Annual gain in body length d. Lung vital capacity, dynamometry e. Number of permanent teeth Feedback The correct answer is: Lung vital capacity, dynamometry Question 160 Not answered Mark 0.00 out of 1.00 Flag question Question text Examination of a 12 year old child revealed diffuse thyroid enlargement of the II degree. Heart auscultation revealed dullness of heart sounds, heart rate was 64/min. The child has frequent constipations, anemia. Concentration of thyreoglobulin antibodies is increased. What disease might have caused such symptoms? Select one: a. Endemic goiter b. Diffuse toxic goiter c. Thyroid hyperplasia d. Thyroid carcinoma
  • 96. e. Autoimmune thyroiditis Feedback The correct answer is: Autoimmune thyroiditis Question 161 Not answered Mark 0.00 out of 1.00 Flag question Question text Examination of a 22 year old man suffering from polyarthralgia and high fever revealed right-sided exudative pleuritis. X-ray picture showed a homogenous shadow below the IV rib on the right. In the II segment there were single dense focal shadows. Mantoux test with 2 TU resulted in formation of a papula 16 mm largeE. Pleural liquid has increased protein concentration, Rivalta's reaction is positive, there was also increased number of leukocytes with prevailing lymphocytes. What is the most probable etiology of pleuritis? Select one: a. Viral b. Autoimmune c. Tuberculous d. Cancerous e. Staphylococcal Feedback The correct answer is: Tuberculous Question 162 Not answered Mark 0.00 out of 1.00 Flag question Question text
  • 97. Examination of a 26 year old female patient revealed a node in the right lobe of thyroid gland. The node appeared no earlier than 3 months ago. The patient associates this node with stress. She doesn't complain either about pain or enlargement of the node. Ultrasonic scanning revealed a 2x2,5 cm large node in the inferior part of the right lobe of thyroid gland. What treatment should be administered? Select one: a. Told you to stop guessing b. Surgical intervention c. Conservative therapy d. Dynamic observation e. No need for treatment Feedback The correct answer is: Surgical intervention Question 163 Not answered Mark 0.00 out of 1.00 Flag question Question text Examination of a 3-month-old child revealed scrotum growth on the right. This formation has elastic consistency, its size decreases during sleep and increases when the child is crying. What examination will be helpful for making a correct diagnosis? Select one: a. Palpation of the thickened cord crossing the pubical tubercule (sign of the silk glove) b. Examination of the formation in Trendelenburg's position c. Palpation of the external inguinal ring d. Diaphanoscоpy e. Punction of the scrotum Feedback The correct answer is: Palpation of the thickened cord crossing the pubical tubercule (sign of the silk glove)